[Ответить в тред] Ответить в тред



[Назад][Обновить тред][Вниз][Каталог] [ Автообновление ] 987 | 104 | 247
Назад Вниз Каталог Обновить

Клуб любителей изучать PHP #88 Аноним 04/04/17 Втр 14:08:00  966608  
14901370728420.jpg (102Кб, 500x500)
Добро пожаловать в наш уютный тред. Тут мы изучаем язык PHP (а также JS/CSS/HTML/SQL), решаем задачки и даже делаем простые сайты! Зачем? Кто-то хочет научиться программировать, кто-то - делать сайты, кто-то - просто размять мозги и заняться чем-то полезным.

Пожалуйста, пишите один большой пост вместо нескольких маленьких и не флудите не по теме. ОПу ведь все это читать придется.

Это тред для начинающих. Не написал за свою жизнь ни одной программы и имеешь тройку по математике? Ты наш человек.

Устанавливать пока что ничего не требуется, разве что редактор кода вроде Sublime Text 3, Notepad++, Visual Studio Code, Netbeans PHP или PhpStorm (с ним будет удобнее).

Что самое главное для программиста? Умение аккуратно оформлять код (читай второй пост, прежде чем писать код).

Правила: ведем себя воспитанно, помогаем новичкам, постим ссылки на решения задачек, ОП их проверяет и дает советы и замечания. ОП заходит редко, где-то раз в 2-3 дня, у него мало времени, не жди его, решай задачки дальше. ОП отвечает на все вопросы по его задачкам и учебнику, а вот насчет каких-то других вещей - только если останется время. Но в треде немало анонимных экспертов разного уровня, так что вряд ли вопрос останется без ответа.

У нас есть уроки по основам PHP, они собраны и выложены по адресу http://archive-ipq-co.narod.ru/ Это учебник для изучающих с нуля, то есть если ты вообще ничего не знаешь, то надо начать с него. Он простой и понятный (по крайней мере в начале). Там есть задачи, их надо решать обязательно (чтобы стать программистом, надо писать код — иначе никак). Пости ссылки на решения в тред, мы их проверим, напишем замечания и дадим советы по улучшению.

Если не знаешь как решать, запости код, напиши в каком месте остановился и попроси подсказку.

Ты прошел весь учебник? Молодец, но это были лишь основы языка PHP, этого недостаточно. Вот что в идеале надо изучить еще: ООП, как работает веб-сервер, HTML/CSS, SQL, PDO, работа с таблицами в БД, работа с формами, MVC, git, composer, JS, фреймворки, автоматизированное тестирование.

Надо переходить к более серьезным задачкам, которые научат тебя всему этому.

- для начала прочти урок https://github.com/codedokode/pasta/blob/master/soft/web-server.md
- установи Апач + PHP (советы выше и ниже) и читай туториал http://php.net/manual/ru/tutorial.php
- Учи HTML/CSS и SQL, PDO, хотя бы основы
- Далее простая, но полезная задача сделать список студентов, в ней много полезных советов: https://github.com/codedokode/pasta/blob/master/student-list.md
- Более сложная задача сделать файлообменник на микрофреймворке Slim: https://gist.github.com/codedokode/9424217
- Еще более сложная и долгая задача на Yii/Symfony: https://gist.github.com/codedokode/8733007
- После нее можно изучать автоматизированное тестирование https://gist.github.com/codedokode/a455bde7d0748c0a351a
- Если ты все решил, переходи к Symfony 2/Doctrine 2
- Почитать про паттерны http://designpatternsphp.readthedocs.org/ru/latest/README.html (если ты не изучил ни одного фреймворка, то это будет рановато), тут с примерами кода http://designpatternsphp.readthedocs.org/ru/latest/README.html . Имей в виду что без примеров использования их учить бесполезно - не поймешь, хочешь увидеть примеры использования паттернов - ковыряй исходники Симфони, например Symfony Forms. Не заучивай паттерны - смотри код и думай, зачем тут они использованы.

Чтобы делать эти задания, тебе надо установить Апач + PHP (можно заодно сразу и MySQL) на компьютер. Вот полезные инструкции:

https://github.com/codedokode/pasta/blob/master/soft/php-install.md
https://github.com/codedokode/pasta/blob/master/soft/apache-install.md

Может тебе понадобится пользоваться командной строкой, вот гайд https://github.com/codedokode/pasta/blob/master/soft/cli.md

Решения задач лучше показать мне, особенно на ООП,так как сам ты вряд ли увидишь все ошибки. Пости свой код на гитхаб и вкидывай ссылку в тред по мере решения. Я прокомментирую и укажу на ошибки.

Также, у нас есть задачи которые позволят тебе изучить или подтянуть до нормального уровня знания JS/HTML/CSS/SQL. Решай их параллельно с задачами выше.

- HTML/CSS: https://github.com/codedokode/pasta/blob/master/html/html.md
- JS: https://gist.github.com/codedokode/ce30e7a036f18f416ae0
- SPA (сложно): https://github.com/codedokode/pasta/blob/master/js/spa.md
- Проверялка решений на JS: http://dkab.github.io/jasmine-tests/
- MySQL: https://github.com/codedokode/pasta/blob/master/db/databases.md

Что почитать

- Мануал по PHP — http://www.php.net/manual/ru/langref.php
- Сайт phptherightway (перевод на русский: http://getjump.me/ru-php-the-right-way/ )
- По PHP: Профессиональное программирование на PHP Джордж Шлосснейгл
- По PHP: Мэтт Зандстра — PHP: Объекты, шаблоны, методики программирования
- JS: learn.javascript.ru
- Про Git: https://git-scm.com/book/ru/v1

Оформляй код аккуратно!!! — например пропусти через phpformatter.com . Также, если ты пользуешься IDE вроде PhpStorm, Netbeans, Eclipse, то в них эта опция встроена, подробнее: https://gist.github.com/codedokode/8759492

У ОПа нет аккаунтов и групп вконтакте, в фейсбуке, в твиттере, все "пхп-треды" там поддельные.

Платиновые вопросы

- Почему PHP? Потому что фейсбук и википедия на нем написаны, и вакансий море, и учить легко.
- Сайт опять упал!!!!! — Не паникуй, а открой http://rghost.ru/6bfCY9lfl и получи личную немного устаревшую оффлайновую копию сайта (можно читать хоть на андроиде без интернета)
- Что надо знать чтобы найти работу - разработчику: PHP, SQL, HTML/CSS, JS, ООП, Git, композер, MVC, фреймворк. Верстальщику - HTML/CSS, JS, jQuery
- Можно подробнее про поиск работы, собеседования - нет, ОП писать не будет, но может кто из анонов захочет рассказать. Поищите тред перезвонивших, а также раздел /wrk/.
- Сколько времени надо изучать все это? - все зависит от тебя, но не меньше 6-8 месяцев
- Посоветуйте редактор кода - Sublime Text 3, Notepad++, PhpStorm
- Нужен ли ООП, фреймворки, MVC, git, composer? — Да, однозначно. Посмотри любую вакансию.
- Что самое главное для программиста? Умение аккуратно оформлять код.
- ОП, сделай за меня мою работу или домашнее задание? — Это конечно, хорошая идея, но нет.
- Подскажи сайты для поиска работы, я не умею гуглить? — hh.ru, geekjob.ru, moikrug.ru (склеен с brainstorage.me), fl.ru, upwork.com (бывший одеск). Имей в виду, что кроме фриланса есть еще постоянная удаленная работа (remote job) когда тебе не надо тратить время на поиск заказов и переговоры с неадекватными заказчиками.


-------------------

Код нужно писать не как попало, а аккуратно и по правилам. Почему? Потому, что на неакуратно написанный код не хочется даже смотреть.

Если тебе лень выравнивать код руками, закачай его на http://beta.phpformatter.com/ и нажми «format». Робот исправит выравнивание и отступы в мгновение ока (да, прогресс не стоит на месте). Если ты используешь мощную IDE вроде PhpStorm, там тоже есть функция форматирования кода.

Горячие клавиши для форматирования кода в разных IDE: https://gist.github.com/codedokode/8759492

Вообще, в PHP долгое время не было единого стандарта оформления кода, все писали как попало и было много бардака, но сейчас дело лучше — есть стандарты PSR-1 и 2. Вот как надо оформлять код:

- переменные и функции пишутся с маленькой буквы, подчеркивание не используется, используется camelCase, пример: $x, $numberOfPeople, printResults()
- Название функции начинается с глагола, в стиле «сделайЧтоТо»
- не знаешь английский? Не беда, в 21 веке есть решение этой проблемы. Не пиши транслитом, открой лучше Гугл Транслейт или slovari.yandex.ru и найди название для переменной там
- в именах классов используется CamelCase, первая буква большая, «_» может использоваться
- мы предпочитаем подстановку переменных вместо конкатенации строк: "I am $age years old" — хорошо, 'I am ' . $age . ' years old' — плохо из-за обилия точек и кавычек
- мы используем для отступов 4 пробела (можно настроить редактор, чтобы при нажатии Tab он вставлял 4 пробела)

Вот ссылка на стандарты, где все это описано подробнее и даны примеры оформления:

PSR-1: https://github.com/samdark/fig-standards-ru/blob/master/accepted/ru/PSR-1-basic-coding-standard.md
PSR-2: https://github.com/samdark/fig-standards-ru/blob/master/accepted/ru/PSR-2-coding-style-guide.md
Аноним 04/04/17 Втр 14:30:13  966622
ОП, ты вроде шарил в Symfony.
Нужно получать кол-во новых комментов к статье по последнему просмотру ее юзером.
Таблица последнего просмотра - article_id, user_id, timestamp; Сущность ActicleLastView.

Сущность Atricle, сериализация через JMS.

Как по уму сделать подсчет кол-ва комментариев появившихся после timestamp по заданному юзеру? Вытаскивать QueryBuilder через leftJoin? Как замэпить полученное число в сущность Article?
Аноним 04/04/17 Втр 15:31:31  966660
>>966622
>Нужно получать кол-во новых комментов к статье по последнему просмотру ее юзером.
Типо, посмотрел коммепнты, поставил временной маркер, а при следующем просмотре просто подгружаешь, комменты которые появились после временни отмеченного временным маркером?

Скинь код и скрин и диаграмму базы.
Аноним 04/04/17 Втр 16:23:44  966678
Анон, подскажи, пожалуйста, готовое решение "long polling сервера на php" (если таковое имеется).
Интересует не простейший пример, коих в инете полно. А работоспособное, проверенное решение, которое может держать пару тысяч юзеров, раздавая им свежую информацию.
Аноним 04/04/17 Втр 17:10:54  966706
>>966660
В целом верно, только кол-во новых показывается в списке, типа стикера с кол-вом новых комментариев в статье после последнего просмотра.

Когда юзер открывает статью - пишу временную метку в ActicleLastView (для таблицы primary key => user_id + article_id).

Потом в списке нужно как-то получить для текущего юзера кол-во комментов оставленных в статье после этой временной метки.

В принципе я уже решил вопрос двумя leftJoin (ActicleLastView, Comments) с кучей условий джойна + groupBy(article_id), но хочется понять, есть ли более элегантное решение, например, с запросами к отношениям как в laravel.

Кстати, вызывают ли @ORM\OneToMany(targetEntity="AppBundle\Entity\Comment", mappedBy="application", orphanRemoval=true, cascade={"remove"}) дополнительные подзапросы при обращении к свойствам или вытягиваются при любом запросе (в т.ч. и через QB)?
Аноним 04/04/17 Втр 17:29:02  966717
>>966706
Я бы сделал так.
Смотрим комменты.
Записываем номер последнего комментария.
Все не просмотренные комменты - это все комменты у которых собственный номер больше, чем номе последнего коммента.
Чтобы посчитать кол-во их используй конструкцию.
getQuery()->getSingleScalarResult(); Она тебе сразу цифру даст.

Тяжело обьяснять и понимать, кода не видно самой бд. Хуй знает как у тебя там таблицы организованы, какая нормальная форма и тд.

>Кстати, вызывают ли @ORM\OneToMany(targetEntity="AppBundle\Entity\Comment", mappedBy="application", orphanRemoval=true, cascade={"remove"}) дополнительные подзапросы при обращении к свойствам или вытягиваются при любом запросе (в т.ч. и через QB)?

Вроде, когда обьект удаляется, он смотрит обьекты связанные с ним по ссылке и тоже их удалят, чтобы не оставалось "висящих" ссылок, которые никуда не указываю.

Аноним 04/04/17 Втр 19:20:03  966783
>>966744
Когда переделаете в супергруппу, тогда и зайду.
Аноним 04/04/17 Втр 19:27:37  966792
>>966744
Это Оп?
Аноним 04/04/17 Втр 20:26:12  966850
>>966792
Нет.
Аноним 04/04/17 Втр 20:41:20  966864
Бля анон, я знаю ты скажешь я тупой, но я не могу исправить эту ебаную ошибку в проге про айфон и кредит!! Пиздец у меня горит уже просто. http://archive-ipq-co.narod.ru/l1/loops.html Дай подсказку, пожалуйста!
Аноним 04/04/17 Втр 21:22:04  966900
>>966864
Не переживай, ты не первый сюда с таким вопросом приходишь. Я тоже спрашивал где-то неделю назад.
Пример кода к задаче, который даёт ОП, слегка сбивает с толку. Перепиши весь цикл заново, попробуй решить с помощью while.
Аноним 04/04/17 Втр 22:00:49  966920
http://ideone.com/H6PIAO УРА, АНОН! Я же не говнокод сделал,да? (Задача с айфоном в кредит_ http://archive-ipq-co.narod.ru/l1/loops.html)6
Аноним 04/04/17 Втр 22:05:19  966928
>>966920
Было бы где наговнокодить, там меньше 100 строчек
Аноним 04/04/17 Втр 22:08:40  966929
2a484983e0c09a8[...].png (154Кб, 1480x809)
Не могу зарендерить форму (Symfony Form Builder), код https://github.com/Si0n/fileshare-project . Twig и FormBuilder подключаются в dependency.
Аноним 04/04/17 Втр 22:13:13  966933
>>966929
Рендерить пытаюсь в Controller/Home.php
Аноним 04/04/17 Втр 22:16:30  966939
>>966920
А если изменить выплаты будут продолжаться дольше 20 месяцев?
Аноним 04/04/17 Втр 22:19:08  966943
>>966929
Я думал над этим. Хочу вот сейчас сделать так, что бы останавливался цикл когда баланс уже 0.
Аноним 04/04/17 Втр 22:19:31  966944
>>966929
>>966939
Промахнулся
Аноним 04/04/17 Втр 22:42:26  966957
>>966929
Нашел в чем проблема: не было
Аноним 04/04/17 Втр 22:57:22  966976
6e720b99924ed2f[...].png (216Кб, 1605x996)
>>966957
Зато появилась другая: шаблон взял с документации:
https://github.com/Si0n/fileshare-project/blob/master/App/view/form-default.twig
Аноним 04/04/17 Втр 23:01:42  966982
Снимок.PNG (20Кб, 656x267)
Анон, сломал голову об такую мелочь. Есть один код (на пикрил он же):

{
"work_status": [{
"id": 293,
"received": "2016-11-12T05:40:09.372Z",
"status_current": "suspended", // rejected suspended finished
"status_date": {
"rejected": false,
"suspended": "2016-11-12T05:40:09.372Z",
"finished": false
}
}]
}

Длинный json с таким контентом.
В php у меня j,sxysq парсер для вывода этого всего через foreach. Считаю общее количество работы вот так <?php echo count($json['work_status']) ?>. Даже сумел сортировать рабjту по status_current и выводить её по запросу одного из 3х статусов, но как ни пытался, не получается подсчитать и вывести, сколько всего
status_current suspended, к примеру.
Ну по логике я попробовал конечно
<?php echo count($json['work_status']['status_current'] == 'suspended') ?> или <?php echo count($item['status_current'] == 'suspended') ?> но нифига.
Аноним 04/04/17 Втр 23:34:05  967014
>>966982
array_sum(array_map(function ($w) { return $w['status_current'] == 'suspended'; }, $json['work_status']))
Аноним 04/04/17 Втр 23:37:55  967018
Такой трабл: в БД лежит инфа о юзере, то есть его логи, пассворд и т.п. Пароль естественно прошел через bcrypt().
На аппликухе на андроиде есть логин, который посылает мне в АПИ данные. Но при сравнении двух паролей генерируется разная хеш-функция, и в итоге проверить на равность неполучается. Есть какие-то способы это сделать? Laravel 5.4
Аноним 04/04/17 Втр 23:40:10  967022
>>967014
Ну и сложности. Я даже и близко не рыл в эту сторону. Спасибо анон, всё работает как надо. Теперь пойду разберу на части и выясню, почему это работает.
Аноним 04/04/17 Втр 23:47:35  967026
125677.png (160Кб, 1920x1028)
>>966944
Анон, помоги с регулярными выражениями. Никак не получается написать шаблон. Вот что у меня есть - $regexp='/[(\+7)|8][0-9]{10}/';. Не понимаю как сюда впихнуть условие на любое количество скобок, пробелов или минусов.
Аноним 04/04/17 Втр 23:48:17  967027
>>967026
Не туда ответил.
Аноним 04/04/17 Втр 23:56:00  967038
>>966943
Если подумать как следует, то задача становится легкой. Тебе всего лишь надо написать:
Пока кредит больше нуля - считаем общую сумму с учетом сервиса и процента. Если эта сумма больше 5000, то вычитаем из неё 5000 и прибавляем эти 5000 к $total, иначе вычитаем из неё остаток кредита и прибавляем эту сумму к $total. Всё.
Аноним 04/04/17 Втр 23:56:20  967039
>>967026
^((8|\+7)[\- ]?)?(\(?\d{3}\)?[\- ]?)?[\d\- ]{7,10}$
Аноним 04/04/17 Втр 23:59:24  967041
>>967039
Не проходит тест:(.
Аноним 05/04/17 Срд 00:01:50  967044
>>967041
http://ideone.com/yAuKgH
Аноним 05/04/17 Срд 00:06:50  967046
>>967038
>иначе вычитаем её остатка кредита и прибавляем эту число к $total.
Фикс
Аноним 05/04/17 Срд 00:07:46  967047
>>967046
Охуенно пофиксил. Пора, наверное, спать идти.
Аноним 05/04/17 Срд 00:09:12  967048
Я в задании на числа прописью чот какую-то ересь написал, но работает, посоветуйте, куда идти (https://3v4l.org/jOnfA).

Удалил все ОПовские $amount1 = mt_rand(1,99999999);, кроме одной, т.к. eval все равно гоняет по всем версиям php.
Аноним 05/04/17 Срд 00:10:12  967050
>>967048
ссылка без скобки должна быть https://3v4l.org/jOnfA
Аноним 05/04/17 Срд 00:28:56  967058
>>966744
Теперь мы не просто группа, а публичная супергруппа , прости Господи! Добро пожаловать!:
https://t.me/PHP_club
Аноним 05/04/17 Срд 00:31:42  967060
>>967022
count() не принимает параметром что-то сложнее массива, никаких условий, функций и тому подобного.
array_map() принимает функцию и массив, потом применяет функцию к каждому элементу и возвращает новый массив. То есть после её применения тебе вернется массив вроде [1, 1, 0, 1, 0, 0, 0, 0, 1, 0, 1] или с булинами, хз.
С array_sum() думаю уже сам сообразишь как и что.
Аноним 05/04/17 Срд 01:20:39  967077
tt.jpg (30Кб, 450x250)
>>967058
Success!
Аноним 05/04/17 Срд 10:20:56  967147
Как сделать html-форму с выпадающим списком, который предлагает варианты в зависимости от выбора в предыдущем выпадающем списке?
Аноним 05/04/17 Срд 10:33:40  967157
>>967147
Метод, у которого в аргументы будет передаваться массив $previousDropDown, что-то такое. При этом в нём будет создаваться новый $previousDropDown - из только что выбранного.
Аноним 05/04/17 Срд 10:39:40  967159
>>967147
Js+ Ajax
Аноним 05/04/17 Срд 13:29:28  967240
>>967060
Вчера лёг спать до этого поста, а утром читал про array_map и пробовал примерчики. Но это >>967022 как-то слишком ювелирно выглядит, толком в логику не вник - с толку сбивала функция и ретурн.
Вот спасибо за разжёвывание.
Аноним 05/04/17 Срд 13:30:21  967241
>>967240
>Но это
>>967014 это.
ниндзяфикс
Аноним 05/04/17 Срд 13:43:42  967259
А что, расширения для пхп 7.0 могут уже не подойти к 7.1? Такая большая разница или особенность какая-то?
Аноним 05/04/17 Срд 13:47:39  967265
Бамп вопросу про MVC >>966100 из прошлого треда.
Аноним 05/04/17 Срд 14:09:44  967295
Объясните мне, что делает strpos. Я прочёл мануал, но не понял.
Аноним 05/04/17 Срд 14:16:34  967302
>>967295
Ищет строку в строке и возвращает позицию.
strpos("qwe asd zxc rty fgh vbn", "fgh"); => 16
Аноним 05/04/17 Срд 14:31:37  967318
И ещё, где можно подробно и с примерами почитать про вёрстку под мобильные устройства?
Аноним 05/04/17 Срд 14:36:56  967323
>>967265
Для начала поясни зачем тебе там вообще жаваскрипт и что за проект такой. Про echo и то как пхп-скрипт должен отдавать аякс-ответ - ну блин, если ты уж и пользуешься технологией то знать должен.
Аноним 05/04/17 Срд 14:39:08  967325
Сделал наконец задачу с регулярными выражениями - http://ideone.com/yAuKgH. Подскажите можно ли сделать это задание проще и лучше? А то у меня есть ощущение, что я говнище какое-то сделал.
Аноним 05/04/17 Срд 14:39:40  967326
>>967325
Точка прилипла.
http://ideone.com/yAuKgH
Аноним 05/04/17 Срд 14:55:57  967338
МДА...
Аноним 05/04/17 Срд 15:09:41  967347
>Only variables should be passed by reference
Разверните суть сего посыла в целом плз.
Аноним 05/04/17 Срд 16:05:10  967375
>>967325
Избавься от повторов:
>*[\d][\s-()]
Аноним 05/04/17 Срд 16:30:00  967385
>>967323
>Про echo и то как пхп-скрипт должен отдавать аякс-ответ - ну блин, если ты уж и пользуешься технологией то знать должен.
Когда я читал про аякс, там было сказано только, что он возвращает данные одним куском через echo.

Проект следующий: есть большая таблица (типа экселевской) с данными. В каждой строке информация об отдельном заказе(кто, что, когда, и т.д.). Её вносит менеджер через браузер. А тот, кто имеет дело с конкретным заказом, должен внести некоторые данные (например, серийный номер и точное время принятия). Для этого он заходит на сайт, авторизуется и видит календарь, на котором отмечены дни, в которые у него запланированы заказы. Кликает по любому из них - появляется модальное окно. Сверху инфа о заказе, вынутая из таблицы в БД, снизу поля для ввода инфы.

JS нужен всю дорогу, как же без него?
Аноним 05/04/17 Срд 16:35:11  967389
>>967385
В данном случае - тебе надо обычное MVC-приложение, такое можно сделать за пару часов. Так и не понял зачем тебе там аякс, только лишнее усложнение.
Аноним 05/04/17 Срд 16:36:41  967392
>>967389
>Кликает по любому из них - появляется модальное окно. Сверху инфа о заказе, вынутая из таблицы в БД, снизу поля для ввода инфы.
А, епты, не дочитал. Вообще такое лучше в фреймворке делать, там все вообще просто.
Аноним 05/04/17 Срд 16:40:26  967394
Делаю файлообменник, нихуя не понял как отдавать файлы из не публичной папки. Из того что пишет ОП - думаю сделать вариант отдавать файлы приложением на PHP, только я чегот не понял как это делается. Вот я хочу например превьюшки для картинок делать. Надо запилить, ф-ю и вызывать ее в шаблоне? А возвращает она все эти ебучие заголовки?
Аноним 05/04/17 Срд 16:49:26  967397
>>967389
>тебе надо обычное MVC-приложение
Да блин. Я НЕ ПОНИМАЮ, как это грёбаное обычное приложение сделать. Вернее, сделать-то я могу, но вот что бы MVC и по фен-шую - не понимаю. Вот конкретно. Написано, что модель не должна срать в эхо. Не должна, блядь, и всё тут. Но как она передаст обратно в контроллер данные? У меня этим занимается аякс. Получается, айск выкинуть? Они там совсем охуели? Что происходит? По идее, паттерн (любой) - это некой HOW TO, призванное облегчать разработку, поддержку и использование. А тут что? Набор гнуси, вот это что такое. Причём переусложнённой и запутанной. Я уже штук пять разных описаний MVC для веба прочитал и все они разные.


Аноним 05/04/17 Срд 17:52:30  967430
>>967397
>что модель не должна срать в эхо. Не должна, блядь, и всё тут
Ну правильно, модель должна что-то возвращать в контроллер, а контроллер уже передаст данные в твой скрипт, что непонятного то?
Аноним 05/04/17 Срд 18:03:24  967431
>>967430
Как конкретно модель возвращает данные?
Аноним 05/04/17 Срд 18:04:50  967432
>>967431
Ты прикалываешься?
Аноним 05/04/17 Срд 18:46:27  967454
>>967431
Через EventDispatcher кидаешь событие, а сервис перехватывает и записывает их в статический объект, за которым в режиме бесконечного лупа следит контроллер
Аноним 05/04/17 Срд 18:47:44  967457
>>967431
Ну а если ты серьезно не в курсе, то гугли return.
Аноним 05/04/17 Срд 18:48:08  967458
>>967394
Ну хз как надо, я это сделал в виде 1 индекс.пхп и 1-й папки рядом. Ну типа юзер через форму загружает файл на сервер и этот файл становится тут же виден на страничке и его можно скачать. Если это картинка, то можно сделать валидацию на это дело и выдавать в обёртке <img>. Без фреймворка всё отлично работает. Теперь осталось как-то усложнить всё в MVC.
Аноним 05/04/17 Срд 18:57:30  967466
>>967458
>MVC
Slim
Аноним 05/04/17 Срд 19:52:38  967498
ОП-чан нас окончательно покинул?
Аноним 05/04/17 Срд 20:20:56  967515
>>967375
Как? Я правда не понимаю.
Аноним 05/04/17 Срд 20:25:27  967518
>>967515
Лол. Получилось. $regexp='/^ ?(\+ ?7|8)([- ()]*\d){10}$/m';
Аноним 05/04/17 Срд 20:53:32  967527
Как оно работает из терминала? Он запускает какой-то локальный сервер или этот сервер постоянно работает?
Я думал, для того чтобы оно заработало в терминале, надо поставить апач,а оно и без апача работает. Объясните, пожалуйста этот момент.
Аноним 05/04/17 Срд 21:28:36  967563
>>967527
"Оно" это что? Фреймворк, файлик? Если ты про интерпритатор php то для него никакой сервер не нужен.
Аноним 05/04/17 Срд 21:38:31  967575
>>967563
Я вот про интерпритатор, да. А как он не нужен-то?
Аноним 05/04/17 Срд 22:23:22  967659
>>967575
> А как он не нужен-то?
Так для IDE, например. Где достаточно php.exe и указать к нему путь. Там же и консолька. Смотря что и для чего оно тебе нужно.
Аноним 05/04/17 Срд 22:27:02  967666
Я нуб и столкнулся с тем, что фор и форич могут одно и то же. Так во что углубляться при изучении, если например я могу реализовать функцию и с тем и с этим?
Аноним 05/04/17 Срд 22:29:35  967671
>>967666
>функцию
...в смысле работающую одинаково, одну и ту же.
Аноним 05/04/17 Срд 22:46:21  967683
14884107863140.jpg (37Кб, 604x604)
бананы, кто осваивал Ларавель, легко вам давалось сие? я вот в самом начале. Посмотрел сейчас урок про мидлваре и почувствовал себя дебилом как похоже и есть
Аноним 06/04/17 Чтв 01:41:39  967795
>>967498

ОП сидит в основном треде: >>958715 (OP)

Если у тебя вопросы к ОПу или по каким-то задачам ОПа или хочется проверить решение, то стоит запостить хотя бы ссылку на пост в тот тред, ближайшие несколько дней ОП точно будет там, не тут.
Аноним 06/04/17 Чтв 02:11:16  967809
>>967683
Уже 5 лет пишу на пхп и последние 2 года использую ларавель в своих проектах. В милдварях нет ничего сложного.
Аноним 06/04/17 Чтв 03:03:46  967815
>>967809
Чем ларавель лучше yii2, чому многие прогеры на него переходят, особенно для своих проектов? Я смотрю уроки по yii - ну вроде заебок, всё чётко, понятно, gii хорош.
А что в ларавеле так привлекает?
Аноним 06/04/17 Чтв 04:39:04  967820
>>967815
Ну как бы со стороны архитектуры - уии полное говно. Пройдись по классам и дойди до классов Object \ Component , там лютый трешак. Или посмотри на govnokod.ru . Ларавел также может и покруче создать админку причем более изи см SleepingOwl. Есть крутой DI (IoC container) в отличии от галимого локатора в уии. В уии идут после того как написал на пхп "хелло ворд я вася 2+2=4" но порог входа в Ларавел будет совсем иной. Ну и еще как бы ларку на сколько знаю то финансируют т.е есть инвестиции а уии пишется в свободное время и то судя по репозиторию там все очень плохо ( посмотри сколько пр висит даже), я сам контрибьютил немного в ларку и там как то нормально принимают, в уии аж стремно чет пр кидать сразу понимаешь что упустят (хз). Но есть + в уии что раб сила очень дешевая и создать типичный говносайт не дорого на нем. Но для чего то серьезнее говносайта я категорически против уии. (ору с конфигов в адванседе)
Аноним 06/04/17 Чтв 04:45:03  967822
MalikYobacroppe[...].jpeg (175Кб, 876x1228)
Сап, программач. Долго гуглил, читал мануал PHP, но всё ещё задаюсь вопросом: как можно обезопасить сайт от инклуд-иньекции или чего-нибудь подобного, если атака происходит через залив файла?

Поясню, что я нашел:
- Отфильтровать расширения файлов, и тогда не смогут залить какой-нибудь yobaskriptxss.php
Резонно. Но я тут провел эксперимент:
>написал в блокноте <?php echo '<img src="dir://yoba.jpg">';?>
>сохранил это в формате .jpg
>залил на сайт через форму
>заинклудил этот jpg на главную страницу
>выдало пикрелейтед

-Так проверь MIME!
Проверял. На уровне post-запроса всё заебись. Вот только где-то я вычитал, что этот самый mime легко подделать.
Т.е., даже getimagesize() иногда за милую душу хавает запрятанный php-код!

Может, нужно как-то аплоудить, проверять файл, а потом его удалять в той же итерации, если он какой-то не правильный? А что, если там скриптина гадкая, которая всё мне сломает нахуй?
Помоги, анон!
Аноним 06/04/17 Чтв 05:39:22  967824
>>967815
Юии это что-то спизженное с асп дот нета, а ларавель это скорее спизженные рельсы.

Лично мне сразу не понравился кодстайл юии и я толком его даже не ковырял и никаких серьезных прожектов на нем не мутил. Ларавел же дал нормальный кодстайл, симпатичный сайт, толковый хелп (но всё же не всё покрывает, но сорцы поглядеть и понять не проблема), всякую хуйню вроде кучи блогов, кастов, плугинов для редакторов итп. Фрилансил на нем без проблем.

Но потом я прозрел и перешел на слим, а сейчас вообще частично перевел некоторую свою хуйню на опенрести
Аноним 06/04/17 Чтв 05:41:41  967825
>>967822
Выноси аплоады в отдельную папку или на отдельный поддомен который про пхп ничего не знает, а просто отдаёт файлы.
Аноним 06/04/17 Чтв 07:58:52  967836
>>966853

Возьми пока на пробу вот это, тут тредов 10-20 с картинками:

- http://www.mediafire.com/file/e8jt8v4fny3m5sk/threads-archive-66-72.zip
- http://www.mediafire.com/file/urj17w5s4w00jqc/threads-60-80.zip
- http://www.mediafire.com/file/gza5360wdzqd743/threads-archive-pr-1..17.zip

Если все нормально получится импортировать, я и оставшееся скину. Учти, что там верстка со временем менялась.

Я сам собирался когда-нибудь сделать архив, но все руки не доходили.

Также, есть еще такой архив, тут все HTML-файлы вcех тредов, без картинок, в исходном необработанном виде. Часть тредов была восстановлена из гуглокеша, что-то было восстановлено из DOM в браузере уже после того как тред был удален, потому там верстка может различаться: http://www.mediafire.com/file/aahizd9bi5kvmnk/threads-1-86-html-only.zip

Ну и если есть более важные дела, не трать слишком много времени на это.

И если будешь писать парсер, я советую предусмотреть возможность заново распарсить тред, это наверно не раз пригодится, если будет найден какой-нибудь баг.

>>966596

Такое решение тоже подходит, оно чем-то даже лучше STI, так как защищает от ситуаций, когда один лайк ссылается на несколько записей.

> Если захочу запретить пользователю ставить лайк самому себе, то в MySQL нужно писать триггер, а Postgres просто использовать CHECK?
Видимо, да. Судя по http://stackoverflow.com/questions/2538786/how-to-abort-insert-operation-in-mysql-trigger/8559284#8559284 там есть костыль, чтобы выбросить ошибку в ходе операции. Но конечно CHECK лучше так как он виден в SQL коде таблицы.


>>966516

Да, на яваскрипте. Программы на яваскрипте можно встраивать в страницу. В ОП посте есть задачки на JS, но изучать придется с самого начала, а не с того как что-то сделать с формой.

>>966393

Есть мнение что главная цель ампа - лучше отслеживать действия пользователей гуглом и контролировать размещение рекламы.

Аноним 06/04/17 Чтв 07:59:13  967837
>>966275

В формах есть пара специальных типов полей:

- http://symfony.com/doc/current/reference/forms/types/collection.html
- также, можно вкладывать поля друг в друга (то есть у класса формы может быть поле-подформа) для отношений один-к-одному

Добавление/удаление вопросов и переключение их типов придется скорее всего делать яваскриптом.

Роуты может быть удобнее описывать не аннотациями, а в отдельном конфиге.

> if (!$tests) {
> throw $this->createNotFoundException(
> 'No tests'
Если тестов пока нет, то стоит выводить вместо списка сообщение об этом.

> ->createQuery("select a from AppBundle\Entity\User a
> where a.id=".$test->getAuthor()->getId())
Надо использовать плейсхолдеры, так же для поиска по простым условиям есть функции findBy(), findOneBy(), find().

Также, возможно стоит выносить функции вроде "найти тест", "найти все тесты определенного вида" в отдельный сервис. Так как позже там будут добавляться условия (вроде того, что тест должен быть не скрыт), и придется копипастить проверки этих условий.

> if (isset($_GET['status'])){
Надо использовать Request

>>966146

Ты про класс HttpKernel или компонент? Если посмотреть компонент https://github.com/symfony/http-kernel то там довольно много, так что наверно вопрос был про класс.

Вот интерфейс: https://github.com/symfony/http-kernel/blob/master/HttpKernelInterface.php
Вот реализация по умолчанию: https://github.com/symfony/http-kernel/blob/master/HttpKernel.php

Как видно, HttpKernel является чем-то вроде Front Controller, он обеспечивает нахождение и вызов контроллера, который будет обрабатывать запрос, обработку исключений и генерацию событий. События довлоьно важны, например, роутер в Симфони вызывается по одному из событий вроде REQUEST (а по событию EXCEPTION запускается обработчик, выводящий страницу ошибки). Хотя мне это не очень и нравится, так как такой неявный вызов роутера создает свои сложности.

Аноним 06/04/17 Чтв 07:59:59  967838
>>966103

Если ты используешь синтаксис вроде $this->... внутри коллбека, то надо проставить как-то тайп-хинт для this, может через @var $this, но не уверен, что это будет работать. Или откуда ты берешь объект ContainerWrapper? Надо как-то на эту переменную поставить тайп-хинт или phpDoc конструкцию.

>>966100

Ты просто неправильно поделил приложение на M/V/C.

Твое приложение можно рассматривать двояко:

- либо как единое целое, где MVC приложение находится на сервере, а JS-код - это лишь часть интерфейса, часть HTML-страницы, "тонкий клиент", который просто передает запросы на сервер и отображает ответы. То есть является дополнением к View.
- либо как 2 независимых приложения: серверное на PHP, и отдельное на JS, которое взаимодействует с сервером, но содержит свои модели, свое хранилище данных, может само их обрабатывать. Тут будет 2 набора M/V/C на клиенте и сервере.

Если JS код это отдельное приложение, то код на JS - долгоживущий, и ему нужен вариант MVC с активной моделью. Если что, вот тут на примере игры разбираются такие реализации MVC: https://github.com/codedokode/pasta/blob/master/js/minesweeper-mvc.md

Проще наверно рассматривать JS код как "тонкого" (с маленьким объемом логики) клиента, а PHP-код как отдельное MVC приложение. Тогда у нас на сервере мы используем обычные контроллеры, модели и вью. А задача JS приложения просто передать запрос пользователя на сервер и отобразить результат, оно просто служит продолжением контроллера и вью в PHP.

Делать JS как отдельное приложение - это значит, надо делать хранилище данных на клиенте, синхронизировать его с сервером, делать какие-то операции независимо от сервера. Это дает преимуещства вроде возможности работать в оффлайне или обрабатывать данные на клиенте, но тебе это вряд ли нужно. Если ты видел мобильные приложения, они примерно так же работают - они берут данные с сервера, но какие-то операции могут делать сами, и многда могут работать при пропадании интернета.

> написано, что он должен возвращать результат через return. Но пхп-скрипт, который вызывается в аяксе, оформлен у меня просто кодом на пхп, а не функцией (вернее, функции там тоже есть, но уже внутри).
Потому что ты не совсем верно разделил код. Если мы считаем что JS - это полноценное приложение, то в JS коде должна быть модель, а в ней функция, которая принимает данные, отправляет запрос, и позже возвращает результат. Если мы считаем что приложение у нас только на сервере, то эта функция должна быть только там.

> Там же написано, что пхп-скрипт не должен выдавать ничего в echo.
Выводом должен заниматься только View. Модель точно не должна, контроллер обычно тоже.

> Единственный вариант, который я вижу - json.
Это то же самое, что вывод HTML.

> Первый стул - это вернуть из пхп-скрипта (Модели) эти значения и уже в js-скрипте (Вид) подставить их в ячейки документа
> Второй стул - это оставить в index.php только место для вставки, а полностью весь код таблицы набрать в пхп-скрипте, передать его через айкс обратно js-скрипту и вставить одним действием

Это просто выбор, где делать шаблонизацию: на сервере или клиенте. Надо взвесить достоинства и недостатки и выбрать более подходящий вариант.

> Но в том же гайде пишут, что нельзя Виду работать с document.
Виду как раз можно, это модели нельзя.

Аноним 06/04/17 Чтв 08:00:26  967839
>>965989

замечу еще что mysql-функции уже давно устарели.

>>965899

>>Сделай все в одной функции, можно даже без классов.
> Я же ООП учу, зачем мне это?
Ну например чтобы проверить, насколько запутаннее получается код без применения ООП.

> public function getEmpolyees($position, $rank, $boss, $quantity)
Во-первых, название неправильное. Функции getSomething обычно что-то возвращают, но здесь это не так (здесь подойдет add). Во-вторых, ты искуственно ограничиваешь возможности по добавлению работников. Можно было бы сделать так:

addEmployee(Employee $employee)

Это позволяет добавлять любых работников с любыми свойствами. Ты же искуственно добавляешь ограничение, что можно добавлять только работников с определенным конструктором и ровно с 2 аргументами в конструкторе. Мой подход дает больше свободы и с ним департамент не обязан знать про то, как создаются объекты работников, меньше зависимостей между классами.

Также, у тебя почему-то в getEmployees не проставляется поле quantity, хотя оно исплоьзуется в других функциях ниже. Почему?

Также, у тебя в коде ошибки:

> Notice: Undefined index: allDepSalary in /in/28B8k on line 186


> public function calculateall() //Произвести все расчеты и вывести на экран.
Это не задача департамента, что-то выводить на экран, это лучше сделать снаружи класса.

В классе Employee есть поле quantity, но оно почему-то почти нигде не используется. Зачем оно тогда?

> $salary = $salary + ($salary)*25/100;
Тут проще было определять только коэффициент, вместо того чтобы копипастить эту формулу несколько раз.

> If ($this->boss) $salary=$salary
Надо ставить фигурные скобки.

Слова class, if пишутся с маленькой буквы.

Сумму можно считать либо отдельной функцией, либо сделать класс Компания, а в нем методы для подсчета суммы.

Насчет поля quantity - надо его либо использовать, либо убрать. Либо ты помещаешь его в класс ГруппаРаботников, либо еще куда-то, либо убираешь. Но не как сейчас, что поле есть, но не используется.

Если ты хочешь, ты можешь сделать класс для группы работников, а не одного работника. Только назови его правильно. Но сделай тогда и вторую часть задачи, про антикризисные меры, иначе ООП не поймешь.

> Может перенести это поле в департамент?
Количество разное для разных групп работников и свойством департамента оно быть не может.

В реализации наследования работников есть небольшая проблема. У тебя при наследовании надо обязательно прописать значения зарплаты, потребления кофе итд. Но как об этом догадаться? Как это проконтроллировать? Никак. Здесь лучше исользовать абстрактные методы вроде получитьБазовуюЗарплату(), сделать эти методы в базовом классе, чтобы нельзя было унаследоваться, не переопределив их.

Аноним 06/04/17 Чтв 08:00:45  967840
>>967820

На мой взгляд, в ларавель слишком много статических методов. После Симфони как-то не очень.

>>967822

Нужно не инклудить что попало. Либо жестко прописывать пути к файлам. Инъекция полявляется когда ты путь формируешь на основе переданных от пользователя перемеенных.

Про меры безопасности при загрузке файлов можно прочесть в комментариях к задаче про файлообменник: https://gist.github.com/codedokode/9424217

Аноним 06/04/17 Чтв 08:27:03  967844
>>966622

Возможно, что проще всего SQL-запросом. У тебя на пару (user, article) хранится только последний просмотр или все? Если второе, то конечно запрос будет посложнее.

> Как замэпить полученное число в сущность Article?
Никак. Одна сущность - одна таблица. Можно сделать только связь M-N между User, Article через промежуточную таблицу с timetamp, но я думаю, что эффективнее просто брать нужные данные SQL запросом.

> Вытаскивать QueryBuilder через leftJoin?
QueryBuilder нужен при сборке запроса в зависимости от условий

>>966678

Был какой-то php multiplexor.

>>966706

SQL запросом лучше всего, не в сущности, а в отдельном сервисе. Получив дату, мы выбираем комментарии по условию time > ?

Зачем усложнять? Сущности/DQL надо использовать когда тебе нужен маппинг, получить 1 цифру из БД проще SQL или DQL запросом (DQL запрос тоже может возвращать 1 число, а не сущность).

> Кстати, вызывают ли @ORM\OneToMany(targetEntity="AppBundle\Entity\Comment", mappedBy="application", orphanRemoval=true, cascade={"remove"}) дополнительные подзапросы при обращении к свойствам или вытягиваются при любом запросе (в т.ч. и через QB)?

OneToMany не должен вызывать запросов, при загрузке сущности там в поле просто прописывается прокси-коллекция которая лениво загрузится только при обращении к ней.

Имей в виду, что это может вызывать свои проблемы. Представь что у тебя у Поста 1000 комментариев. Если ты сделаешь count($this->comments) то даже это приведет к выборке всех 1000 комментариев в память. Добавление 1 комментария в коллекцию скорее всего тоже вызовет подгрузку существующих.

Хотя по моему, именно для COUNT там есть какая-то опция, которая позволяет сделать SELECT COUNT в такой ситуации.

А вот обратная сторона OneToOne отношения (та сторона, где нет внешнего ключа) по моему вызвает дополнительный джойн, так как без него неясно, что записать в поле - null или прокси-объект с определенным id.

Ну то есть, если представить как работает ленивая загрузка, станет понятно, когда какие запросы вызываются. по умолчанию доктрина пытается откладывать загрузку всего на потом.
Аноним 06/04/17 Чтв 08:33:57  967845
PbGRqH1oNw.jpg (92Кб, 569x535)
>>967820
кстати, раз уж ларавель-кун в треде, ответь на вопрос, будь ласка. мне надо админку делать к тестовому заданию, как правильно и грамотно? что читать, куда копать?
Аноним 06/04/17 Чтв 08:42:51  967847
>>966976

Ссылка не работает, но по моему в сообщении черным по белым написана причина. Или у тебя использован этот тег, но все равно ошибка?

При использовании форм обрати внимание, что в отдельном компоненте форм не доступны все фичи, которые есть при использовании форм в составе фреймворка симфони.

>>967018

Сравнить хеши от разных функций нельзя. Либо ты что-то делаешь неправильно.

>>967026

Напиши выражение для

"ровно 1 цифра, за ней любое число пробелов-минусов-скобок"

Затем возьми его в скобки и напиши что оно повторяется ровно 10 раз.

>>967265

Ответ чуть выше.

>>967318

Поищи статьи вроде "N лучших адаптивных сайтов". Посмотри, какие решения использованы.

Изучи:

- тег meta viewport, подробно
- CSS-медиазапросы @media (max-width) подробно
- атрибуты srcset, теги picture, source

Для начала наверно хватит.

>>967347

Если у функции аргумент, который принимает не значение, а ссылку на переменную:

function (&$x)

То в нее можно передать только переменную, элемента массива, но нельзя например передать число или строку. Потому что функции нужна ссылка, по которой что-то можно записать, а в число записать ничего неьзя, это значение, а не переменная или элемент массива.

Почитай PHP мануал про ссылки.

>>967397

Читай оповское на гитхабе.

>>967431

Через return. Открой пример кода у ОПа и посмотри как сделан PostService: https://github.com/codedokode/pasta/blob/master/arch/mvc.md

Это и есть модель.

>>967454

Зачем пишешь бред?

>>967394

По определению, публичная папка это та, из которой раздает файлы веб-сервер. Из непуьличной можно раздавать только на уровне приложения, например PHP скрипт читает этот файл и потихоньку склармливает браузеру.

>>967527

В PHP встроен мленький веб-сервер, почитай: https://github.com/codedokode/pasta/blob/master/soft/web-server.md

>>967666

foreach нужен для перебора массива, for для циклов со счетчиком или условием.

Аноним 06/04/17 Чтв 09:45:27  967865
>>967840
Это не статика наверное, а фасады. Инфа гуглится
Аноним 06/04/17 Чтв 09:47:28  967866
>>967845
Спроси у своего Мб погонщика нужна ли админка. Может тебя просто наябывают . И если нужна спроси прокатит ли вариант с совой
Аноним 06/04/17 Чтв 10:15:49  967873
>>967839
>ты можешь сделать класс для группы работников
nananananan
Мы уже проходили это. Это очень бредовая идея.

>Насчет поля quantity - надо его либо использовать, либо убрать.
Используется же для создания работников в цикле getEmpolyees.

>У тебя при наследовании надо обязательно прописать значения зарплаты, потребления кофе итд.
Это подразумевается, когда ты создаёшь новую профессию, зачем её создавать не задав ей никаких параметров?
Аноним 06/04/17 Чтв 10:15:58  967874
Выделение004.png (17Кб, 649x172)
>>967866
нужна, надо простую написать самому
Аноним 06/04/17 Чтв 10:51:07  967885
>>966608 (OP)
Уже не знаю, что делать. В общем есть два блока:

<div>Блок1</div>
<div>Блок2</div>

Хочу разместить их строго друг за другом. Для этого прописываю в css display: inline-block; - везде пишут, что блоки приобретают свойство строчных элементов и располагаются строго друг за другом, а не переносятся наследующую строку. В Html-academy написано то же самое.

В результате у меня блок2 все равно переносится на новую строку. Вообще блядь ничего не меняется. Меня это дерьмо уже заебало. Помогите пожалуйста.
Аноним 06/04/17 Чтв 11:28:15  967908
>>967885
Ебани бутстрап точнее отдельно сетку оттуда, номером с нуля делаешь то враппать 2 твоих блока надо классом с инлайном и ширину в % блока которые вложены. Считай все правильно делаешь, врапни и чилдам ширину добавь
Аноним 06/04/17 Чтв 11:38:22  967917
>>967885
Ты все курсы в htmlacademy прошел? Такое ощущение, что нет.
Аноним 06/04/17 Чтв 11:39:26  967919
>>967838
Окей. Я так понял, что мне о MVC пока думать рано и кодить, как кодится.
Аноним 06/04/17 Чтв 11:42:38  967925
>>967919
>пока думать рано
А потом тебе станет 30 лет, останешься без работы и сбомжуешься насмерть.
Аноним 06/04/17 Чтв 11:44:40  967927
Делаю сайт, закидываю изменения сразу на хостинг.
Внезапно, браузер начинает тормозить. В консоли сайта вот такая хрень. Штоэта? У меня таких js-ок вообще нет, а в шапке подгружаются только такие:

<script type="text/javascript" src="http://ajax.googleapis.com/ajax/libs/jquery/1.5.1/jquery.min.js"></script><!-- для загрузки фото -->
<script type="text/javascript" src="http://scriptjava.net/source/scriptjava/scriptjava.js"></script><!-- для загрузки фото -->
<script type="text/javascript" src="js/external/jquery/jquery.js"></script>
<script type="text/javascript" src="js/jquery-ui.js"></script>
<script type="text/javascript" src="js/jquery-ui-datepicker-ru.js"></script>
<script type="text/javascript" src="js/jquery.main.js"></script>
<script type="text/javascript" src="core.js"></script><!-- Основная логика сайта -->


И так постоянно теперь. Зашёл на страницу, нажал что-нибудь - и пошло поехало. Проверял винду на вирусы - чисто. В другом браузере то же самое.

Главное, я не понимаю, откуда вообще это подгружается? Как определить?
Аноним 06/04/17 Чтв 11:45:25  967928
>>967925
>А потом тебе станет 30 лет
Мне 32.
Аноним 06/04/17 Чтв 11:50:17  967931
>>967927
На хостинге тебя взломали. Вытаскивай оттуда код и смотри что к чему
Аноним 06/04/17 Чтв 11:55:02  967938
>>967927
На хостинг - в смысле на модную одминку для веб-сайтов уровня бложеков, в которой даже ssh нет? Или на секурный тобою лично настроенный сервер?
Аноним 06/04/17 Чтв 11:56:55  967939
>>967928
Пенсию небось получаешь? Или просто хикка?
Аноним 06/04/17 Чтв 12:15:50  967973
>>967885
float: right
Аноним 06/04/17 Чтв 12:16:54  967975
>>967973
> Аноним 06/04/17 Чтв 12:15:50  №967973
>
Руки бы тебе оторвать за флоаты
Аноним 06/04/17 Чтв 12:17:21  967977
>>967939
Получаю, мамкину. Ну и второе тоже.
Аноним 06/04/17 Чтв 12:21:38  967980
>>967975
Но ведь это решит его проблему.
Аноним 06/04/17 Чтв 12:25:10  967986
>>967938
>>967931
Сервер Ру-центра. Там у нас самописный сайт на голом html+css+js. Внутри есть каталог уже с тем проектом, который я ковыряю. Левые запросы с самого сайта вроде не идут. А с моего подкаталога - идут, но не всегда. Не могу понять, что является спусковым крючком.
Аноним 06/04/17 Чтв 12:25:52  967988
>>967939
Работаю админом-эникеем.
Аноним 06/04/17 Чтв 12:28:18  967992
>>967986
Возможно это от хостинга. 200 лет тому назад, на бесплатных хостингах былла такая фича, как инжект скриптов в сайты юзеров. Так же бывало в настройках, подключались так библиотеки разные.
Аноним 06/04/17 Чтв 12:31:29  967996
>>967986
Ну вот и говорю - если бы настроил сам все порты, базы, поставил бы всё вручную - никто бы не взломал.
Я уже, кстати, не первый раз сталкиваюсь с таким - жил был сайт на уважаемом "хостинге", а потом хуяк и взломали.
Почему-то если соблюдать элементарные правила, то всё работает как часы годами.
Аноним 06/04/17 Чтв 12:36:07  968002
>>967885
Всё работает, о чём ты?
<div style="display:inline-block">Блок1</div>
<div style="display:inline-block">Блок2</div>
Аноним 06/04/17 Чтв 12:37:25  968004
>>967992
Просканил сайт через virustotal.com и 2ip.ru/site-virus-scaner - пишут, что чисто.
Аноним 06/04/17 Чтв 12:41:18  968007
>>968004
Так вирустотал же не может залезть во всю логику и внутренний код. Какой-то ты совсем профан, не понимаешь элементарных вещей.
Лучше подучись юниксам и прочим девопсам, в жизни пригодится.
Аноним 06/04/17 Чтв 12:56:21  968017
>>968007
>Лучше подучись юниксам и прочим девопсам, в жизни пригодится.
Не пригодится. Админство умирает.
>Какой-то ты совсем профан
Раньше как-то не приходилось сталкиваться с вирусами на сайтах.

И ещё я выяснил, что запросы начинают идти сразу после процедуры логина. Странно. Там у меня цепочка login.php -> index.php -> manager.php либо guard.php. Всё по гайду из https://habrahabr.ru/post/13726/
login.php просто стучит в дыню в бд, генерирует хеш, сравнивает пароли, ставит куки и редиректит на index.php.
header("Location: index.php"); exit();
Что тут можно сломать - хрен знает. Я даже все файлы уже заменил на заведомо чистые. Не помогло.
Аноним 06/04/17 Чтв 13:14:27  968025
>>968017
>Админство умирает.
А кто будет разворачивать всё и поддерживать? Дед Мороз?
Даже чтобы paas типа амазоновских приблуд настроить нужно дофига знания.
Аноним 06/04/17 Чтв 13:15:40  968027
у кого есть ненужный макет сайта фотографа? мне в шарагу принести, да чтоб отъебались
Аноним 06/04/17 Чтв 13:22:16  968030
>>968027
Погугли и скочай
Аноним 06/04/17 Чтв 13:30:09  968033
>>968025
>А кто будет разворачивать всё и поддерживать? Дед Мороз?
Данный срач не для этого треда. Но всё же не удержусь: количество админских вакансий с каждым годлом уменьшается, зарплата либо стоит на месте, либо уменьшается при очередном кризисе, а требования растут. Это статистика. Что там внутри происходит, какие процессы влияют - разговор отдельный. Но по итогу админство умирает. Я сейчас получаю килобакс за эникейство. Это с MCSA и CCNA. Могу я получать больше? Ну, положим, могу. 80к за въёбывание от рассвета до заката и ещё чуть-чут. Но это уже уровень руления цисковскими шеститонниками. Нужен мне такой геморрой за лишние 20к? Нет.
Аноним 06/04/17 Чтв 13:34:10  968034
>>968030
хотел добротный от анона
Аноним 06/04/17 Чтв 13:48:23  968038
>>968034
>>968030
а не от левого дяди
Аноним 06/04/17 Чтв 14:47:24  968081
Мучу файлообменник по задачке из ОП-поста.
И как бороться с повторным отправлением файла при возврате или обновлении странички? Вот это вот "повторная отправка формы".
Я сделал какую-то хуйню, чтоб избежать этого: заставил вылезать поп-ап после отправки файла - "файл отправлен" и когда тыкаешь на закрытие поп-апа, то идёт переход на страницу загрузки - ну слишком убогий костыль. Анон, что делать, а?
Аноним 06/04/17 Чтв 14:56:34  968089
>>967874
>
ну ебашь с нуля тогда
Аноним 06/04/17 Чтв 14:58:12  968090
>>968081
После загрузки файла редиректь на его страницу сразу . Поидее сработает
Аноним 06/04/17 Чтв 15:03:18  968095
>>968090
Не, если так, то при обновлении страницы или переходе назад - повторная отправка. Ну я и сделал по сути редирект, только через джаваскрипт и диалог, но это не очень.
Аноним 06/04/17 Чтв 15:08:38  968103
И кстати почему-то не могу открыть файлы, отправленные и принятые на моём WAMP'е. Отказано в доступе, картинки не отображаются, архивы не открываются в общем. В свойствах нет владельца вообще. Чтоб открыть такой файл, приходится в свойствах назначать себя его владельцем или переместить на не системный диск. Но это походу дело в винде.
Аноним 06/04/17 Чтв 15:30:23  968117
>>968103
Купи уже нормальную линукс-впску и установи lamp.
Ты не будешь разворачивать на винде заказчику.
Аноним 06/04/17 Чтв 17:03:36  968202
http://archive-ipq-co.narod.ru/l1/arrays.html делаю последнюю задачу в этом гайде, анончик подскажи пожалуйста как мне при выборе числа брать слог, который стоит в массиве на месте выбранного числа?
Аноним 06/04/17 Чтв 17:05:46  968205
>>968089
я и спрашиваю как с нуля ебашить. добавить страницу, роут и разрешить только юзеру админ туда войти? надо мидлваре контроллер написать?
Аноним 06/04/17 Чтв 17:21:13  968215
>>968202
бамп
Аноним 06/04/17 Чтв 17:24:50  968219
>>968215
Ты нахуй бампаешь свой пост, который оставил меньше 20 минут назад, и который даже уплыть не успел сука? Вот нихуя тебе не буду подсказывать. Сиди думай сам, а не бампай, как ебанутый.
Аноним 06/04/17 Чтв 17:27:12  968222
>>968219
ну лол, я сижу думаю уже часа 4 наверное. заебался если честно , уже хочется дальше пойти.
Аноним 06/04/17 Чтв 18:02:08  968236
>>966744
Э бля, там че та качать надо, иди ка ты нахуй со своими зондами, репорт.
Аноним 06/04/17 Чтв 18:13:19  968244
>>968236
Это телега же, лол
Аноним 06/04/17 Чтв 18:16:27  968249
>>968236
web.telegram.org
Аноним 06/04/17 Чтв 18:18:51  968253
>>968222
Диды без двачей писали духам завещали дидов не увожаешь
Даже стэковерфлоу не было
Дана тебе документация, есть куча книг - жуй не хочу, нет, буду ждать идиотских ответов на дваче.
На дваче только поверхностное могут сказать, беги отсюда, пока молодой мелокбуквенный, оно тебя сожрет.
Аноним 06/04/17 Чтв 18:22:25  968254
>>968253
в гугеле не знаю как сформулировать вопрос. блядь, разве так сложно мне ответить? Просто я не знаю как указать в массиве то что записано в $random
Аноним 06/04/17 Чтв 18:35:09  968256
Как мне записать то что лежит в массиве $letters под номером $random????
Аноним 06/04/17 Чтв 19:33:44  968288
Решаю задачу из урока "Повторим?", где надо изобразить калькулятор. Такой вопрос - мы храним в переменной "$op" одно из трех значений: "+", "-", "*". Такой вопрос - как из этой переменной вытащить строку и использовать как оператор? Такое вообще возможно? Ну типа:
$op = '+';
$result = 5 $op 4;

и результат будет 9.
Аноним 06/04/17 Чтв 19:47:03  968294
>>968288
Хотя уже не надо, я задачу с калькулятором так решил. Проверьте/посоветуйте, пожалуйста.

https://3v4l.org/1ensi
Аноним 06/04/17 Чтв 21:59:58  968442
>>968202
Код свой покажи, а то непонятно, подойдёт ли тебе array_rand() или нет.
Аноним 06/04/17 Чтв 22:02:12  968450
>>968442
там же есть код, самое последне задание
Аноним 06/04/17 Чтв 22:14:05  968465
>>968450
Ты свой покажи.
Короче, тебе нужна функция mt_rand() - выберешь число из количества элементов в массиве - посчитай это количество, а ещё нужно вспомнить, как доставать значение из простого массива, имея ключ - число.
Аноним 07/04/17 Птн 02:43:24  968585
>>968249
Это как-то сложно, там сайт с сотовым.
Аноним 07/04/17 Птн 02:52:23  968588
>>965842
>Электросеть
https://jsfiddle.net/6591a2sL/7/

>> Если что, я не согласен с вашим замечанием - потому что мне не нужно было давать ссылку на Сеть, только передать уже посчитанный баланс. Можно было считать Сеть за контроллер, а ЛЭП за модель которая содержит вспомогательные методы расчета. А теперь в эти методы нужно передавать ссылку на ЛЭП. Мне из-за этого кажется что я понял и сделал всё ещё более худшим образом.
>Я уже не очень понимаю, о чем речь. Тот метод, что используется сейчас, вполне подходит. Если хочется обсудить другой вариант, то хорошо бы увидеть хотя бы кусочек кода.

Было:
function ElectricalNetwork(...) {
...
}

function PowerLine(..., price) {
...
this.price = price;
}

PowerLine.prototype.countPowerAfterPass= function(power) {
...
}

PowerLine.prototype.countPrice = function(power) {
...
}

Потом стало:
function ElectricalNetwork(...) {
...
}

ElectricalNetwork.prototype.countPowerAfterPass= function(linkToPowerLine, power) {
...
}

ElectricalNetwork.prototype.countPrice = function(linkToPowerLine, power) {
...
}

function PowerLine(..., price) {
...
this.price = price;
}

>> Можно я не буду эту менять? Придётся заново переосмысливать мою программу.
>По идее, ты должен стараться писать программу так, что в ней было легко разобраться. И соответственно небольшие изменения не должны вызывать сложностей и ради них не требуется всю программу переделывать, а только небольшую часть. А если все оставить как есть - может ты и дальше будешь писать цикл там, где он не требуется.
На самом деле, сейчас я припоминаю, что сначала я пытался решить через min()\max(), но у меня что-то не получалось и я решил написать счетчик через цикл, как и должно быть работают счетчики в реальном мире.


>Определение типа переменной
https://jsfiddle.net/5q3r473h/5/


>Напиши функцию неглубокого копирования объектов и массивов
https://jsfiddle.net/uyey3at1/6/

>> var clone = object.bind(clone);
>Тут ошибка. Во-первых, клонировать функции не требуется (да и это невозможно наверно), во-вторых, если уж клонировать, то непонятно почему надо привязывать this к пустому на тот момент значению clone. Что делает этот bind?
Я пытался получить новую функцию из старой.

>> Включая примитивные значения, которые на самом деле тоже объекты?
>Это не так. Почитай пожалуйста еще раз про боксинг, например в моих заданиях по JS ( https://gist.github.com/codedokode/ce30e7a036f18f416ae0#Боксинг ) или в других статьях: http://www.jisaacks.com/javascript-boxing/
>
>Примитивные значения - это не объекты, но если пытаться их использовать как объекты (обращаться к свойствам или методам), для них автоматически создается временный объект, в который заворачивается примитивное значение.
Ах да точно, я забыл про это.


>Глубокое копирование
>Тут те же замечания, что и к предыдущей задаче.
https://jsfiddle.net/b0a7tk75/3/
Аноним 07/04/17 Птн 10:34:43  968654
1325368323001.jpg (17Кб, 414x600)
>>968236
Не только этот долбоёб, но ещё и модератор долбоёб.....
Сажи вам с картинкой и бугуртом...
Аноним 07/04/17 Птн 10:35:23  968655
>>968654
Sage отвалилась
Аноним 07/04/17 Птн 11:46:06  968667
В общем есть некая страница профиля студента. Нужно реализовать функционал, благодаря которому студент сможет загрузить свое фото и оно будет отображаться в его профиле. Как это проще всего реализовать? Буду благодарен за любые советы.
Аноним 07/04/17 Птн 11:52:09  968669
Есть конструкция вида (function($){…})(jQuery). Когда она находится внутри <script></script> прямо под тем html-кодом, который её вызывает, всё ок. Но стоит перенести функцию в отдельный js-файл, как работать перестаёт, просто не реагирует на нажатие кнопок, как будто его и нет.
Весь код: https://jsfiddle.net/ex16zd7h/

>«(function($){…})(jQuery) » создает анонимную функцию, и тут же
>вызывает ее, передавая в качестве параметра объект jQuery, таким
>образом внутри анонимной функции мы можем использовать алиас $
>не боясь за конфликты с другими библиотеками — так как теперь $
>находится лишь в области видимости нашей функции, и мы имеем
>полный контроль над ней.

Как её правильно вынести в отдельный файл?
Аноним 07/04/17 Птн 11:58:36  968671
>>968669
Ты же в начале своё барахло запускать пытаешься?
https://learn.jquery.com/using-jquery-core/document-ready/
Или запускай его в конце, когда весь документ уже создан.

Аноним 07/04/17 Птн 12:03:02  968672
>>968671
Не понял тебя. По ссылке правила для $( document ).ready(), но у меня скрипт должен срабатывать не после загрузки страницы, а при клике по ссылке. JS-файл загружается в head самым последним. А код, который хочу перенести из html-файла, в js-файле вставляю тоже в самый низ.
Аноним 07/04/17 Птн 12:11:24  968673
>>968672
Браузер выполняет твой html (именно html, не js) сверху вниз. Когда ты подрубаешь свой js файл через тег <script> у тебя в доме находятся только те теги которые выше него в html файле. В твоём коде за авторством Тимура эвент вешается на ссылку через $('.submit.button').click но видимо в этот момент дом пуст, там еще нет этой ссылки. Твои варианты:
1) дерунть код в конце html файла руками через чет типа <script>init();</script>
2) дернуть код через $(document).ready
3) использовать $().live или $().on или как он блять сейчас в этой параше называется чтобы вешать эвенты на те элементы, которых еще нет в доме.
Аноним 07/04/17 Птн 12:14:33  968674
>>968673
>2) дернуть код через $(document).ready
То есть, обернуть js-код в эту обёртку? Спасибо, попробую.
Аноним 07/04/17 Птн 12:22:33  968680
>>968673
Спасибо, получилось.
Аноним 07/04/17 Птн 13:41:47  968720
>>968667
Бамп
Аноним 07/04/17 Птн 13:48:56  968723
>>968667
Ячейку в таблице student назвать avatar.
Затем функционал по загрузке изображения - экшн actionSetAvatar(), там метод по загрузке изображения, проверке-валидации, затем убрать прежний аватар, если есть, а потом сохранить и отрендерить.
Что-то такое, наверное.
Аноним 07/04/17 Птн 13:51:21  968724
>>968723
А, да: при этом в ячейке avatar сохранять кодированное имя изображения - md5 какой-нибудь. Чтобы у всех студентовских аватарок было своё имя, уникальное.
Довольно много всего нужно, конечно.
Аноним 07/04/17 Птн 13:58:43  968735
Ещё вопрос к >>968669. Пытаюсь передать через аякс не одну переменную, а несколько. Но конструкция, типа
data:
{
data1:data1,
data2:data2,
},

Не работает. Хотя в другом скрипте работает.
Аноним 07/04/17 Птн 15:42:09  968779
Черт побери. Как? Как я должен выбрать именно то число которое выпало?? Я не понимаю. Уже второй день ебус с этим, хелп
Аноним 07/04/17 Птн 15:53:00  968781
>>968779
>Как я должен выбрать именно то число которое выпало?
Наверное, ты имеешь в виду:
$selector = mt_rand (0, 999) % 1000;
Где тысяча — количество элементов в массиве (это я так написал, чтобы понятно было).
Аноним 07/04/17 Птн 15:57:29  968782
>>968781
не, смотри. я думал об этом. У меня условие, что надо выбрать 4 элемента из массива, и указать их индекс. Как узнать индексы рандомом я понимаю, как вывести тоже, но как узнать то, что записано в эти индексы и вывести это? Вот это я не понимаю.
Аноним 07/04/17 Птн 16:06:13  968784
>>968779

Ты читал мануал? array_rand выбирает случайный элемент из массива и возвращает его ключ (в твоем случае это скорее всего будет число, так как ты при создании массива не указал ключи явно, то PHP просто присвоил им порядковые номера). Проверить что именно в переменной, можно командой echo или var_dump.

А теперь открой урок про массивы и поищи, как имея массив и ключ элемента, получить значение этого элемента? Это и будет искомый слог. Если не найдешь, напиши, я или кто-то еще напишет правильный ответ.
Аноним 07/04/17 Птн 16:28:33  968795
>>968779
Ты смотри, мелкобуквенный пидр заговорил по-человечески!
Выше тебе уже объяснили всё и не раз.
>>968782
За 4 дня ты не смог дойти до того, что $randomText = $letter[$random]?
Объяснил же тебе выше всё.
Делай вар_дампы постоянно, пробуй, тыкайся, уточняй ответы анонов, хули ж ты.
Аноним 07/04/17 Птн 16:35:06  968798
images (1).jpg (2Кб, 256x192)
>>968795
чувствую себя парнем с пика. Пиздец просто. Я тупо ставил () перед [] и из за этого нихуя не работало, ибо в гайде такой синтаксис был. Вот я еблан
Аноним 07/04/17 Птн 17:07:09  968809
>>967874
Интересно, выложи полное ТЗ.
Аноним 07/04/17 Птн 18:18:18  968846
>>968798

Напиши, где именно был неправильный пример - я проверю и исправлю, если что-то не так.
Аноним 07/04/17 Птн 18:25:55  968851
>>968846
Я посмотрел, на самом деле я еблан, но все же в разделе Повторим в массивах, нету Того как выводить определенный элемент зная его индекс. и там только в обучалке написан пример с echo 'Lalala {$weather[0]} , а я не заметил что там { и ставил(.
Аноним 07/04/17 Птн 18:31:49  968863
>>968851

А, фигурные скобки они нужны для отделения выражения от остальной части строки (которая просто выводится как есть). Подробнее в мануале: http://php.net/manual/ru/language.types.string.php#language.types.string.parsing


Аноним 07/04/17 Птн 20:34:36  968948
Объясните зеленому, как оформить такую плавную смену фона, как на этом сайте?
https://kettlenyc.com/
Аноним 07/04/17 Птн 21:32:15  968966
Решил первую задачу из урока "Еще немного задачек", про банкомат, на основе жадного алгоритма.

https://3v4l.org/WDC9X
Аноним 07/04/17 Птн 22:45:39  968998
>>965621
Не получается обернуть https://jsfiddle.net/b073aLb5/2/ Можно ещё подсказку?
Аноним 08/04/17 Суб 01:51:18  969046
>>968948
Код страницы открывать не пробовал? Стоит сразу в body:
-webkit-transition: background-color 0.5s ease-out,fill 0.5s ease-out;
transition: background-color 0.5s ease-out,fill 0.5s ease-out;
background-color: #fffb93;

background-color меняется через добавление класса с другим background-color через js, плавность через transition
Аноним 08/04/17 Суб 11:57:45  969195
>>969046
Спасибо, добрый анон, я попробую.
Аноним 08/04/17 Суб 15:29:45  969308
Аноны. Еще вопрос по симфони:
Что такое бандл? Это просто часть проекта? Нормально ли создавать бандл без контроллеров и вьюшек чисто для хранения бизнес-логики, чтобы использовать этот бандл в других бандлах? Можно ли настроить роутинг так, чтобы в зависимости от субдомена использовался определенный бандл? Можно ли создать бандл чисто для отдельной части сайта? Например для всех путей сайта бандл MainBundle, а для http://site.com/api использовался ApiBundle?
Аноним 08/04/17 Суб 18:35:39  969407
Выделение005.png (54Кб, 672x440)
>>968809
полное
Аноним 08/04/17 Суб 19:01:22  969420
>>969308

Бандл это компонент сайта, который можно повторно использовать. Что-то вроде сторонних библиотек, которые подключаются через композер, с той разницей, что бандл может вносить изменения в конфиги проекта, например, добавлять свои роуты или ключи конфига.

http://symfony.com/doc/current/bundles.html

Бандл может предоставлять:

- сервисы в контейнере
- роуты
- контроллеры
- сущности доктрины
- шаблоны
- статические файлы

Бандл это что-то повторно используемое. Ну например, бандл форума, который можно прикрутить к любому проекту. Или бандл, добавляющий новые виджеты для форм.

В качестве примеров можно посмотреть бандлы в составе симфони или сторонние бандлы.

Бандлы в большинстве случаев самодостаточны и друг от друга не зависят.

Если ты делаешь приложение то скорее всего тебе бандлы не нужны и хватит одного AppBundle. Некоторые пишут, что можно даже без него, не оформлять код сайта в виде бандла.

> Можно ли создать бандл чисто для отдельной части сайта? Например для всех путей сайта бандл MainBundle, а для http://site.com/api использовался ApiBundle?
Можно

> Можно ли настроить роутинг так, чтобы в зависимости от субдомена использовался определенный бандл?
Можно, но не автоматически, а вручную прописать в разные роуты контроллеры из разных бандлов
Аноним 08/04/17 Суб 19:34:16  969436
>>969407
Любой фреймворк бери, разбирайся и делай на нём, это же не возбраняется.
Или ты прямо с нуля хочешь?
Аноним 08/04/17 Суб 19:58:19  969450
Всем привет. В общем есть обычная формочка для загрузки изображения:

<form enctype="multipart/form-data" action="photos.php" method="POST">
<input name="filename" type="file" />
<input type="submit" value="Send File" />
</form>

Хочется передать переменную $get вместе с изображением, но я не знаю как. Как сделать так, чтобы эта форма передавала и переменную и изображение?
Аноним 08/04/17 Суб 20:04:47  969451
приветик, а где можно найти себе вторую половинку в виде фронтенда? а то я ток в бекенд умею но это не точно, а хочется что-то красивое пилить
Аноним 08/04/17 Суб 20:46:09  969462
В вопросотреде не отвечают ((

Поясните за source страницы. Почему в инспекторе я вижу нормальные значения, а когда захожу в код или беру dom скриптом, то некоторых значений нет? Как мне распарсить dom уже с финальными значениями из инспектора?
Аноним 08/04/17 Суб 20:49:13  969466
>>968655
>>968654
Найс пиарщик порвался. А модератор хороший человек, не проигнорировал. Спасибо Абу.
Аноним 08/04/17 Суб 21:10:20  969476
Все тот же тупой анон на связи. Решаю задачу на строки, и тут возник вопрос, а что переделать в программе? Я нипанимаю( вот текст задачи.
l33tspeak
Если ты когда-нибудь видел хакеров (например, в кино или в интернете), то знаешь, что у них есть свой, особый язык, который позволяет отличать своих от чужих. Он называется l33tspeak, и объяснен в википедии: http://en.wikipedia.org/wiki/Leet (как видишь, википедисты обладают элитными знаниями).

Чем мы хуже хакеров? Давай напишем программу для перевода текста на элитный язык. Опять же, воспользуемся массивом перекодировки и функцией strtr

Задачка: доделай программу по ссылке http://codepad.org/5XEkSKHI для перевода текста на l33tspeak. Покажи нам результат.
Аноним 08/04/17 Суб 21:52:14  969496
>>969476
Это какая-то неприлично простая задача. У меня испанский стыд.
Аноним 08/04/17 Суб 21:52:56  969497
>>969496
я только начинаю изучать и это задача на самом начале изучения строк. я просто не понимаю в чем заключается вопрос, что не так работает в программе?
Аноним 08/04/17 Суб 21:53:46  969499
Сделал программу про стихи(http://archive-ipq-co.narod.ru/l1/strings.html) , чувствую себя макакой. Как можно было дописать ее правильнее? Или только так? http://ideone.com/Ek5k1S
Аноним 08/04/17 Суб 21:55:59  969501
>>969497
Нужно все буквы в строке поменять на хуйню типа: "/7rN|3€t".
Аноним 08/04/17 Суб 21:56:42  969502
>>969501
а, так это правда очень легко.
Аноним 08/04/17 Суб 22:22:20  969514
>>969499
Если что-то повторяется, то надо использовать цикл.
А это решение никуда не годится.
Аноним 08/04/17 Суб 22:32:09  969518
Делается словарь с алфавитом. Циклом проходится каждый символ в строке, вложенным циклом проходит по словарю, заменяет тукущий символ или просто пишет новую строку.
Аноним 09/04/17 Вск 01:07:11  969566
А когда можно будет сказать, что я владею php достаточно неплохо? Что надо уметь?
Аноним 09/04/17 Вск 02:17:48  969589
2016-07-19141324.jpg (79Кб, 368x497)
>>969566
Достаточно не плохо для чего?
Аноним 09/04/17 Вск 08:17:50  969654
14334131747083.jpg (49Кб, 604x455)
>>969589
Чтоб бабаос с лохов рубить, очевидно же.
Аноним 09/04/17 Вск 08:18:30  969655
>>969589
Ну он не понимает просто пока ничего, и ему отправная точка нужна и цель какая-то. В общем что бы можно было проложить маршрут. Причем не длинный, а по конкретным ключевым точкам. Ну тип вроде как пришел на собеседование. А там:какие паттерны вы знаете? И ты такой ну вот эти 10 знаю, и рассказал. И на тебя уже как на человека смотрят. Хочется вот так же что бы было. А то, что на самом деле это за вечер "учится", никого не ебет.
А не дрочишься 2 года такой в разработке. А у тебя всё на сингтоне и норм. А на собеседовании ты такой, ну тип синглтон. А они такие: и что, всё? Ну вы знаете, мы вам перезвоним.
Аноним 09/04/17 Вск 11:45:10  969724
Как вот такая штука реализовывается: https://briefly.ru/aristofan/ljagushki/
При просмотре страницы прогресс-бар внизу увеличивается, показывая, сколько ещё осталось.
Тут анон ещё спрашивал об эизменениия фона при скролле 0 в ту же тему.
Аноним 09/04/17 Вск 13:41:01  969871
>>969589
Чтобы я мог уже не столько учить новые команды, сколько мог уже улучшать понимание изученных. То есть тот уровень, при котором основами я владею хорошо.
Аноним 09/04/17 Вск 13:47:16  969877
Сап, чет не могу сообразить. Вывел изображение из папки кодом:

echo "<img src='/images/".$id.".jpg'>"

Все нормально вывелось. Затем, если заменить это изображение в папке другим изображением с таким же именем, то на сайте все равно остается старое изображение, хотя в папке новое.
Аноним 09/04/17 Вск 13:48:52  969880
>>969877
А, все, чет не работало не работало, а потом стало работать. Странно.
WordPress Аноним 09/04/17 Вск 14:41:16  969982
Кто-то с чего начинал изучать WordPress?
Какие книжечки читали по нем ?или прочее
Аноним 09/04/17 Вск 16:27:34  970087
Снимок.PNG (43Кб, 755x628)
>Notice: Trying to get property of non-object on line 16
Помоги анон.
Foreach + массив - могу
Foreach + объекты - могу
For + массив - могу
For + объекты - не врубаюсь как
Аноним 09/04/17 Вск 16:30:16  970090
>>970087
<?= $obj->info->people[$i]->name ?>
Аноним 09/04/17 Вск 16:33:54  970101
>>970090
Ох, спасибо. Я уже и методом тыка кучу вариантов перебрал... Не нашёл примеров.
Аноним 09/04/17 Вск 16:42:22  970121
>>970090
Кстати, пишут, что ++$i вроде как работает быстрей.
Я попробовал, визуально ничего не изменилось.
Стоит ли использовать в этом примере?

меньше недели учу php, извиняйте
Аноним 09/04/17 Вск 16:53:31  970143
>>970121
Не быстрее, это просто пре-инкремент, когда сначала происходит математическая операция, а затем возвращается значение. А $i++ это пост-инкремент, сначала возвращается значение, а потом происходит математическая операция. В этом >>970087 коде если поменять $i++ на ++$i, то код выведет все элементы массива начиная с индекса 1, пропустив при этом 0 (самый первый).

Это так же имеет отношение к декременту, подробнее можешь тут почитать
http://php.net/manual/en/language.operators.increment.php
Аноним 09/04/17 Вск 17:24:41  970196
>>970143
Спасибо, уже читаю.
Аноним 09/04/17 Вск 18:14:56  970242
В общем есть изображение, которое пользователь загружает в форму. Нужно, чтобы его размер пропорционально изменялся
Аноним 09/04/17 Вск 18:15:19  970243
>>970242
Как сделать?
Аноним 09/04/17 Вск 19:18:51  970366
>>970242
Пропорционально чему?
В процентах от экрана можно.
$width = 200;
$height = 'auto';
Аноним 09/04/17 Вск 20:36:33  970462
>>970196
Прочитал, ++$i сомнительная штука.
>>970242
В html и css например, прокатывало пописывание только одного значения высоты или ширины.
Аноним 09/04/17 Вск 22:11:55  970540
>>969462
Бамп
Аноним 09/04/17 Вск 22:26:31  970551
Ребят, я тут сейчас перекатываюсь в веб разработку, смотрю различные фриланс сайты и т.д. Вы мне можете объяснить, кто такой senior wordpress developer. То есть аноны там что-то зарабатывают работая с html&css и wp. Я с wp не работал, но за что там могут платить? То что я слышал, то там и программирования как такого нету. Ты вот тянешь для разработки html/css/js + php, паттерны там какие то учишь, фремворки, а можно просто делать сайтики на wp? поясните, плиз.
Аноним 10/04/17 Пнд 02:44:44  970693
>>970551
>за что там могут платить
За вредность.

>Ты вот тянешь для разработки html/css/js + php, паттерны там какие то учишь, фремворки, а можно просто делать сайтики на wp?
Ты всю свою карьеру собираешься сайтики на WP разрабатывать?
Советую не тратить на это время, потому что заебет не через год, так через три.
А приобретенные за это время навыки и практики тебе нигде не пригодятся, или чего хуже только навредят.
WP, друпал и прочие джумлы это тупик. Если хочешь быть разработчиком, а не администратором CMS, начни с изучения основ.
Аноним 10/04/17 Пнд 02:47:43  970694
>>970551
>senior wordpress developer
Петушара это зашкваренный. Ни один серьезный разраб не будет такое про себя писать.

>Я с wp не работал, но за что там могут платить?
Говносайтов туча на нем, заказчиков много. Паттерны-фреймворки там не нужны, достаточно доку по вордпрессу прочитать. Платят мало, ибо скиллов особо не требуется.
Аноним 10/04/17 Пнд 11:43:34  970784
Взялся за задачу про студентов, несколько дней разбирался в css, запилил формы реги/логина, почитал про БД, поковырялся в mysql admin'е, опять же почитал про MVC и остальные вводные со страницы задачи, а дальше то чтооооо? Оп предлагает пердолится с ходу с фреймворками типо автоподключения классов и PDO, меня это в первую очередь пугает.

Может кто скинет годный урок по созданию первого простенького сайта с подключением БД и какими-то действами в php? Хотя бы понять алгоритм действий и увидеть как это всё строится.
Аноним 10/04/17 Пнд 11:47:13  970787
>>970784
>Может кто скинет годный урок
Я сам недавно вникал в это, нашёл такое:
http://phpfaq.ru/pdo
Попробуй, мне нормально зашло.
Аноним 10/04/17 Пнд 15:13:27  970899
>>970784

Вообще, если ты читал урок по MVC, то можно для начала сделать как там, без базы данных, просто пример данных в коде записать. А позже добавить работу с базой данных.
Аноним 10/04/17 Пнд 19:49:33  971106
>>970551
Senior Microsoft Word VBA Scripts Developer
Аноним 10/04/17 Пнд 20:52:54  971171
ОП, я ниасилил задачку из твоего учебника на навигацию по Питеру. Я совсем безнадёжен, да?

зы. Задачку на рекурсию про "Ханойскую башню" с другого сайта решил за пару минут, не заглядывая в подсказки (просто почитав, что такое вообще рекурсия), а эта что-то вообще не зашла, но очень заинтересовала. Что курить для её решения своими силами?
Аноним 10/04/17 Пнд 22:00:10  971231
>>970784
Автоподключение и pdo это не фреймворки, а часть php же, в любой книге по php их проходят. Фреймворки это когда symphony какой-нибудь натягиваешь.
Аноним 10/04/17 Пнд 22:03:07  971237
>>971171
Норм все. Я когда начинал, тоже всякое такое примитивное осилить не мог неделями, щас достаточно сложные проекты пилю. Мозги раскачиваются постепенно на программирование, сразу ничего не бывает. Просто пробуй дальше каждый день, потом легче пойдет.
Аноним 10/04/17 Пнд 22:53:28  971307
14628031818000.jpg (68Кб, 609x609)
>>971237
>тоже всякое такое примитивное
>всякое такое примитивное
>такое примитивное
>примитивное
>алгоритм дейкстры-хуекстры
Аноним 10/04/17 Пнд 23:11:36  971336
>>971171
я не могу банкомат решить на не жадном алгоритме, так что ты даже умнее меня
Аноним 11/04/17 Втр 00:18:07  971416
>>971336
Никто не может вообще-то из присутсвующих.
жопа 11/04/17 Втр 02:23:38  971490
examcode.png (39Кб, 2088x738)
exam.png (27Кб, 1338x768)
Сап пыхыпач, решил вкатиться в веб, для себя начал вкатывание с пыхи, имею базовые знания в программирования, в качестве первой задачи выбрал написание двачаанонимного форума
За 2 дня была успешно сделана система постинга без картинок картинки постараюсь сделать завтра если не буду сильно занят
Мне интересно, насколько некорректно делать отображение как я сделал ? с использованием хтмл тегов в пхп коде
Посоветуйте как прикрутить по больше функционала и сделать код красивее.
Аноним 11/04/17 Втр 02:30:45  971496
>>971490
нихуя, тут были гайды всё это время, пошёл читать, раз такое дело просто оцените моё говно по степени говняности \10, где-то 7-10 часов моего старания вот.
Аноним 11/04/17 Втр 02:50:27  971508
>>971496
Убер говно конечно-же.
Главное то, что вообще сделал.
Аноним 11/04/17 Втр 02:53:46  971510
>>971508
Мне нужны объяснения
хотя читая гайды я кое что понимаю
пожалуйста
Аноним 11/04/17 Втр 03:09:13  971513
>>971490
а ещё вопрос, насколько предпочтительнее ООП нежели его не использование.
Аноним 11/04/17 Втр 03:19:18  971517
>>971510
>>971513
Сначала гуиды, а потом вопросы. Там все расписанно.
Аноним 11/04/17 Втр 03:20:18  971518
>>971517
но, но ...
мне интересно сейчас
Аноним 11/04/17 Втр 03:44:00  971520
>>971518
Слишком много писать, по этому все собрали воедину и запилили одним гуидом.
Аноним 11/04/17 Втр 07:36:59  971546
>>970787
Сразу же в начале куча не известных мне слов. Объяснятся очевидно для тех кто уже работать с БД.

>- во-вторых, потому что исключение всегда содержит в себе незаменимый stack trace,
- в-третьих - исключения чрезвычайно удобно обрабатывать.

Плюс очень удобно задать FETCH_MODE по умолчанию, чтобы не писать его в КАЖДОМ запросе, как это очень любят делать прилежные хомячки.
Также здесь можно задавать режим pconnect-а, эмуляции подготовленных выражений и много других страшных слов.

Вот абсолютно ничего не понятно.
Аноним 11/04/17 Втр 10:14:48  971579
>>971513
С объектами проще управляться в крупном проекте.
Представь, у тебя куча функций простых, которые явно никак друг с другом не связаны, замучаешься разбираться.
А представь класс, у которого всё определено в свойствах и методах, методы (те же функции) относятся только к этому классу (не обязательно, можно статические методы, которые будут работать независимо), всё удобно и понятно.
Аноним 11/04/17 Втр 12:09:12  971643
Ребята. Только вкатился в php и программирование, в общем. Есть проблема с заданием из учебника опа. Подскажите, что не так с моим кодом http://ideone.com/hQmnsB ?
Аноним 11/04/17 Втр 12:46:34  971671
>>971643
Попробуй поставить if с (creditBalance < 0) в начале цикла, а не конце.
Аноним 11/04/17 Втр 13:03:15  971682
Аноны, поясните, что за путаница. Почему во всех русскоязычных мануалах padding - поля, а margin - отступы? Ведь правильный перевод с англ. padding - отступы, а margin - поля. Я учил по второму варианту, и теперь читаю русскоязычные уроки, задания и мозг плавит. Как канонично?
Аноним 11/04/17 Втр 13:08:47  971690
>>971682
> правильный перевод с англ
> padding
падинг
> margin
маржин

Так проще.
Аноним 11/04/17 Втр 13:17:01  971699
>>971643
Сначала проверяй на отрицательность, потом только сохраняй значение.
Аноним 11/04/17 Втр 13:40:17  971712
Такой вопрос, а в CSS всегда в body указывается margin: 0px;? Если не написать этот кусок кода, то съезжает разметка слегка в сторону. WTF?
Аноним 11/04/17 Втр 14:05:16  971742
>>971546
Понял, что без PDO уже нихуя не сделать, дайте по нему нормальный гайд для даунов, потому что въехать в prepare и execute просто не возможно, на php-manuale как обычно хуйня написана без пояснений.
Аноним 11/04/17 Втр 16:09:40  971843
>>971690
>падик
>маржа
Аноним 11/04/17 Втр 16:15:23  971850
ОП, можно я использую структуру твоего учебника для создания курса по РНР?
Хочу сделать блог с курсами: от сих до сих изучаешь что-нибудь, после чего можешь считать себя тем-то тем-то.
Курсы там будут по Photoshop, PHPStorm, по Yii2 уже почти готов, ну вот и хочу сделать по РНР, скопировав структуру, задачи (но там и куча других будет разных, а многих сложных и не будет - из "дополнительных").
Сам текст я переработаю, но вот структуру могу ли взять, формулировку задач?
Часть у тебя, часть у Робина Никсона, там тоже толково многое и понятно.
Аноны так-то возмутятся всё равно, наверное.
Аноним 11/04/17 Втр 17:50:07  971922
>>971850

А что ты подразумеваешь под структурой? Если ты используешь такие же темы уроков, но тексты сам напишешь, то это будет твое самостоятельное произведение, и тут даже мое разрешение не нужно.

Насчет задач - я в общем-то не против, бери безо всякий условий (я их все равно хочу переделать как-нибудь), но советую добавить пояснения, как их решать, примеры правильных ответов, а то как видно из треда, люди часто путаются.

Тексты/картинки копировать целиком не советую, хотя бы потому что поисковые системы увидят плагиат и пессимизируют твой сайт и его никто не найдет.

Вообще, я давно собирался опубликовать тексты и картинки под какой-нибудь открытой лицензией creative commons (но пока не выбрал, под какой), но в этом случае там свои подвохи - надо указывать автора и надо публиковать свой текст под аналогичной лицензией. Если ты на такое готов в будущем (когда я выберу лицензию), то можешь копировать. Нет - пиши сам.
Напомню Аноним 11/04/17 Втр 17:50:40  971924
Напомню себе, что уже пару тредов лежат непроверенными:

https://github.com/someApprentice/maintaskforlayout/ >>919288
https://github.com/someApprentice/Students/ >>926219
https://github.com/sylenien/php-news-site/ >>927983
https://github.com/grigoryMovchan/zuihitsu >>932473
https://github.com/anotherCodeMunkey/fileshare >>938323

Если я еще кого-то пропустил, напомните о себе.
Аноним 11/04/17 Втр 17:51:25  971925
>>971712

Так и задумано. В браузере для некоторых тегов прописаны правила CSS по умолчанию. Например, для заголовков задан увеличенный жирный шрифт, для абзацев - вертикальные отступы сверху и снизу, а для body или html - задан margin, чтобы текст на странице по умолчанию не прилипал к краям окна браузера. В книгах тоже ведь поля делают.

Если тебя это не устраивает, просто задай свои значения через CSS.

Не используй CSS reset, так как он отменяет много полезных вещей (те же отступы между абзацами, отступы в списках).

>>971682

Я обычно использую такие термины:

- margin: отступ
- padding: поля

Так как padding - это поля внутри элемента (как поля на странице книги), а margin - это отступы между соседними элементами (хотя в английском да, поля на странице называют page margins).

>>971643

Тут тебе бы лучше не копировать готовый код из учебника, где специально вставлена ошибка, а попробовать написать самому. Логика должна быть примерно такая:

- прибавляем проценты и комиссию к остатку долга (!не вычитаем ничего пока!)
- если остаток маленький, выплачиваем сколько осталось и уходим
- иначе платим 5000

«Платим» здесь значит уменьшаем долг и увеличиваем общую сумму выплаченного.

Код в учебнике явно неправильный, так как он вычитает 5000 даже если там осталось 1000 долга, он все равно пытается вычесть 5000 и уходит в минус.

>>971513

ООП нужен для двух целей:

1) класс объектов сочетает в себе свойства (что-то вроде переменных) и методы для работы с ними (что-то вроде функций). Многие вещи из реального мира хорошо описываются именно в виде объектов. Ну например, мы можем сделать класс Компания, у которого будут свойства "название", "список работников", "прибыль" и методы "нанять работника", "уволить работника".

Без объектов получается менее удобно - мы должны отдельно где-то хранить информацию о компании и отдельно функции для работы с ними. Удобнее, когда это собрано в одном классе с помощью ООП.

2) классы используют для организации кода, когда его много. Чтобы разбить код на N классов, каждый из которых отвечает за свою область, умеет решать одну задачу. Вообще, ООП тут не единственное решение, можно например разбивать код на модули (этого нет в PHP, но есть в других языках)

ООП описан в учебнике из ОП поста, в последней главе, подробно, и даны задачи для закрепления знаний.

Если ты хочешь работать программистом, то без знания ООП шансов мало. Везде он нужен. Фреймворки без него тоже не понять.

>>971546

Урок по исключениям, лучше изучать после ООП: https://github.com/codedokode/pasta/blob/master/php/exceptions.md
Аноним 11/04/17 Втр 17:51:47  971927
>>971490

Функции mysql.. устарели. Я бы советовал сразу переходить на PDO. Но там надо знать ООП.

Далее, у тебя еще есть такая проблема. Начинающие, когда пишут свои первые программы, просто пишут команды подряд, не разбивая код на части. Пока программы маленькие, это приемлемо. Но веб-приложения сложнее и больше и тут такой подход плохо работает. Надо отучаться писать стены кода и надо приучаться разбивать его на части, на отдельные функции. Разделять логику и HTML-шаблоны. Для этого можно почитать мой урок по MVC https://github.com/codedokode/pasta/blob/master/arch/mvc.md

Далее, у тебя банально идет незнание синтаксиса PHP. Нельзя писать $array[x], надо $array['x']. PHP должен выдавать ошибку, просто у тебя наверно отключен display_errors и ты их не видишь. Тебе надо перечитать мануал PHP внимательно.

Дальше, у тебя там скорее всего могут быть уязвимости. Мне лень тут все объяснять, предлагаю взять задание про студентов из ОП поста и прочесть там все комментарии, там все подробно расписано и большинство из написанного относится и к твоей задаче.

А пока у тебя просто не хватает знаний, код очень низкокачественный, ты пытаешься борду написать в том стиле, в котором начинающие пишут простые скрипты на 20 строчек.

Ну и HTML/CSS тоже надо подучить - <br> в HTML пишется без слеша в конце. Это не XHTML.

Советую изучить ООП, потом читать задачу про студентов.

>>971336

Ну там логика примерно такая: берем старшую купюру, считаем, сколько их надо выдать, берем купюру поменьше, считаем и так до самой младшей.

>>971171

А ты читал описание алгоритма Дейкстры? И вообще, про алгоритмы поиска пути? В Википедии есть список. Надо просто реализовать Дейкстру или любой другой алгоритм поиска пути на PHP. Вообще, алгоритмов много, есть например А-star и другие. В играх например они часто нужны.

>>970784

Еще добавлю, что по автозагрузке классов у меня есть отдельный урок https://github.com/codedokode/pasta/blob/master/php/autoload.md
Аноним 11/04/17 Втр 17:52:07  971928
>>970551

Установка плагинов, верстка, натягивание верстки, написание своих тем и плагинов. Я бы не советовал, это чуть лучше просто верстальщика.

>>970242

Изучи расширение gd к PHP, там есть функция imagecopyresampled для изменения размера картинки. Обычно пользователь загружает картинку, из нее делают 1 или несколько уменьшенных и далее выводят на сайте. Дополнительно иногда еще используют CSS, если размеры картинки должны как-то адаптироваться под размеры экрана.

>>970121

> Кстати, пишут, что ++$i вроде как работает быстрей.
А ты померяй.

>>970087

Прочитай мануал по json_decode. Он умеет выдавать массив, а не пародию на объект. Там по задумке по умолчанию для словарей JS создается объект класа stdClass в PHP, но на практике это не имеет особого смысла, лучше массив.

>>969877

В браузере есть кеш откуда он и взял старую картинку. Надо жать Ctrl + F5 по моему, чтобы заставить его принудиительно использовать картинку с сайта. Можешь почитать подробнее про кеширование в HTTP, но надо понимать протокол HTTP.

Также, советую не формировать ссылку прямо в HTML, а сделать функцию, которая генерирует ссылку на картинку.

>>969566

Если ты хочешь работать PHP-разработчиком, то от тебя потребуется писать и поддерживать веб-прилоежния, скорее всего на фреймворке, с использованием ООП и MVC, работать с SQL базой данных, писать скрипты на JS.

>>969499

Первая и вторая строка формируются одинаково, можно было использовать цикл из 2 шагов. Также, можно убрать промежуточные переменные $w1-$w8
Аноним 11/04/17 Втр 17:52:34  971929
>>969501

Нужно кириллические буквы заменить на другие похожие на них символы.

>>969462

На странице могут быть программы на языке Яваскрипт, которые изменяют DOM в процессе работы. Также, если исходный код содержит ошибочный HTML код, то он не попадет в DOM или будет исправлен браузером. view-source показывает тот код, который пришел с сервера.

Получить текущий DOM можно так в хромоподобных браузерах: открыть отладчик в браузере (Ctrl + Shift + I) и набрать команды:

var html = document.documentElement.outerHTML;
copy(html);

Текущий HTML-код страницы будет скопирован в буфер обмена.

Насчет ФФ не знаю, может сработает, может нет, надо проверять.

>>969451

Можно освоить фронтенд, задания по HTML/CSS/JS ждут тебя в ОП-посте. И тогда половинка тебе не понадобится.

>>969450

Изучи HTML формы. Там есть возможность добавлять скрытые поля в форму.

>>968998

Ты создаешь локальную переименную Super, которая видна только внутри той маленькой функции и никак не видна снаружи нее. Лучше в момент вызова функции копировать ее название и прототип предка в глобальные переменные. А mySuper будет использовать эти переменные:

mySuper.currentParentFunction = Object.getPrototypeOf(object)[name];

Также, ты вызываешь this() не передавая родительской функции правильный контекст (она будет вызвана не на объекте, а с this = window).

Но тут есть еще один подвох: что, если функция, вызванная через super(), вызовет super() еще раз? Поясню кодом:

Есть классы Parent extends Grandparent, Child extends Parent:

addMethod(Child.prototype, 'test', function () {
console.log('in Child');
var a = super(1, 2);
var b = super(3, 4);
return a + b;
});

addMethod(Parent.prototype, 'test', function (x, y) {
console.log('in Parent');
return super() + x + super() + y;
});

addMethod(Grandparent.prototype, 'test', function() {
console.log('in Grandparent');
return 100;
});

var child = new Child;
console.log(child.test()); // 410 если не ошибаюсь

Тут при исплоьзовании глобальных переменных они затрут друг друга и будет ошибка. Предлагаю подумать над решением этой проблемы.

Вместо __proto__ я советую использовать стандартный getPrototypeOf из ES5.

Также, есть еще одна оптимизация. Добавление обертки над методом делает его немного медленнее. Некоторые JS движки позволяют получить код функции в виде строки и проверить, есть ли там вызов super(). Если нет - можно не добавлять обертку. Так делает например бибилиотека classy. Хотя тут есть свой подвох: можно сохранить ссылку на super в другую переменную и вызывать через нее.

Ты можешь потом, если хочешь, посмотреть, как проблему решили авторы библиотек:

- classy: https://github.com/mitsuhiko/classy/
- наследование от John Resig: http://ejohn.org/blog/simple-javascript-inheritance/
- klass: https://github.com/ded/klass

Сейчас эти библиотеки отмирают потхоньку, так как в JS в ES6 (?) появился синтаксис для классов.
Аноним 11/04/17 Втр 17:53:24  971930
>>968966

> if ($i <= $quantity){
> }else {
Тут можно избавиться от if используя конструкцию: число купюр = наименьшее из (результат деления), (число купюр в банкомате).

Работает верно.

>>968669

Во-первых, оборачивать все в функцию не требуется. Никаких конфликтов там не будет, если ты только не пытаешься подключить несколько версий джейквери на страницу.

Во-вторых, как тебе написали ниже, DOM создается не мгновенно, а постепенно, по мере разбора документа. Если ты подключаешь свой скрипт в head, то в этот момент body еще не создано и поля с файлом нет, и повесить на него событие не получится.

Тебе там советуют использовать событие ready, но я дам другой совет. Помести во внешний файл функцию вроде function initFileUpload, а в HTML-коде под формой с файлом вставь ее вызов. Тогда она вызовется уже после того как форма появится в DOM.

А с ready надо ждать пока страница загрузится целком, это значит что событе будет навешено позже, может даже уже после того как пользователь что-то сделает на странце.

>>968673

Лучше не ready, а поместить вызов инициализации сразу после формы. Так она раньше иницииализируется, а в твоем случае достаточно одного долго грузяющегося скрипта чтобы ready задержалось.
Аноним 11/04/17 Втр 17:53:57  971932
>>968588

Электросеть

> PowerLine.prototype.countPowerAfterPass= function(power) {
Вот в этом варианте мне не нравится то, что мы должны вызывать Powerline чтобы просто вычесть 2 числа. И там ведь не может быть других вариантов, что остаток непереданной энергии вычисляется как-то по другому. То есть мне не очень нравится идея делегировать эту операцию вычистания в PowerLine.

Мне больше нравится вариант, когда мы запрашиваем у PowerLine, сколько она может передать энергии и сколько это стоит, а потом сами решаем, что делать, сколько мы передадим. Но каких-то рациональных объяснений я дать не могу, мне так просто больше нравится.

> и я решил написать счетчик через цикл, как и должно быть работают счетчики в реальном мире.
Ну а моделировать работу счетчика можно и делением.

> function House(apartments = 0) {
Тут интересное значение по умолчанию, видимо бывают жилые дома без квартир? Мне кажется, что с точки зрения логики тут не должно быть значения по умолчанию, число квартир обязательно надо указывать.

Ну и еще одно, я тут подумал, что если совсем-совсем идеально стараться делать, то правильнее всего было бы в базовом классе не делать свойств power/nightPower, а только 2 абстрактных метода getPower или getNightPower (в JS правда нет абстрактных методов, но это не моя вина). Ведь когда мы делаем свойство power, мы предполагаем что мощность - это какое-то свойство, которое сохраняет свое значение. Но в каких-то случаях она вычисляется по формуле, в House например это поле даже не заполняется, получается что в базовом классе его быть не должно. Извини, если сбил с толку противоречивыми объяснениями, просто сейчас это стало заметно в коде. Ну и это уже совсем придирки, в принципе текущий вариант меня устраивает полностью.

Определение типа переменной

> whatIsType
Имя функции начинают с глагола, getType().

А так, все верно.

Напиши функцию неглубокого копирования объектов и массивов

> Я пытался получить новую функцию из старой.
Ну да, но ты привязал не то значение this, ты привязал значение clone в качестве this, и при таком вызове:

function fn() {}
var fnCopy = clone(fn);
fnCopy.call({});

Функция fn получит другой this.

Клонировать функцию можно наверно так:

function () { return fn.apply(this, arguments); }

и потом скопировать свойства объекта, но в задаче этого все равно не требуется.

> var clone = new Date();
> clone.setTime(object.getTime());
лучше new Date(object.getTime())

Решение теперь верное.

Глубокое копирование

Ок, верно.
Аноним 11/04/17 Втр 17:54:16  971933
>>968294

>reporting($op, $result, $number, $char);
Это можно было не копировать 4 раза а просто поставить перед if.

А так, верно решено.

>>968288

Нет, нельзя. Есть eval, но от него вреда больше чем пользы.

>>968256

$letters[$random] наверно?

>>968254

"найти значение элемента массива по индексу в PHP". Эти термины по моему описаны в начале урока про массивы.

>>968103

Наверно потому что сервер работает от какого-нибудь пользователя вроде SYSTEM. Ты там umask или chmod не вызывал?

>>968081

Надо делать редирект на страницу с файлом после успешной загрузки. Это назвается паттерн Post/Redirect/Get.

>>967927

Скорее всего сервер взломан. Самые популярные способы взлома:

- кто-то сохранил пароль от FTP, вирус его нашел и закачал вирус на сервер
- на сервере стоит какая-нибудь старая дырявая CMS
- кто-то упер пароль от панели управления хостингом

В простейших случаях вирус дописывает вредоносный код в HTML/PHP файлы, в более сложных - прописывается куда-нибудь в .htaccess. Лучше всего забекапить файлы, очистить папку на хостинге, проверить файлы и закачать заново. И поменять пароли.
Аноним 11/04/17 Втр 17:54:54  971934
>>967992

Тут ддос, крупный хостинг таким заниматься не будет.

>>967885

Может у тебя там маргины какие стоят или просто блоки слишком широкие и не помещаются? Бери отладчик в браузере и изучай.

>>967873

> Используется же для создания работников в цикле getEmpolyees.
Там переменная, а не поле. Поле в объекте не используется.

>>У тебя при наследовании надо обязательно прописать значения зарплаты, потребления кофе итд.
> Это подразумевается, когда ты создаёшь новую профессию, зачем её создавать не задав ей никаких параметров?

Это подразумывается в твоей голове, а я предлагаю заложить это ограничение в код, чтобы любому сразу было видно. Для этого и придуманы абстрактные методы - чтобы указать, что обязан задать тот, кто пишет класс-наследник.
Аноним 11/04/17 Втр 18:51:23  971965
>>971671
>>971699
>>971925
А как теперь остаток минуснуть? Я вроде это прописал, но что-то не так http://ideone.com/T5CDtr
Аноним 11/04/17 Втр 19:55:55  971996
Норм метод для скачивания файлов, или забыл чего важного?
public function download(\Slim\Http\Request $request, \Slim\Http\Response $response, $args)
{
$id = $request->getAttribute('fileId');
$file = $this->container->getFilesGateaway()->getFileByID($id);
$path = '..\\' . $file->link;
$fileName = $file->realName;
if (PHP_OS == 'WINNT') {
$path = iconv('utf-8', 'windows-1251', $path);
$fileName = iconv('utf-8', 'windows-1251', $fileName);
}
$newResponse = $response->withHeader('Content-type', 'application/octet-stream')
->withHeader('Content-Description', 'File Transfer')
->withHeader('Content-Disposition', 'attachment; filename=' . $fileName)
->withHeader('Content-Length', filesize($path));
readfile($path);
return($newResponse);
}
Аноним 11/04/17 Втр 19:56:59  971997
>>971965
Короче, анон. Я тут в этом треде html&css учу, сам с java работаю. У тебя какой-то хуевый очень выходит цикл, ты берешь for на 20 итераций, который делает 12 из них и выходишь из него через break. Это индийский стиль программирования, за него православные пацаны могут с вертухи прописать. Гугли, в php должен быть цикл while, когда ты точно не знаешь какое число итераций сделает цикл. Чуть позже детально вникну почему у тебя остаток там.
Аноним 11/04/17 Втр 20:06:53  972001
>>971965
>>971997
Я вник вообщем. Из-за того что ты выходишь из цикла через break (пальцы тебе за такое сломать), у тебя не доходит до последней строчки, где выводит последнюю оплату. Давай не гавнокодь, исправляй.
Аноним 11/04/17 Втр 22:26:51  972123
>>971922
Спасибо!
Картинки не думал брать, конечно. Я как раз хотел всё сделать текстом, чтобы удобно было искать интересующие моменты: я когда сам проходил всё по твоему учебнику, часто путался, постоянно пересматривал всё (в принципе, в этом тоже был плюс).
Задачи - только формулировки, пояснения и прочее хочу сделать для совсем начинающих.
Хочу ещё много добавить простых задач, чтобы после просто машинально применяли циклы разные, switch, допустим, разные встроенные функции. По ООП хочу гораздо больше всего сделать, но не с такими сложными задачами. Зандстра страницы с 80-й показался совсем сложным, не таким наглядным, каким хотелось бы всё видеть. Многое пропустил у него, а сейчас читаю про проектирование - ну неплохо, вроде бы многое понятно.
Всё это хочу сделать и для того, чтобы самому лучше понять многое.
Люблю наглядность, прогресс-бары, всякие milestones - вот просто в духе этого хочу многое реализовать, сделать такой чёткий курс.
Чувствую, что так или иначе буду и из твоего учебника черпать, вот и хотелось испросить разрешения.
Аноним 11/04/17 Втр 22:28:26  972125
Выручайте ОП, аноны. Не могу в 10 задание по HTML&СSS, там где картинки кошек.
https://github.com/codedokode/pasta/blob/master/html/html.md
Вот сам код:
https://codeshare.io/a3yENm
Проблема собственно с чем:
- Как правильно сделать внутри картинки заголовок? Я делал через position: relative и в него вложил position:absolute. Не уверен, что так и задумывалось.
- Как кусочек текста из figure который обвернут в span перенести на след строку? Его тоже делать absolute и выравнивать/выставлять margin`ом?
- Я никак не могу нагуглить, как сделать сноски слева от текста. Там в тег em вставлен собственный атрибут, я как понимаю, нужно его как-то использовать, но как именно? Можно ссылку какую-то или как это называется?
Аноним 12/04/17 Срд 00:31:10  972190
>>972125
- так пойдет
- display:block
- предполагаю, что что-то вроде этого:
em[data-ref="1"]:before {
left: 20px;
display: block;
position: absolute;
color: grey;
font-size: 0.7em;
content: '[1]';
}
Аноним 12/04/17 Срд 02:02:14  972216
Поясните кратко за ООП. Наткнулся сегодня, как то не очень понимаю.
Единственное, что уразумел, так это то, что это, по сути своей, взаимодействие классов, которые имеют разные добавляемые опции, вроде интерфейсов. Хотя, очень вероятно я могу и ошибаться
Аноним 12/04/17 Срд 02:04:48  972217
>>972216
$apple->type = "apple";
Зачем эти стрелочки >? Что оно относится к типу, указанному в главклассе?
Аноним 12/04/17 Срд 02:07:49  972218
>>972217
Для чего нужно public? Чтобы его видели?
Аноним 12/04/17 Срд 02:12:03  972219
>>972218
Гугл блять!
Аноним 12/04/17 Срд 02:12:55  972220
>>972218
<?php
class Person {
public $isAlive = true;
public $firstname;
public $lastname;
public $age;
}
$teacher = new Person();
$student = new Person();

echo $teacher->isAlive;
?>

Оно мне 1 показало. Почему? Откуда?
Аноним 12/04/17 Срд 02:13:53  972221
>>972219
Да ладно тебе, не стукай. Давай, постою, как цапля?Курлык
Аноним 12/04/17 Срд 02:39:51  972224
>>972220
http://php.net/manual/ru/language.types.boolean.php
Аноним 12/04/17 Срд 02:44:41  972225
>>972224
>>972217
>>972216
Разобрался уже. Примерно.
Аноним 12/04/17 Срд 02:46:01  972227
>>972224
То есть $teacher имеет только одно isAlive? Или isAlive только один учитель?
Аноним 12/04/17 Срд 02:50:14  972228
>>972227
Нет.
Аноним 12/04/17 Срд 02:51:11  972230
14874149879340.jpg (5Кб, 200x195)
>>972228
Аноним 12/04/17 Срд 02:51:34  972231
>>972230
Почитай ОП пост.
Аноним 12/04/17 Срд 04:20:06  972242
Салам. Обязательно ли в задаче про список студентов использовать Апач? В ПХП же есть встроенный веб-сервер и все задачи из учебника решались норм без Апача
Аноним 12/04/17 Срд 06:32:21  972254
>>972242
А ты вектор решил?
Аноним 12/04/17 Срд 09:48:33  972289
f193b01405290e8[...].jpg (10Кб, 184x187)
Может в тг или конфе вк cоберёмся? Заебало гуглить тупые вопросы часами.
Выгода для всезнаек и не нуфагов сомнительна от такого кооператива.

Например вынес подключение к БД в отдельный файл bd.php, в нём прописаны настройки подключения и собственно подключение к бд, подключаю этот файл к скрипту, в котором необходимо выполнить действия с БД, но внезапно записанный объект в $database не является глобальной переменноq, т.е использовать его в своём коде я не могу, нужно походу объявлять глобальную переменную или ещё какой-нибудь хуйни наворотить.
Аноним 12/04/17 Срд 10:38:42  972305
>>972289
Ну так запиши это в виде класса, подключить класс и создай объект класса, использовать лучше синглетон
Аноним 12/04/17 Срд 11:08:09  972316
>>972289
Иди, собирайся. Тут сидят люди которые не пользуются социалочками и прочей ебаниной.
Аноним 12/04/17 Срд 11:17:48  972320
Аноны, тут кто-то недавно вкидывал ссылки на канал какого-то чувака, который там еще рассказывал про свою историю, типа он пять лет мучился с самообучением и все в таком духе. У кого осталось? Просто ссылка проебалась, очень обидно.
Аноним 12/04/17 Срд 11:20:12  972321
>>972227
Булево значение true - это 1, а false - 0, NULL.
Поставь там свойство public $isAlive = false; - посмотри сам.
Иногда нужно для работы чего-либо возвращать true или false.
Например, ты дойдёшь до задачи про "Вектор", там у работника будет свойство $isBoss. Вот если $isBoss = true, то от этого зарплата в два раза выше, а количество отчётов в два раза меньше.
Но как так ты дошёл до ООП, а с таким не сталкивался? Непонятно.
Аноним 12/04/17 Срд 11:22:05  972323
>>972289
Давно есть у нас конфа в Телеграме: https://t.me/PHP_club
Помогаем по мере сил стремящимся.
Аноним 12/04/17 Срд 11:34:07  972327
Сап. Нужен совет от знaющего Laravel

В контроллер запихнул метод, который тригерится в определённом случае и отправляет всем email через Mail::to...
Пока в базе около 5 юзеров, и оно нормально работает, но какова вероятность что когда их будет несколько сотен, оно всё наебнётся?
Аноним 12/04/17 Срд 12:01:36  972335
>>972216

Кратко тут не пояснить. В ОП посте есть учебник, в нем глава про ООП - прочти ее и реши задачи в ней, а если что-то непонятно - то напиши вопрос.
Аноним 12/04/17 Срд 12:01:58  972336
>>972242

Не обязательно.
Аноним 12/04/17 Срд 13:10:54  972355
>>972327
Такие вещи вряд ли завалят основной процесс, но если пользователей будет несколько тысяч, то запрос к серверу, который вызвал это событие будет достаточно медленоват. Лучше такие блокирующие вещи выносит в фон, почитай про сервера очереди задач типа gearman или rabbitMQ.
Аноним 12/04/17 Срд 13:23:18  972357
>>972355

Вообще, рассылка может отвалиться по таймауту. Обычно там стоит execution_time_limit или как оно называется.
Аноним 12/04/17 Срд 13:27:22  972358
>>972254
Да
Аноним 12/04/17 Срд 13:32:44  972361
>>972358
Где?
Аноним 12/04/17 Срд 13:52:20  972370
http://phpclub.rf.gd/pr/res/945059.html#955473
>Также, если интересен вопрос оптимизации выборки, то тут придется усложнять схему БД. Сейчас, чтобы вывести посты на главной, приходится делать много запросов, и они не очень эффективные (например, подсчет числа постов в каждом треде отдельно).
>
>Можно добавить в тред поля для хранения числа постов, а также ссылки на последние посты (либо сделать это через отдельную таблицу связи тред - последние посты). Соответственно данные можно будет выбрать в 2 захода: выбираем треды, затем собираем из них id ОП-поста и последних постов и выбираем их одним запросом. Но это конечно немного усложнит работу с базой, так как при изменениях надо обновлять эти дополнительные поля. Но зато снизит нагрузку на базу при выборке.
Этот совет будет актуален, если я перешел на Доктрину? Я реализовал ассоциацию Один-Ко-Многим и теперь, когда я получаю Тред, я получаю его Посты и Файлы, и теперь подсчет количества постов выполняется посредством метода count() класса ArrayCollection.

https://github.com/someApprentice/phpClub/blob/master/src/Threader.php#L115
https://github.com/someApprentice/phpClub/blob/master/src/Thread.php#L15
https://github.com/someApprentice/phpClub/blob/master/src/Post.php#L12

К тому же, из-за того что в этом классе не предусмотрена возможность получения только определенных записей, мне приходиться "выкидывать" лишние посты чтобы получить первый и последние три поста.

https://github.com/someApprentice/phpClub/blob/master/src/Threader.php#L117-L125

Мне следует получать посты через свои запросы?


>Также, строить цепочку постов через выбор всех постов в БД очень неэффективно. Лучше сделать таблицу связи "многие-ко-многим", связывающую посты, может быть такого вида:
>
>from_id | to_id
>
>или такого
>
>from_id | to_id | depth
>
>То есть мы берем пост, и для каждой ссылки в нем вставляем записи в таблицу связи. Можно вставлять только ссылки из самого поста, тогда надо делать несколько SQL запросов, чтобы построить цепочку, а можно вставлять ссылки еще и на посты с большей глубиной вложенности, тогда цепочка выбирается одним запросом, но таблица будет больше по объему.
А как из с таблицы с глубиной вложенности получить всю цепочку одним запросом?

Из примера выше (http://phpclub.rf.gd/pr/res/945059.html#956138):

>2) таблица хранит для каждого поста ссылки на все листья дерева, то есть для примера выше там будут записи

>A -> B
>A -> C
>A -> D
>B -> C
>B -> D
>C -> D

Допустим нужно получить цепочку поста B. Ссылки на дочерние ветки он имеет, но на родительские нет. В любом случае, нужно будет искать родителей в несколько запросов.


>И еще, может стоит добавить опцию, чтобы сохранялись и выводились ссылки на тред в архиве /pr/ и в архиваче, если они есть. Просто, чтобы было.
А как это реализовать? В /pr/ к ссылке на архив треда добавляется дата сохранения, которую неизвестно как получить, а на архиваче используются свои собственные id тредов.
Аноним 12/04/17 Срд 14:00:38  972375
>>972357
Понял, спасибо. Пойду гуглить
Аноним 12/04/17 Срд 14:04:21  972377
>>972370

> Я реализовал ассоциацию Один-Ко-Многим и теперь, когда я получаю Тред, я получаю его Посты и Файлы, и теперь подсчет количества постов выполняется посредством метода count() класса ArrayCollection.

Ну так это сверхнеэффективно, выбрать кучу постов в память только для того, чтобы получить их количество или получить первые 3.

> Мне следует получать посты через свои запросы?
Да.

> А как из с таблицы с глубиной вложенности получить всю цепочку одним запросом?
Давай я пример приведу. Допустим пост A ссылается на B, C, пост B ссылается на D, E. Получаются такие записи:

post | reference | depth
A | B | 1
A | C | 1
A | D | 2
A | E | 2
B | D | 1
B | E | 1

Соответственно чтобы получить всю цепочку начиная с A, бы делаем выборку WHERE post = A

Если же мы делаем одноуровневую таблицу, то получается так:

post | ref
A | B
A | C
B | D
B | E

И мы должны сделать 3 запроса:

WHERE post = A // получаем B, C
WHERE post IN (B, C) // получаем D, E
WHERE post IN (D, E) // ничего

> А как это реализовать? В /pr/ к ссылке на архив треда добавляется дата сохранения, которую неизвестно как получить, а на архиваче используются свои собственные id тредов.
Прописывать в конфиг какой-нибудь вручную.
Аноним 12/04/17 Срд 15:26:40  972417
>>972377
>Соответственно чтобы получить всю цепочку начиная с A, бы делаем выборку WHERE post = A
Но если мы хотим получить всю цепочку начиная с B, нужно будет сделать ещё один запрос WHERE reference=B, и так далее, пока мы не найдем всех предков.

Аноним 12/04/17 Срд 15:28:51  972418
>>972361
Дома
Аноним 12/04/17 Срд 16:10:58  972429
>>972418
И думает, что выкрутился.
Аноним 12/04/17 Срд 16:21:13  972436
Подскажите, пожалуйста, дебилу простым ЧЕЛОВЕЧЕСКИМ языком, как работает оператор JOIN (и RIGHT JOIN с LEFT JOIN). И пример практический. Бесконечное спс.
Аноним 12/04/17 Срд 16:25:22  972441
>>972436
>Бесконечное спс
Ну что ты, не стоит благодарности.
Аноним 12/04/17 Срд 16:31:06  972445
>>972436
Гугл, поиск картинок, "SQL JOIN" вон там -->>
Аноним 12/04/17 Срд 16:36:22  972449
>>972436

A JOIN B ON cond берет по очереди каждую запись из таблицы A и присоединяет к ней по очереди все строки из таблицы B. Это называется полное декартово произведение таблиц. Если указано условие ON, то соединяются только пары записей, соответствующие условию в нем.

Допустим таблица A содержит записи A1, A2, A3, а таблица B - B1 и B2.

SELECT * FROM A JOIN B вернет такую таблицу:

A1 | B1
A1 | B2
A2 | B1
A2 | B2
A3 | B1
A3 | B2

То есть результат содержит все возможные сочетания записей из 2 таблиц. Если добавить условие ON, то будут взяты только пары, соответствующие указанному условию. Оно может быть любым, и будет проверяться по очереди для каждой пары.

Если в таблице A - N записей, а в B - M то в итоге после джойна без условия получится N*M записей.

LEFT JOIN отличается от STRAIGHT JOIN тем, что если для записи Ax не нашлось ни одной пары в B, соответствующей условию, то в результат добавляется пара

Ax | NULL

То есть при LEFT JOIN в результате гарантированно встретится хотя бы 1 раз каждая запись из A.

RIGHT JOIN - то же самое, то только для таблицы B, гарантируется что в результате будут все записи из нее, даже если им не найдется пары, соответствующей условию.

Если что-то непонятно - уточняй.

У нас в ОП посте есть задачи по SQL. Советую решить.
Аноним 12/04/17 Срд 16:49:37  972460
>>972125
> Я никак не могу нагуглить
Я загуглил "css data attributes" и нашёл такую статью: https://developer.mozilla.org/en-US/docs/Learn/HTML/Howto/Use_data_attributes

>>972436
> дебилу простым ЧЕЛОВЕЧЕСКИМ языком
>>972216
> Поясните кратко за ООП
Почему у вас такой странный подход к изучению: "поясните кратко", "объясните дебилу", "простым языком"? За что вам платить тогда, вас же с таким подходом любой заменить может.
Аноним 12/04/17 Срд 17:11:37  972472
>>972125

> - Как правильно сделать внутри картинки заголовок? Я делал через position: relative и в него вложил position:absolute. Не уверен, что так и задумывалось.

Тут надо использовать абс. поз., так как темная плашка позиционируется поверх блока с картинкой, не влияет на расположение других блоков

> - Как кусочек текста из figure который обвернут в span перенести на след строку? Его тоже делать absolute и выравнивать/выставлять margin`ом?

Какой способ позиционирования позволяет располагать блоки вертикально? Очевидно, display: block

> - Я никак не могу нагуглить, как сделать сноски слева от текста. Там в тег em вставлен собственный атрибут, я как понимаю, нужно его как-то использовать, но как именно? Можно ссылку какую-то или как это называется?

Тебе надо почитать про генерируемый контент и CSS-свойство content:

http://htmlbook.ru/css/content
http://htmlbook.ru/css/attr
http://htmlbook.ru/css/counter-increment
http://htmlbook.ru/css/before
http://www.umade.ru/resources/specifications/CSS2/generate.html



Аноним 12/04/17 Срд 21:25:27  972615
>All Unicode regex engines discussed in this tutorial treat any single Unicode code point as a single character. When this tutorial tells you that the dot matches any single character, this translates into Unicode parlance as "the dot matches any single Unicode code point". In Unicode, à can be encoded as two code points: U+0061 (a) followed by U+0300 (grave accent). In this situation, . applied to à will match a without the accent. ^.$ will fail to match, since the string consists of two code points. ^..$ matches à.
>Matching a single grapheme, whether it's encoded as a single code point, or as multiple code points using combining marks, is easy in Perl, PCRE, PHP, Boost, Ruby 2.0, and the Just Great Software applications: simply use \X. You can consider \X the Unicode version of the dot.
http://www.regular-expressions.info/unicode.html
Это правда, что с юникодом регексы надо писать иначе, чем до юникода? В том, что касается "количества символов", например в "этот символ повторяется n раз", речь идёт о юникод-графемах или о "code points"? Я как-то не замечал этого, работая с юникодовскими регексами.
Аноним 12/04/17 Срд 21:32:13  972624
>>972615

Юникод - сложная штука, к сожалению. действительно, буквы вроде "а с кружочком сверху" может быть закодирована 2 способами: как один символ или как буква a + модификатор-кружочек. Это не единственный подвох, в Юникоде полно других.

Более того, если ты пишешь эту букву в регулярке, тоже неизвестно, как текстовый редактор ее закодирует.

Данная проблема отчасти решается нормализацией строки перед применением регулярки. Нормализация приводит такие символы к единому виду.

Про \X кстати первый раз слышу.

> В том, что касается "количества символов", например в "этот символ повторяется n раз", речь идёт о юникод-графемах или о "code points"?
Число повторений идущего перед квантификатором выражения, то есть:

.{10} = 10 codepoints
\X{10} = 10 graphemes

К счастью, ситуации, когда надо отмерять ровно N сложносочиненных символов, довольно редки.
Аноним 12/04/17 Срд 21:44:36  972653
устроился в крупную контору. я так убого верстаю, убейти миня. меня по ходу держат тока из жалости и то ненадолго.
Аноним 12/04/17 Срд 21:56:53  972667
>>972216
>Мэтт Зандстра — PHP: Объекты, шаблоны, методики программирования
Качай это, он тебе кратко за одну книгу весь ООП пояснит.
Аноним 12/04/17 Срд 22:00:29  972670
>>972653

В ОП посте есть задания на HTML, изучай.
Аноним 12/04/17 Срд 22:08:23  972678
>>972670
нет, у нас в конторе уровень гораздо выше ваши школо-задачек из оп-поста.
сейчас уже одна практика и рефакторинг от коллег мне, наверное, поможет. или не поможет.
Аноним 12/04/17 Срд 22:17:32  972684
>>972436
Есть две таблицы, piki_tochenu и hui_drochenu.

piki_tochenu
pika_id sidit_id
1 я
3 мать

hui_drochenu
hui_id sidit_id
1 я
2 мать
3 я

select * from piki_tochenu
join hui_drochenu on hui_drochenu.sidit_id = piki_tochenu.sidit_id
where piki_tochenu.sidit_id = 'я';

Сделаешь джойн, будешь сидеть сразу на хуе 1 и пике 1, потом на хуе 1 и пике 3. До джойна сразу на стольких не мог сидеть.
Аноним 12/04/17 Срд 22:38:26  972692
>>972624
> > В том, что касается "количества символов", например в "этот символ повторяется n раз", речь идёт о юникод-графемах или о "code points"?
> Число повторений идущего перед квантификатором выражения, то есть:

> .{10} = 10 codepoints
> \X{10} = 10 graphemes
Бло, я мог бы так критически облажаться. Не попадись мне эта статья.
А php вполне мог бы заменять кодпойнты на графемы, если ему указать флаг u, это вроде как логично. Теперь кроме флага u придётся ещё и \X ставить, если есть хотя бы намёк, скажем, на шведский алфавит, или на китайские знаки.
Аноним 12/04/17 Срд 22:40:40  972694
>>972472
Да, посмотрел все, спасибо. Вроде внешне вышло все как и должно быть, но меня не покидает ощущение, что я гавнокожу. Я правильно понимаю, что по CSS и HTML вообще нету никаких code conventions, или канонов по оформлению разметки? Если не впадлу и есть время скажите, где кривая реализация в CSS. P.S. Все тот же урок с котом.
https://codeshare.io/21V8Z1
Аноним 13/04/17 Чтв 00:03:50  972726
>>972678

Может быть ты приведешь примеры сложных задач? Чтобы можно было дополнить наши задачи?
Аноним 13/04/17 Чтв 00:25:47  972738
>>972694
Двачую этого. Есть где-то свод правил по типу PSR для PHP, как CSS и HTML оформлять? Имена классов, селекторы и прочее.
Аноним 13/04/17 Чтв 08:08:35  972808
скажите, пожалуйста, в пхп мире используют докер?
Аноним 13/04/17 Чтв 08:17:09  972810
Снимок.PNG (25Кб, 1323x416)
2.PNG (9Кб, 1057x95)
3.png (6Кб, 345x163)
Получается, чтобы регекс мог найти любой допустимый словесный символ без цифр и знаков, надо использовать юникод-класс который обещает быть не быстрым. А вот знаки в него не включены, для включения нижних подчёркиваний нужно использовать старый добрый \w (с указанием 'u' после регекса). По сути, \w всё так же остаётся самым удобным для проверки слов классом.
А вот дефисы \w не видит, чтобы проверять слова,интересно, почему так сделали. Чтобы проверять слова с ними, придётся дописать его отдельно.

занимательный юникод
Аноним 13/04/17 Чтв 09:03:45  972820
>>972694
>>972738
гуглите бэм-нэйминг.
но это только один из возможных вариантов
Аноним 13/04/17 Чтв 09:14:01  972823
Как сделать полный жесткий revert в git до состояния определенного коммита? Чтобы не просто вернуть файлы в состояние, но и полностью удалить новые файлы, которые были с того момента добавлены.
Аноним 13/04/17 Чтв 10:35:55  972842
>>972823

git checkout commit-id

Это не удалит коммиты из репозтория, но приведет файлы в рабочей дректории в соответствие с указанным коммитом.

Аноним 13/04/17 Чтв 12:29:57  972870
>>972823
git reset --hard
Аноним 13/04/17 Чтв 13:18:06  972880
Добрый пхпанон, поясни за эту тему. О чем тут речь?
https://www.owasp.org/index.php/AJAX_Security_Cheat_Sheet#Always_return_JSON_with_an_Object_on_the_outside
Аноним 13/04/17 Чтв 16:04:14  972975
>>971928

>Прочитай мануал по json_decode. Он умеет выдавать массив, а не пародию на объект. Там по задумке по умолчанию для словарей JS создается объект класа stdClass в PHP, но на практике это не имеет особого смысла, лучше массив.

Не всё так просто, если я не знаю тонкостей json и php. Читал. Про массив-то я знаю. Собственно почему в этом случаее объекты хуже массивов я так и не понял. Расскажи пожалуйста. Видел комменты в стаковерфлоу к похожим задачкам и там видел пару реакций на stdClass - что это зло, но не понял сути.

Т.к. я нуб, то погуглив, наткнулся на это
http://stackoverflow.com/questions/18640607/what-is-better-stdclass-or-object-array-to-store-related-data
Там был тест скорости, который показал, что у меня объекты быстрей на 0.5ms.

И такой тип записи выглядит полее удобочитаемым
$json_array->sub_1->sub_2[$i]->sub_3
чем такой
$json_array["sub_1"]["sub_2"]["sub_3"]

А так, конечно хотелось бы знать, что правильней и применять это.
Аноним 13/04/17 Чтв 16:12:39  972982
>>972975
>объекты
Имею ввиду классы vs массивы, конечно.
Аноним 13/04/17 Чтв 16:23:15  972994
>>972975

Для массивов есть много функций для работы с ними (array_keys и так далее), а для stdClass - нет. При этом stdClass это пародия на нормальный класс (у него нет ни полей ни методов). Таким образом, плюсов у stdClass нет, а минусы есть - для него нет удобных функций, как для массивов.
Аноним 13/04/17 Чтв 16:24:14  972995
>>972975

Ну то есть stdClass это по сути массив, для работы с которым нет функций вроде array_keys.
Аноним 13/04/17 Чтв 18:07:03  973059
Как годно оптимизировать сайт под разные экраны?
Аноним 13/04/17 Чтв 18:26:19  973074
>>973059
width: 100% на все
Аноним 13/04/17 Чтв 19:42:21  973134
>>972994
>>972995
Яааасно. тогда конечно нет смысла его юзать. Спасибо анончик.
Аноним 13/04/17 Чтв 19:47:55  973135
ОП, аноны, помогите. Смотрите, я делаю урок №11 на три кнопки (те что ОП советовал, по CSS). Все работает, кроме одной вещи, у меня кнопка хоть убей, но не хочется оставаться нажатой. Уже все перепробовал. Нашел похожие кнопки на bootstrap, посмотрел как они там сделаны (в source коде), запутался окончательно. НЕ работает именно момент .button:focused, input:checked тоже самое. ЧЯДНТ
Собственно сам код: https://codeshare.io/aJ7x76
Аноним 13/04/17 Чтв 20:05:16  973151
>>973135
Я не читал 11й урок ещё, но может поможет: http://jsfiddle.net/tovic/ve8mU/light/
Аноним 13/04/17 Чтв 20:22:05  973158
>>973151
Блин, ОП - линковал похожее, тут такое ощущение что трабла какая-то из-за того что input вложен в label. Если его достать и прописать через for и id, что просили не делать, то кнопка нажимается, но слетает почему то border-radius. Как так жить, ОП надежда на тебя.
Аноним 13/04/17 Чтв 20:47:45  973182
>>973158
+ ОП, еще пишет что для кнопки использовал три тэга, label + input + i. Зачем нужен i?
Аноним 13/04/17 Чтв 20:53:38  973187
Премного благодарен за предоставленный гайд для начинающих, все что гуглится просто невероятный кал, лишь редкие исключение можно изучать но даже там много пердолинга даже с самой базой. Добра.
Вопрос по SQL Аноним 13/04/17 Чтв 23:19:45  973263
2017-04-13-1512[...].png (27Кб, 408x633)
В общем треде не ответили, спрошу тут.
Задача с sql-ex.ru: "Get the makers who produce only one product type and more than one model. Output: maker, type."

Я написал вот такой запрос:
select maker from product
group by maker
having count(model) > 1 and count(distinct type) = 1

Он правильный, но в решении требуется помимо maker выбрать ещё и type, пишу так:
select maker, type from product
...

И получаю ошибку: "Column is invalid in the select list because it is not contained in either an aggregate function or the GROUP BY clause SQL Server". Насколько понял, это из-за того, что SQL-сервер не понимает, какой type выбрать из сгрупированных. Переписываю запрос так:
select maker, max(type) from product
...

Такое решение мне засчитали. Но не выглядит ли это костыльно? Тут полный перебор сгруппированных значений, когда хватило бы получения первого значения.
Аноним 14/04/17 Птн 00:57:26  973302
>>973263

> Тут полный перебор сгруппированных значений, когда хватило бы получения первого значения.

А какое из них первое? Порядок ведь не задан. Конечно если они одинаковые то разницы нет, но видимо в SQL решили не заморачиваться с исключениями и обязать всегда использовать агрегатные функции.

Алсо у тебя в COUNT(model) по моему DISTINCT пропущен.

COUNT(model) это SUM(model IS NOT NULL) и это явно не то что нужно.
Аноним 14/04/17 Птн 01:39:36  973312
Поясните за do while.

$i = 2;
do {
$i++;
echo $i;
}

while($i == 10);
echo $i;

Он мне показывает 3. Но разве он, после while($i == 10);, не должен показывать 10? Ведь задал условие, что бы $i = 10.
Аноним 14/04/17 Птн 07:36:04  973342
>>973135
Бамп, так и не разобрался. У меня вышло сделать с не вложенным input в label. По другому никак.
>>973312
Смотри, ты задаешь условие в while, а не конечный результат. У тебя сейчас цикл выглядит таким образом, пока i РАВНО - 10, выполнять условие цикла. После первой итерации (которая выполнится даже если условие выполнено, это особенность do-while) i у тебя равен 3 и происходит выход из цикла. Короче говоря, тебе нужно поменять условие в цикле с i равно 10, на - пока i не станет равно 10.
Аноним 14/04/17 Птн 07:37:59  973343
>>973342
которая выполнится даже если условие НЕ выполнено.
fix
Аноним 14/04/17 Птн 08:09:44  973345
Опчик, проверь на телефоны задачу плз https://regex101.com/r/y1Q0Mm/5
Аноним 14/04/17 Птн 10:05:41  973376
Можно ли как нибудь представить перечисление в виде (x, y, z) как таблицу чтобы при-JOIN'ить её к другой таблице? IN не подходит
Аноним 14/04/17 Птн 10:28:23  973382
>>973376
То-есть изначально мне надо выбрать все данные по дням, но у меня есть дни в которые данных в нужной таблице нет, и мне нельзя эти дни пропускать, а надо вывести день и 0 результатов.
К примеру '2017-04-14', 0. In не подходит, так как он игнорит эти дни, идеальным вариантом взять список дней и приJOIN'ить их к таблице нужной по которой шла бы группировка, но я не могу написать в виде SELECT * FROM ('2017-04-14', '2017-04-13')
Аноним 14/04/17 Птн 10:28:58  973383
>>973302
http://www.sql-ex.ru/help/select13.php#db_1
Извиняюсь, что сразу схему не показал. Столбец model в таблице product уникален, поэтому от DISTINCT ничего не поменяется.

Ещё вопрос, в таком запросе:
select from A where id > (select min(id) from B)
Сколько раз выполнится вложенный запрос? Столько, сколько строк в A? Если да, то можно ли как-то это оптимизировать?

Аноним 14/04/17 Птн 10:32:07  973384
>>973382
Как вариант - создавать временную таблицу но думаю это будет довольно медленный вариант в 4 хода - создание временной таблицы, наполнение данными, выборка, удаление таблицы
Аноним 14/04/17 Птн 10:38:43  973387
>>973384
Что то наподобие вышло

SELECT d.date, if (o.count is null, 0, o.count) as count, if(o.salary is null, 0, o.salary) as salary FROM `#date` as d
LEFT JOIN (SELECT count(*) as count, sum(total) as salary, DATE(date_added) as `date` FROM `order`) as o on o.date = d.date

Но лучше бы без временной таблицы.
Аноним 14/04/17 Птн 10:41:30  973389
>>973387
Извините что нафлудил - забыл группировку по DATE(date_added) во внутреннем запросе
Аноним 14/04/17 Птн 10:57:50  973395
121.png (33Кб, 792x682)
Как сделать дешифровку через array_flip(), что бы текст обратно переводился ?
Аноним 14/04/17 Птн 12:49:49  973439
>>973395
$flipped = array_flip($code);
Аноним 14/04/17 Птн 13:51:30  973465
>>973345
Оп и еще следом за ней на приведение номера в корректный вид https://ideone.com/DV6lCb
Аноним 14/04/17 Птн 14:16:36  973479
Показатель в ~400 rps на машине с конфигурацией 2 x 2.7 ghz, 512 mb ram можно считать нормальной?
Кстати, интересует еще опыт местных в масштабирвоание базы под инсерты апдейты.
Аноним 14/04/17 Птн 14:49:34  973499
ОП, аноны, поясните пожалуйста. Вот 12 задание по CSS, нужно сверстать эти вкладки. Тут как я понял анимация сделана на JS. Без него никак правильно?
Аноним 14/04/17 Птн 14:49:55  973501
>>973499
Забыл ссылку http://cssdeck.com/labs/full/css-responsive-tabs//noframe#dontkillanim
Аноним 14/04/17 Птн 14:55:52  973506
Подскажите, как лучше фильтровать полученные от пользователя данные?
Аноним 14/04/17 Птн 14:58:10  973510
>>973499
Нет, не правильно. Можно сделать и без него. В задании есть подсказка:
>подсказка: http://habrahabr.ru/post/138020/
Аноним 14/04/17 Птн 15:49:47  973548
>>973389
>>973387
Если кому понадобится, решил без временных таблиц:

SELECT
d.date,
IF(o.count IS NULL, 0, o.count) AS count,
IF(o.salary IS NULL, 0, o.salary) AS salary
FROM
(SELECT
`date`
FROM
(SELECT ('2017-04-01') AS date UNION SELECT ('2017-04-02') AS date UNION SELECT ('2017-04-03') AS date UNION SELECT ('2017-04-04') AS date UNION SELECT ('2017-04-05') AS date UNION SELECT ('2017-04-06') AS date UNION SELECT ('2017-04-07') AS date UNION SELECT ('2017-04-08') AS date UNION SELECT ('2017-04-09') AS date UNION SELECT ('2017-04-10') AS date UNION SELECT ('2017-04-11') AS date UNION SELECT ('2017-04-12') AS date UNION SELECT ('2017-04-13') AS date UNION SELECT ('2017-04-14') AS date) AS date_tables) AS d
LEFT JOIN
(SELECT
COUNT(*) AS count,
SUM(total) AS salary,
DATE(date_added) AS `date`
FROM
`order`
GROUP BY DATE(date_added)) AS o ON o.date = d.date
Аноним 14/04/17 Птн 15:51:42  973549
>>973548
Точнее можно ещё избавиться от одного лишнего селекта в первом селекте
Аноним 14/04/17 Птн 16:25:49  973573
>>973479

Сама по себе цифра ничего не говорит. Если на эту машину поставить нгинкс и раздавать текстовый файл - он наверняка выдаст намного больше.
Аноним 14/04/17 Птн 16:36:16  973586
>>973573
Там приложение на фалконе, которое полгода уже разрабатывается. В структуре базы 40 таблиц, активное использование связей. Энжинкс само собой. Сейчас просто посматриваю на то, как нагрузку держит текущий вариант.
Аноним 14/04/17 Птн 17:54:31  973647
Вот допустим у меня есть метод, который может принимать на вход объекты двух типов, и делать с ними идентичные действия. Как быть с тайпхинтами? Делить метод на два для каждого класса, использовать func_get_args(), вписать условия в __call(), поставить в качестве тайпхинта род.класс либо дописать для каждого из этих классов интерфейс?
Аноним 14/04/17 Птн 18:10:49  973668
>>973647
Тайпхинтинг же для такого и нужен, разве не?
Аноним 14/04/17 Птн 18:22:32  973684
>>973647
Конечно же интерфейс, алло блять!
Аноним 14/04/17 Птн 18:55:57  973704
4jiYlNsEWZg.jpg (159Кб, 666x655)
Вопрос опу по файлшарингу.

Хранить ли в модели файла, а также в базе данных полный путь на скачку (http://fileshare.loc/download/id) и полный путь к папке downloads (C:\Apache24\htdocs\file-sharing\app\Controller/../../public/downloads/2017/04/14) или же обойтись короткими как у меня? /downloads/2017/14/04 папка и /download/id скачка соответственно.

Также неизбежно ли отказываться от конструктора класса-модели и создавать отдельный метод setProperties из-за того, что set_fetch_type(\PDO::fetch_class, 'ClassName') будет требовать третьим аргументом передачу свойств для конструктора, которые проблематично в моем FIleGateway получить?
Аноним 14/04/17 Птн 19:47:49  973746
>>973704
Это все не в модели хранится, а в конфиг файле/классе. Модели либо конфиг класс передается, либо готовые значения из конфиг класса (лучше). В конфиг классе может как относительный через _DIR_, так и абсолютный храниться, на то он и конфиг.
Аноним 14/04/17 Птн 19:51:23  973748
>>973684
А какой смысл этого? Метод принимает объект, ему надо понять, к какому классу он относится. При чем тут интерфейс сам по себе?
В методе условия ставить для вытягивания соответствующих свойств из объекта соответствующего класса и т.п.?
Аноним 14/04/17 Птн 20:13:15  973760
>>973748
You doing it wrong. Если метод может принимать два разных объекта и делать с ними разные вещи в зависимости от того, какой объект был ему скормлен, значит у тебя один метод, который выполняет роль двух методов, а это значит, что тебе надо разделить этот метод на два метода, каждый из которых принимает свой объект. Если у тебя метод может принимать больше одного класса объектов, но делает с ними одинаковые вещи, то тогда он должен принимать интерфейс. Либо так, либо говнокодить bad practice и тогда тебя не должны волновать type hinting и прочие ништяки нормального адекватного кода.
Если тебе нужно получать одинаковые данные, но с разных параметров объекта, то надо сделать два прокси метода, которые каждый выдергивает нужные параметры с твоих объектов и вызывает один основной метод, который уже делает всю логику вне зависимости от того, какие там объекты вначале были. Это позволит тебе затем, не переопределяя этот основной логический метод, добавить еще один прокси-метод, который будет принимать третий вид объекта и также скармливать нужные данные основному методу, которому по прежнему будет пофиг, от какого объекта эти данные были получены.
Аноним 14/04/17 Птн 20:15:44  973763
>>973760
Чисто для примера:

public function proxyMethod1(objectType1 $object)
{
$this->logicMethod($object->$c, $object->d);
}

public function proxyMethod2(objectType2 $object)
{
$this->logicMethod($object->e, $object->f);
}

public function logicMethod(int $a, int $b)
{
return $a + $b;
}
Аноним 14/04/17 Птн 20:24:40  973770
>>973760
>>973763
Я ничего не пони.
Но я был мимо, просто интерсно тоже, Зандстра что-то писал об этом, но у него вроде именно с проверкой класса у объекта.
Аноним 14/04/17 Птн 20:29:35  973773
>>973770
Я бы избегал кода, в котором метод может принимать два разных вида объектов в качестве одного аргумента. Либо делаешь два метода, каждый из которых принимает нужный объект, либо пилишь интерфейс с методом, который позволяет получить нужные данные, а твои объекты реализуют этот интерфейс и позволяют получить эти данные независимо от того какого типа сам объект.
Аноним 14/04/17 Птн 20:31:31  973774
>>973760
Короче чтобы долго не объяснять, вот пример кода:
https://github.com/MakeevD/MicroCrm/blob/master/app/Http/Controllers/OrderController.php#L121 - этот метод вызывает статический метод TempHandler'а (метод сохраняет изменения в базу), https://github.com/MakeevD/MicroCrm/blob/master/app/Helpers/TempHandler.php , у которого есть метод destroy (44 строка). Модель Temp имеет два внешних ключа, к двум моделям.
Короче я создал пустой интерфейс для обеих моделей, но уже убедился в том, что этот метод TempHandler'а вообще не нужен, как и сам TempHandler как класс, а его метод store переписать в конструктор модели Temp.
Аноним 14/04/17 Птн 21:59:41  973807
помогите решить эту сранную(интересную,просто уже заебавшую) с регулярными выражениями. Сижу как даун,видимо судьба, втыкаю 1+ час и не могу никак сделать. Вот то что смог понять,но никак все равно не получается:(:( https://regex101.com/r/tTXSls/1
http://archive-ipq-co.narod.ru/l1/regexp.html - тут естессна сама задача,там с массивом правильных и неправильных номеров.
Аноним 14/04/17 Птн 22:11:10  973809
>>973773
Сейчас посмотрел: Зандстра советует делать суперкласс и одинаковы методы в каждом дочернем, которые будут вытягивать, допустим, количество страниц или время прослушивания для дочерних классов CdProduct или BookProduct.
При этом у него в конструктор суперкласса передаются и время прослушивания, и количество страниц, то есть не одно вместо другого, но и то, и сё.
Ещё он говорит про instanceof, но мне такое не нравится тоже.
Аноним 14/04/17 Птн 22:12:04  973811
>>973704

Так у меня динамически создаются эти пути (год/месяц/день), как мне это хранить в конфиге-то.
Аноним 14/04/17 Птн 22:12:20  973812
>>973811
>>973746
Аноним 14/04/17 Птн 22:15:29  973813
>>973807
Что повторяется, то сокращай.
Я вообще не понимаю этого сайта с регулярками, сама тема такая, что редко пригождается (уже подо всё написаны регулярки, а сам начнёшь использовать - обязательно где-нибудь проколешься).
Аноним 14/04/17 Птн 22:17:54  973815
>>973813

так,если тебе конечно не сложно,можешь попробовать написать руглярку для этой задачки,чтобы я понял?
Аноним 14/04/17 Птн 22:26:20  973817
>>973813

забудь
проблема для детишек - не поставил в начале и конце регулярного выражения ^ $ как писалось в уроке...
Аноним 15/04/17 Суб 01:16:26  973866
Вопрос по поводу ПДО, допустим есть такая конструкция:

public function getUsers()
{
$sql = "SELECT * FROM users";
$result = $this->db->query($sql);
$users = $result->fetchAll(PDO::FETCH_OBJ);
return $users;
}

Как правильно проверить что запрос сработал, достаточно будет сделать просто вот так?

if($result = $this->db->query($sql)) {

//остальной код

}
Аноним 15/04/17 Суб 01:22:36  973867
Как сериализовать объект класса в XML текст? Есть какие нибудь готовые хорошие решения, или придется изобретать брать с stackoverflow свой велосипед?
Аноним 15/04/17 Суб 01:39:04  973871
И да, в дополнение к этому >>973866 когда происходит вставка каких-либо данных в подобной ситуации:

public function addQuestion(Question $question) {
if($stmt = $this->db->prepare("INSERT INTO question (id, question_text, user_ip, user_cookie) VALUES (NULL, ?, ?, ?)")) {
$stmt->execute(array($question->question_text, $question->user_ip, $question->user_cookie));
}
}

Что лучше вернуть из такой функции? Чтобы можно было как-то проверить успешность выполненного запроса уже вне класса?
Аноним 15/04/17 Суб 12:50:54  973960
>>973867
Зачем тебе XML, может тебе на самом деле Json нужен?
Аноним 15/04/17 Суб 13:13:24  973968
$stmt = $pdo->prepare('SELECT FROM students ORDER BY :test DESC');
$stmt->execute(array(':test' => $sort));
Аноны, почему это не работает? Если делать сортировку без prepare, то все отлично работает.
$stmt = $pdo->query('SELECT
FROM students ORDER BY test DESC'); - работает.
Аноним 15/04/17 Суб 14:23:59  973998
>>973968
>array(':test' => $sort)
Это еще что такое?
Аноним 15/04/17 Суб 14:27:41  974000
>>973998
Точнее в таком случае у тебя должно быть так:
$stmt = $pdo->prepare('SELECT FROM students ORDER BY :test DESC');
$stmt->bindValue(':test', $sort);
$stmt->execute();
Короче ты путаешься в синтаксисе. Почитай про подстановку значений в ПДО, у ОПа еще есть где-то.
Аноним 15/04/17 Суб 15:03:34  974013
Появилась задача выведения огромного списка (пусть, например, товаров) в select инпуте. При этом открывать весь список клиенту каждый раз необязательно, поэтому сразу его весь отдавать нецелесообразно. Вариант кэширование + ajax запрос при фокусе очевиден, но, во-первых, список может обновляться очень часто, а во-вторых, при этом всё равно присутствует какая-то задержка. Как мне сделать так, чтобы список "заполнялся" в реальном времени?
Т.е., допустим, по фокусу на инпут идёт один запрос на сервер, откуда в потоке приходят записи и колбэком заносятся в общий селект. Мне кажется, что это задача простая, но сходу в голову ничего не лезет.
Аноним 15/04/17 Суб 15:08:16  974016
>>974013
На клиенте запоминай сколько записей вернул ajax запрос, потом при необходимости догрузить еще записей, отправляй такой же запрос, только с параметром count, который будет использоваться бэкендом для выборки данных с оффсетом.
Аноним 15/04/17 Суб 15:08:33  974017
>>973867
Выбирай: https://packagist.org/search/?q=XML%20Serializer

>>973968
После подстановки твой запрос будет выглядеть примерно так: SELECT FROM students ORDER BY 'column' DESC
То есть вместо :test - строка, а нужен столбец (без кавычек).
Почему строка написано тут: https://secure.php.net/manual/en/pdostatement.execute.php

В качестве параметров PDO не используют столбцы или названия таблиц, так как параметры - это какие-то значения (строки, числа), пришедшие от пользователя. Для столбцов на голом PDO обычно пишут руками проверку: if (!in_array($column, ['age', 'name', ...])) { ...

>>973871
Если произошла ошибка, то PDO выбросит исключение. Это в старых MySQL расширениях для PHP нужно было постоянно проверки писать.

>>973998
Это синтаксис для массивов в PHP < 5.4

>>974000
В этом примере тоже будет ORDER BY 'строка' и сортировка не сработает. Строка потому что bindParam по дефолту всё приводит к PDO::PARAM_STR https://secure.php.net/manual/en/pdostatement.bindparam.php
Аноним 15/04/17 Суб 15:16:43  974022
>>974017
>В этом примере тоже будет ORDER BY 'строка' и сортировка не сработает. Строка потому что bindParam по дефолту всё приводит к PDO::PARAM_STR
Да, забыл. Можно эмуляцию отключить, ну или там просто отдельно пихать параметр сортировки.
Аноним 15/04/17 Суб 15:19:09  974024
>>974016
Ну это понятно, типа обычная пагинация по скроллу или кнопке. А в реальном времени? Типа, допустим, 10 000 записей, выводить по 100 => написать рекурсивную функцию, которая отправит 100 запросов? Просто хочется всё это в один запрос уместить.
Аноним 15/04/17 Суб 15:22:13  974025
>>974024
> А в реальном времени?
Что ты сделать-то хочешь? Тут два варианта, или пагинация или отправка всего списка сразу, сохранение его на клиенте и вывод его по кускам. Мне кажется делать несколько аякс запросов будет логичнее, если учитывать что список огромный.
Аноним 15/04/17 Суб 16:03:46  974046
Посоветуйте нормального обучалова (видео, книги и тп), где доходчиво рассказывается про symfony 2.7+. Вот прям чтоб от и до
Аноним 15/04/17 Суб 16:32:37  974063
Как в регулярке,при помощи preg_raplace, заменить любое число пробелов на один единственный?
Аноним 15/04/17 Суб 16:46:19  974070
>>974063
$string = 'April 15, 2003';
$pattern = '/(\w+) (\d+), (\d+)/i';
$replacement = '${1}1,$3';
echo preg_replace($pattern, $replacement, $string);
Что непонятного то?
Аноним 15/04/17 Суб 16:51:18  974071
>>974070
Окей, если ты такой молодец, можешь помочь еще кое с чем? Мне бы знать, как букву из нижнего в верхний регистр свичнуть
Аноним 15/04/17 Суб 16:52:23  974072
>>974070

И кстати, это бред какой-то и на мой первый вопрос ты не ответил
Аноним 15/04/17 Суб 16:53:21  974073
>>974071
Если горит сделать с регуляркой - напиши регулярку которая ищет нужную тебе букву в нижнем регистре и меняй ну букву в верхнем, делов то.
Аноним 15/04/17 Суб 16:58:02  974074
>>974072
$string = "qwe rty tyu ";
$string = preg_replace (‘/\s+/’, ‘ ‘, $string) ;
Такие вещи легко гуглятся, на самом деле.
Аноним 15/04/17 Суб 16:59:29  974075
>>974063
preg_replace('/[\s]{2,}/', ' ', $string);
Аноним 15/04/17 Суб 17:32:52  974089
>>974073
В этом то и проблема...
Не знаю как просто
Я сейчас напишу свой реплейс и ты скажешь,что не так:
preg_replace("/^(\s+)?([а-яё]/u"#но только проблема в том,что мне надо все предложение просмотреть,а я только первый символ смотрю(а как сделать я хз)#,'вот тут то я и не знаю,что написать',$'какой-то текст')
Аноним 15/04/17 Суб 17:39:41  974090
>>974089
Не совсем понял что ты хочешь, если ты хочешь чтобы искалось по всей фразе, то метасимвол ^ лишний в твоем выражении.

>("/^(\s+)?([а-яё]/u"

> ^ декларирует начало данных (или строки в многострочном режиме)
https://secure.php.net/manual/ru/regexp.reference.meta.php
Аноним 15/04/17 Суб 17:45:28  974093
>>974090
Мне нужно, заменить одну букву в начале предложения, поэтому я и поставил ^
Аноним 15/04/17 Суб 17:54:14  974095
>>974090
Если не сложно,помоги(кинь реплейс весь для замены первой буквы со строчной на заглавную):3
Аноним 15/04/17 Суб 17:55:34  974096
4jiYlNsEWZg.jpg (37Кб, 468x306)
Вот так можно переносить аргументы и параметры, если они длинные? #stylewars
Аноним 15/04/17 Суб 18:05:37  974098
>>974096
(){
arg1,
arg2,
arg3,
arg9999
}
А вообще сокращать надо. Тут достаточно и $thumbWidth например.
Аноним 15/04/17 Суб 18:14:12  974106
>>974096

На вопрос отвечают рекомендации по оформлению кода PSR:

http://www.php-fig.org/psr/psr-2/#method-arguments

PSR это не совсем стандарт, это рекомендации, которым следуют разработчики нескольких популярных фреймворков, но лучше у нас все равно ничего нет.

IDE умеют форматировать код в соответствии со стандартом, если настроить нужные опции: https://ratanparai.wordpress.com/2014/10/16/prepare-eclipse-for-php-psr-standard/
Аноним 15/04/17 Суб 18:26:20  974116
>>974106
Раз уж тут появился вопрос про оформление кода, то спрошу -
как относятся к такому оформлению:

// \t - табуляция
$entity = new Entity();
\t$entity->setFoo();
\t$entity->setBar();
\t$entity->set...;
Аноним 15/04/17 Суб 19:50:04  974146
Бля, где можно найти больше работы удаленно?
Нашел сначала в группе вк одну, работаем понемножку, с оплатой по задачам, но работы очень мало сейчас.
Нашел вторую, там же, писать модули под одну систему. 8 тысяч рублей за один модуль, поговорили по скайпу, оплатили уже первые 2 задачи по текущему модули, все четко, все получается почти всегда
Но хочу немного больше нагрузки, а в офис лень идти. Нравится удаленная работа, вкатился недавно, уже есть что показать.
Может фриланс?Или есть еще где то места, где публикуются обьявления по рабте
Аноним 15/04/17 Суб 19:59:19  974160
>>974046
Да почитай документацию на сайте. Я освоился за пару часов более менее
Аноним 15/04/17 Суб 21:02:59  974194
>>974116

Так не делают. Отступ добавляется внутри блоков вроде if, for.

>>974146

fl.ru не смотрел что ли? freelansim, geekjob, hh.ru, мой круг.
Аноним 15/04/17 Суб 23:18:48  974246
>>974093
А ucfirst не пробовал использовать?
http://php.net/manual/ru/function.ucfirst.php
Аноним 15/04/17 Суб 23:21:40  974248
>>974146
freelancer, guru, upwork, toptal, crossover
Там все деньги, в твоих контактах гроши по сравнению с тем, что там. Но надо инглиш сначала подучить.
Аноним 15/04/17 Суб 23:31:54  974253
2017-04-1523-26[...].png (86Кб, 1486x836)
>>974248
>freelancer
>>974248
>freelancer

Ты наверное английскую версию имел, а то зашел на русскую и сразу проиграл в голос
Аноним 15/04/17 Суб 23:33:23  974254
>>974248
О, спасибо, то что нужно, английский не особая проблема. Разве что если общаться именно голосом, там могут проблемы возникать. А так писать на нем могу свободно документацию, статьи, книги и т.д. Этого достаточно?
Аноним 15/04/17 Суб 23:35:38  974257
>>974254
читать свободно*
Аноним 16/04/17 Вск 02:20:28  974283
>>974253
Мы с пацанами в 2008 сидели в кроватке на уроке информатики...
Аноним 16/04/17 Вск 08:45:33  974305
>>973439
Падажжи ебана.
Можно подробнее, пожалуйста. Вообще не могу въехать в задачу. То есть знаю функцию, но не понимаю как это все правильно записать, что бы работало.
Аноним 16/04/17 Вск 11:17:50  974353
>>971996

У тебя та же проблема что и у других начинающих: ты пишешь код стеной и не можешь вынести отдельные действия в функции. Ну например конвертирование кодировки в пути надо делать отдельной функцией, а не копипастить многократно по коду. Да и саму отдачу файла наверно тоже.

> $path = '..\\' . $file->link;
Не стоит использовать такие относительные пути, так как они высчитываются относительно текущей директории процесса (не директории в которой php-файл) и могут указывать куда угодно.

> 'attachment; filename=' . $fileName)
Это не будет работать так как в filename по стандарту можно указывать только ASCII (латинницу). Есть способ указать там кодировку, но он работает только в новых браузерах, так что лучше это поле не использовать.

> readfile($path);
> return($newResponse);
Это тоже неправильно. Ты сначала выводишь содержмое файла через readfile, а только потом возвращаешь объект ответа с заголовками, но без тела ответа. Зачем тогда этот объект Response нужен если он не содержит тело ответа?

Конечно Слим перехватывает выводимые данные и добавляет их в ответ, но это неэффективно, так как весь файл будет считан в память. Нужно использовать возможность передать поток файла в Response, и он будет выводе ответа прочтет тело из него.

Если посмотреть на конструктор Response, то там есть возможность передать в конструктор объект, соответствующий StreamInterface.

Ну и наконец, все это подразумевает отдачу файла средствами PHP. Эффективнее было бы переложить эту задачу на веб-сервер, если есть такая возможность. Она есть в Апаче с помощью модуля X-Sendfile и в нгинксе в модуле X-Accel-Redirect. От PHP скрипта только требуется указать имя файла для отдачи. Стоит добавить возможность использовать такой модуль.

>>972123

Имей в виду еще, что на это все может уйти много времени.

>>972289

Тебе надо разобраться с областью видимости переменных. Напрмер, глобальные переменные не видны внутри функций.
Аноним 16/04/17 Вск 11:18:21  974354
>>972305

Синглтон это плохой паттерн в данном случае.

>>972417

> Но если мы хотим получить всю цепочку начиная с B, нужно будет сделать ещё один запрос WHERE reference=B, и так далее, пока мы не найдем всех предков.

Может быть я не так понял условия задачи? Вот в этом примере, когда мы создаем большую таблицу связей:

post | reference | depth
A | B | 1
A | C | 1
A | D | 2
A | E | 2
B | D | 1
B | E | 1

Мы ищем посты не по условию WHERE reference =, а по условию WHERE post = и выборка WHERE pos t = A включает в себя и посты из выборки с post = B (D и E).

То есть если представить посты и связи как дерево, то в этой таблице мы одним запросом выбираем всех потомков узла любой глубины вложенности, а не только непосредственных детей.

>>972692

Проблемы будут не только с регулярками, например mb_lenth() вернет длину в codepoints.

>>972694

В HTML используют отступ в 2 или 4 пробела. Имена тегов пишут в нижнем регистре.

В CSS пишут свойства либо вертикально, либо горизонтально. Имена классов и id пишут маленькими буквами, разделяя слова минусами.

Есть еще БЭМ.

Погуглить стайлгайды можно по словам вроде "html style guide". Вот я например гугловский выгуглил: https://google.github.io/styleguide/htmlcssguide.html

>>972738

Погуглить стайлгайды можно по словам вроде "html style guide". Вот я например гугловский выгуглил: https://google.github.io/styleguide/htmlcssguide.html
Аноним 16/04/17 Вск 11:18:43  974355
>>972808

А что-то запрещает тебе его использовать? Он к конкретному языку не привязан.

>>972810

> юникод-класс который обещает быть не быстрым
А у тебя есть более быстрая альтернатива? Также, это надо мерять, может на практике скорости вполе хватает.

> для включения нижних подчёркиваний нужно использовать старый добрый \w (с указанием 'u' после регекса).
Можно писать так [\p{L}_]

> А вот дефисы \w не видит, чтобы проверять слова,интересно, почему так сделали
Не знаю. Это формально не дефис, а математический знак минус.
Аноним 16/04/17 Вск 11:47:09  974373
121.png (33Кб, 792x682)
>>973395
Объясните плиз, подробно, как решать ? Чето совсем додуматься не могу, как это все правильно записать.
Аноним 16/04/17 Вск 12:02:55  974383
Не нашел отдельного треда про CSS/HTML, напишу сюда, ОП вроде добрый.

Сделал мобильную верстку для сайта на медиа

@media only screen and (min-width: 400px) and (max-width: 500px),
only screen and (min-device-width: 400px) and (max-device-width: 500px){

в браузере в указанном разрешении все норм, но на реальном телефоне отвратительная хрень - позиции выставляются по минимальному значению (< 400px), а вот размеры шрифта, элементов - все дико мелкое. В итоге как будто телефон выбирает css для 400px, а отрисовывает на свои реальные пиксели. Что не так?
CSS Аноним 16/04/17 Вск 13:20:53  974427
>>971913
https://github.com/codedokode/pasta/blob/master/html/html.md

8: https://jsfiddle.net/Lpguk5kx/

10: https://jsfiddle.net/18504y5y/3/

11:
https://jsfiddle.net/3qm145b4/2/
https://jsfiddle.net/3qm145b4/2/embedded/result/
Второе - тест клавиатурной навигации. На jsfiddle немного глючит из-за наличия посторонних элементов на странице. Но если сохранить эту https://pastebin.com/YDuMFQSP страницу и открыть браузером, то навигация работает правильно (стрелочки+Tab для радиокнопок, стрелочки+пробел для чекбоксов).

12:
https://jsfiddle.net/qozy8zsL/7/
https://jsfiddle.net/qozy8zsL/7/embedded/result/
Аноним 16/04/17 Вск 13:36:48  974431
>>974353
>У тебя та же проблема что и у других начинающих: ты пишешь код стеной и не можешь вынести отдельные действия в функции. Ну например конвертирование кодировки в пути надо делать отдельной функцией, а не копипастить многократно по коду. Да и саму отдачу файла наверно тоже.
В целом понял. Я в общем-то так и думал, сделать класс типа FileDownloader и реализвать в нем разные способы отдачи файлов.
>Не стоит использовать такие относительные пути, так как они высчитываются относительно текущей директории процесса (не директории в которой php-файл) и могут указывать куда угодно.
А нормально ли будет сделать пхп-файл в котором константами будут прописаны все эти пути? Или надо как-то хитрее делать?
Аноним 16/04/17 Вск 14:12:33  974437
>>974383

meta viewport поставил? Без него телефон будет рендерить страницу в вьюпорт шириной 800-900 пикс и всячески имитировать десктопный браузер. При этом твое правило min-device-width срабатывает.

https://developer.mozilla.org/en-US/docs/Mozilla/Mobile/Viewport_meta_tag
http://frontender.com.ua/mobile-web/wtf-viewport/

Я бы также убрал min-device-width, какое оно отношение имеет к верстке? Размер окна браузера не обязан совпадать с размером экрана устройства.

Аноним 16/04/17 Вск 14:52:22  974448
Продолжая выше тему про указывание абсолютных путей. Вот есть у меня в приложении папка public, в которой все файлы и исполняются. Выше от нее лежит папка с загруженными файлами. Как правильно всегда получать абсолютный путь к этой папке? Я пока добился такого только с помощью такой вот стремной конструкции dirname(getcwd()).
Аноним 16/04/17 Вск 14:57:02  974449
Поясните пожалуйста, почему див с одной строкой текста в данном случае сместился чуть вниз? Если дополнить текст до двух строк, то смещение становится нормальным, как и у первого.
https://jsfiddle.net/rw01ae7v/
Аноним 16/04/17 Вск 16:33:30  974461
>>974449
vertical-align: top, либо пропиши float.
Vertical-align у тебя по умолчанию baseline, а baseline в данном случае - конец строки первого дива
Аноним 16/04/17 Вск 16:42:37  974463
На винде вообще работает кто-то с PHP? Или во всех конторах на компах только линукс установлен?
Аноним 16/04/17 Вск 16:54:06  974471
Кто-нибудь пробовал макачить на YouDo? Там на первый взгляд задания все простые, можно с лёгкостью за часик заработать на пивас.
Аноним 16/04/17 Вск 18:34:27  974508
>>974354
>> Но если мы хотим получить всю цепочку начиная с B, нужно будет сделать ещё один запрос WHERE reference=B, и так далее, пока мы не найдем всех предков.
>
>Может быть я не так понял условия задачи? Вот в этом примере, когда мы создаем большую таблицу связей:
>
>post | reference | depth
>A | B | 1
>A | C | 1
>A | D | 2
>A | E | 2
>B | D | 1
>B | E | 1
>
>Мы ищем посты не по условию WHERE reference =, а по условию WHERE post = и выборка WHERE pos t = A включает в себя и посты из выборки с post = B (D и E).
>
>То есть если представить посты и связи как дерево, то в этой таблице мы одним запросом выбираем всех потомков узла любой глубины вложенности, а не только непосредственных детей.
>Может быть я не так понял условия задачи?
Проясню на всякий случай условия задачи:

Если мы имеем пост №1, на который ссылается №2, и на номер №2 ссылается №3, то какой бы пост мы не выбрали мы должны получить всю эту цепочку.


Я с самого начала был запутан этой таблицей, я думал что A это самый старший предок, который имеет ответы B, C и т.д, но, тем не менее, это не меняет принципа моего вопроса - Если B имеет ссылки на предков, то он не имеет ссылок на детей и получить всю цепочку только одной выборкой WHERE post=B нельзя, нужно делать


Аноним 16/04/17 Вск 18:45:51  974517
Кто нибудь делал форму фильтрации контента типа тех что в магазинах? Проблема начинается когда сразу отправляются опции которые могут вычитать и добавлять результати(и или). Я пробовал передавать в get параметры и в контроллере на if'ах фильтровать но это говнокод полный. Может кто на ларавел делал и подскажет буду признателен.
Аноним 16/04/17 Вск 18:57:13  974518
>>974508

А, понял, о чем речь. Может быть можно тогда допилить таблицу и добавить туда не только "потомков", но и "предков" поста, с depth = -1, -2 и тд?

Тогда будет один запрос. Но на мой взгляд, это только утяжелит таблицу, и приведет к дублированию данных без особой выгоды, действительно, лучше уж тогда 2 запроса делать:

WHERE post = A
UNION
WHERE reference = A

Вообще, я сейчас поймал себя на мысли, что связи между постами образуют не дерево (где у узла ровно один предок, и много потомков), а граф без циклов (где узел может ссылаться на любое число узлов и любое число узлов может ссылаться на него, но циклов нет так как пост может ссылаться только на предыдущие посты): https://ru.wikipedia.org/wiki/%D0%9D%D0%B0%D0%BF%D1%80%D0%B0%D0%B2%D0%BB%D0%B5%D0%BD%D0%BD%D1%8B%D0%B9_%D0%B0%D1%86%D0%B8%D0%BA%D0%BB%D0%B8%D1%87%D0%B5%D1%81%D0%BA%D0%B8%D0%B9_%D0%B3%D1%80%D0%B0%D1%84

И соответственно мы можем при желании построить таблицу связей в обе стороны, искать как "потомков", так и "предков". Ну и вообще, поняв, какая у нас структура данных, можно понять, что с ней можно делать.

Рекурсия тут нужна в любом случае, просто мы можем делать ее либо при построении таблицы, строя список всех связей, либо ее придется делать при выборке.

> Я с самого начала был запутан этой таблицей,
Тут лучше посмотреть на картинку графа из википедии. Мы можем либо записывать в таблице непосредственные связи (узлы, в которые за 1 шаг можно попасть из текущего), либо записывать все узлы, которые достижимы из текущего, если двигаться по стрелкам.

Аноним 16/04/17 Вск 19:12:47  974522
>>974517
>Проблема начинается когда сразу отправляются опции которые могут вычитать и добавлять результати(и или).
Приведи пример. Типа (A ^ B) V (C ^ D) V (E)? И как на фронте это выглядит?
Аноним 16/04/17 Вск 20:11:33  974551
>>974471
Что за анальная регистрация
Аноним 16/04/17 Вск 21:28:27  974585
>>974522
Ну пример на телефонах: есть чекбоксы hdmi, ips(условие и), ну и чекбоксы моделей с условием или, но не в этом суть. Как вообще правильно организовать передачу параметров и разбирать их в контроллере? Плюс если там будут связующие таблицы с айдишками фичей телефонов то код вообще раздутый выходит. В общем может есть готовые решения, паттерны организации фильтрации контента?
Аноним 16/04/17 Вск 21:36:33  974588
Задачка про дурака с регулярками - https://ideone.com/dbV0vr
Аноним 16/04/17 Вск 22:27:40  974607
>>974585
Ну я свой код уже скидывал выше в треде, вот класс-хелпер для фильтрации:
https://github.com/MakeevD/MicroCrm/blob/master/app/Helpers/SearchOrders.php
и форма:
https://github.com/MakeevD/MicroCrm/blob/master/resources/views/orders/search.blade.php
Контроллер передаёт в этот класс массив search[] с формы, и там всё постепенно строится Builder. В классе-хелпере лежит массив с правилами, по какому методу Builder'а искать тот или иной параметр (например, все даты объединены правилом date). Нужен какой-то другой - дописываешь правило и метод.
Это внешне похоже на то, как это реализовано в Yii2. Но я хуй простой, и интересно было бы послушать мнение других.
Аноним 16/04/17 Вск 22:35:17  974610
>>974607
попробуй почитать документацию, можешь юзать скоупы для моделей и чтоб вернуть json можешь с метода просто массив вернуть.

PS. 6yrs php experience
Аноним 16/04/17 Вск 22:46:08  974612
>>974607
спасибо тебе, посмотрю
Аноним 16/04/17 Вск 23:02:29  974620
>>974610
Ну этот проект в процессе буквально пару недель, впереди ещё много доделывать и рефакторить. Ну да, со скоупами более ларавель-вей, но вкратце суть такая:
1. Определяем правила, по которым ищутся параметры. (после того, как напишу валидацию, думаю, привяжу эти правила и к поиску)
2. Перебираем массив параметров с фронтенда, к каждому одноименному полю вызываем соответствующий правилу метод (скоуп)
3. Возвращаем билдер, и в контроллере уже его сортируем-пагинируем-сериализуем

Мой todo: передавать массив ['логический_оператор' => 'массив_параметров']. Каждый массив_параметров фильтруется вышеописанным способом, а затем они связываются между собой логическими параметрами (в качестве ключа) в том порядке, в котором его принял хелпер. Ну и этот массив может быть многоуровневым.
Аноним 16/04/17 Вск 23:09:13  974624
Начал изучать документацию по Doctrine и такой вопрос: можно ли получить объект продукт который хранится в нескольких таблицах? И вторая часть вопроса - можно ли потом будет сохранить его методом flush() назад в те же таблицы при изменении?
Аноним 16/04/17 Вск 23:29:39  974634
>>974624
relations
Аноним 16/04/17 Вск 23:36:53  974639
А XML сейчас актуален вообще? Ну за исключением rss конечно
Аноним 17/04/17 Пнд 00:49:57  974682
>>974254
На апворке да, на топталах/кроссоверах нет, там устный проверяют.
Аноним 17/04/17 Пнд 00:52:10  974683
>>974639
Все soap веб-сервисы на нем. Rest тоже часто бывают.
Аноним 17/04/17 Пнд 06:06:48  974741
в избранное
Аноним 17/04/17 Пнд 14:04:44  974915
Есть тут кто? Есть пара вопросов.
Аноним 17/04/17 Пнд 14:21:24  974938
>>974915
м?
Аноним 17/04/17 Пнд 14:23:21  974941
>>974938
Давно писал свою простенькую тёмную тему для двача через stylish т.к. нормальной не нашёл, а тут решил чего-то модернизировать, и пришла куча идей, но я не специалист в css, поэтому не уверен - можно ли такое вообще сделать. Это к вашему треду?
Аноним 17/04/17 Пнд 14:37:37  974958
>>974941
Это скорее в Верстка-тред, тут я думаю верстальщиков поменьше. Хотя большинство знает CSS на средне-начальном уровне
Аноним 17/04/17 Пнд 14:39:31  974962
>>974958
Я бы мог составить список вопросов и тут написать их, а то 1 я в этом точно не разберусь.
Аноним 17/04/17 Пнд 15:17:43  974985
>>974962
ну составь, на какие сможем ответим
Аноним 17/04/17 Пнд 15:27:34  974992
Untitled.png (5Кб, 833x102)
>>974985
Все вопросы к стайлишу:
1. Можно ли добавить в строку на 1 пике кнопку "Доска", которая бы вела на главную доски треда. Например, если я тут бы её нажал, то перешёл бы на доску /pr ?
2. Можно ли сделать так, что при автообновлении страницы она сама прокручивалась вниз?
Аноним 17/04/17 Пнд 15:51:59  975025
>>974992
Ничего из списка Stylish'ем не сделать, тебе нужен язык JavaScript и аддон, позволяющий вставить свой JavaScript код в страницу (Greasemonkey, Tampermonkey). Для первой задачи нужно вычленить название доски и вставить элемент-ссылку в ДОМ-дерево, для второй задачи нужно изучать JavaScript макабы, чтобы узнать как добавить прокрутку после автообновления. Если не знаком с JS, то будет туго. Вся инфа есть на learn.javascript.ru
Аноним 17/04/17 Пнд 16:08:50  975040
>>975025
А вы не поможете скрипты написать? Я в цсс худо-бедно с гуглом написал говнотему, а с яс я точно не справлюсь.
Аноним 17/04/17 Пнд 18:14:53  975117
>>972377
>> Я реализовал ассоциацию Один-Ко-Многим и теперь, когда я получаю Тред, я получаю его Посты и Файлы, и теперь подсчет количества постов выполняется посредством метода count() класса ArrayCollection.
>
>Ну так это сверхнеэффективно, выбрать кучу постов в память только для того, чтобы получить их количество или получить первые 3.
>
>> Мне следует получать посты через свои запросы?
>Да.
Ха, даже если я использую NativeSQL запросы, то я всё равно получаю ArrayCollection на самом деле PersistentCollection, не знаю почему со всеми постами. У меня не получилось нагуглить как получить сущность без ассоциаций. Значит нужно отказаться от них?

>>974518
>Тогда будет один запрос. Но на мой взгляд, это только утяжелит таблицу, и приведет к дублированию данных без особой выгоды
Это действительно так плохо? Это решение решило бы проблему.

Аноним 17/04/17 Пнд 18:52:13  975135
>>974517
А почему бы тебе не сортировать в реальном времени прямо на странице, а для получения подробной информации отправлять например аяксом только id уже отфильтрованных записей?
Аноним 17/04/17 Пнд 18:54:31  975137
Аноны, что означает подобная запись?
public function get(): StringReverseWorker
{
код
}
Интересует именно двоеточие
Аноним 17/04/17 Пнд 19:01:44  975144
>>975137
>: StringReverseWorker
Возвращаемый тип.
Гугли: тайпхинтинг в пхп.
Аноним 17/04/17 Пнд 19:09:38  975149
>>975144
Да, оно, спасибо. Впервые вообще такое вижу. Откуда ты об этом узнал?
Аноним 17/04/17 Пнд 19:19:58  975158
>>975149
Это в седьмой пыхе завезли. Strict Mode, все дела
Аноним 17/04/17 Пнд 19:20:14  975159
>>975149
Это супер новая фитча пхп7.
Аноним 17/04/17 Пнд 19:22:06  975160
>>975117
Почему бы тебе не использовать getSingleScalarResult();???

Пример:
public function getTotal()
{
return $this->createQueryBuilder('post')
->select('COUNT(post)')
->getQuery()
->getSingleScalarResult();
}
Аноним 17/04/17 Пнд 19:35:19  975177
>>975160

Зачем вы тут используете Query Builder? Пишите на DQL сразу, будет и проще и читабельнее. Query Builder для сборки запросов из частей, а тут запрос всегда одинаковый.

Алсо тут проще обычный SQL запрос использовать без маппингов, через DBAL.
Аноним 17/04/17 Пнд 19:37:12  975178
>>975160
Да, это скорее всего работает. Сейчас проверю.
Аноним 17/04/17 Пнд 19:43:01  975183
>>975177
> Пишите на DQL сразу, будет и проще и читабельнее.
>Алсо тут проще обычный SQL запрос использовать без маппингов, через DBAL.
Иди своей дорогой, сталкер.
Аноним 17/04/17 Пнд 20:02:35  975191
14920989386350.jpg (45Кб, 538x348)
подскажите, пожалуста. попробовал решить задачку с калькулятором. вот код http://ideone.com/6hUFWJ
1. на идеуан выдает ошибку
Uncaught Error: Call to undefined function mb_strlen() in /home/8DynTa/prog.php:14
2. через терминал все работает на ноуте, но только конкатенирует строки, а не складывает их, не могу понять где ошибка
3. пошпыняйте за сам код
Аноним 17/04/17 Пнд 20:04:41  975192
Screenshot from[...].png (70Кб, 1368x818)
>>975160
>>975178
Да, getArrayResult() сработал. Спасибо.
Аноним 17/04/17 Пнд 20:13:07  975196
>>975191
на ideone может не быть расширения mbstring
Аноним 17/04/17 Пнд 21:55:29  975278
Стоит ли вкатываться в Ларавел без знания php? Саму суть я понимаю, писал на Python Django. Синтаксис выучить это 1-2 дня, но боюсь что иногда буду заходить в тупики и придётся читать документацию с самого начала.
Аноним 17/04/17 Пнд 23:16:27  975333
Как и на чём реализовать следующее:
1. Большая картинка, допустим, нарисована коробка с инструментами, в которой лежат молоток, отвёртка, зубило, рубанок, пила и тд.
2. Инструменты при наведении курсора слегка увеличиваются и являются ссылками на соответствующие страницы (о молотках, отвёртках и тд).
3. В зависимости от наличия инструментов, там может быть рядом лежащих три молотка, три отвёртки (рядом, но со смещением).
4. Какая-то форма в админке для заполнения количества: допустим, ставлю три галочки в чекбоксах - появляются на картинке три молотка, а всего может быть пять молотков.
Вообще там нужно было полку с ручными работами, они готовятся медленно, но вот с молотками понятнее, надеюсь.
Сама визитка на Yii2.
Человеку в админке нужно просто как-то указывать количество молотков-отвёрток, жмакать на "сохранить", а потом на странице видеть изменения.
Как к такому подступиться вообще? На jQuery, на чём?
Аноним 17/04/17 Пнд 23:28:40  975340
>>975196
понятно, тоже так подумал, жаль вот ошибку я еще не нашел
Аноним 18/04/17 Втр 00:19:29  975353
>>975333
css + jquery же, можно бутстрап для упрощения жизни, но не обязательно.

>Как к такому подступиться вообще?
Уроки верстки из шапки делал? Тогда бы знал уже. Бэкграунды, флоаты, абсолютное позиционирование, css эффекты.
Аноним 18/04/17 Втр 10:27:30  975437
>>975353
Я делал очень давно и не до конца доделал тот макет.
Что CSS - это понятно, бутстрап там уже есть. Что jQuery - я предполагал.
Ну а как подступиться-то? Я хз же вообще.
Нужна ведь вкладка в админке для выставления флажков в чекбоксах. Или лучше не чекбоксы, а ползунки jQuery UI, наверное.
Не могу всё собрать воедино, так как никогда не имел с этим дела.
Аноним 18/04/17 Втр 13:13:13  975515
14922918956450.webm (3630Кб, 320x240, 00:02:20)
Где искать заказы? Как должно примерно выглядеть портфолио, чтобы пользоваться спросом хоть каким-то?
Аноним 18/04/17 Втр 13:21:41  975517
>>975437
Позиционируй сначала все при помощи css и эффекты при наведении сделай, а потом уж jquery эвенты вешай и по чекбоксам распределяй.
Аноним 18/04/17 Втр 13:22:31  975518
>>975515
апворк, топтал
тесты заполненные + отзывы клиентов
Аноним 18/04/17 Втр 13:41:03  975525
>>975518
Тесты -- примеры работ?
Аноним 18/04/17 Втр 14:02:06  975537
14551235823923.jpg (118Кб, 1024x768)
Аноним 18/04/17 Втр 14:56:09  975562
>>975278
Ну доки ларавела точно надо почитать. К докам пхп по необходимости будешь обращаться, ничего сложного. Разве что ларавел, конечно, распиаренное говно . Объективно он сейчас самый хуевый для разработки вариант, но уж больно удобный, мимими.
Аноним 18/04/17 Втр 15:45:28  975586
>>975562
Почему самый хуевый? Вроде как в сравнении с yii говорят, там больше наворотов и функицонала.
Аноним 18/04/17 Втр 15:48:52  975591
>>975586
Перделок на уровне кода ага, больше. Полезного функционала меньше. Скорость такая, что кина не будет, посаны. Медленнее ларавела только вордпресс.
Аноним 18/04/17 Втр 16:16:39  975617
Можете помочь по CSS? А то я что-то не понимаю.
Есть такое
<div class = words><div class = italic>n: </div>{{noun}}</div>
Фишка в том, что если {{noun}} пустой, то весь класс words не должен отображаться, если же в {{noun}} будет слово или буква, то должно показаться, типа:
n: test

Аноним 18/04/17 Втр 17:27:04  975655
>>975617
{% if noun is not null %}
{% endif %}
Аноним 18/04/17 Втр 17:32:18  975662
>>975655
Это куда писать?
Аноним 18/04/17 Втр 17:32:55  975663
>>975662
А тебе куда надо?
Аноним 18/04/17 Втр 17:37:58  975666
Untitled.png (26Кб, 959x1157)
>>975663
Делаю свои карты в программе Анки, и хочу так сделать, что если я не напишу ничего в поле, то оно не отображалось бы - пик.
Аноним 18/04/17 Втр 17:40:26  975668
Решил давеча в файлообменнике для коментов запилить эктивРекорд, а то как-то глупо на каждый чих по сущности делать. Ну так вот, пилю я короче такой абстрактный класс ActiveRecord, в нем хочу сделат метод insert, ну вы понели. Проблема появилась с подготовленными запросами. Вот с классом-шлюзом все понятно, там просто хуячишь запроса вида
"INSERT INTO table(name, secondName, nuTiPonel) VALUES (?, ?, ?);
Ну а далее prepare($sql) и экзекут, ну вы понели. Ну или там можно bindParam(':name', $name). Ну тут проблема в том, что я не знаю заранее сколько и каких у меня будет параметров в класс-таблице для подстановки. Как лучше решить проблемку? А то на ум сплошной говнокод приходит, типа хранить все эти данные в специальной параметре-массиве и прочее.
Аноним 18/04/17 Втр 17:53:05  975681
>>975666
Лол.
Тогда никак, если там нельзя java script.
Аноним 18/04/17 Втр 18:11:41  975700
>>975537
Кот на месте, я спокоен.
Аноним 18/04/17 Втр 18:47:33  975719
14816231660091.webm (436Кб, 460x460, 00:00:06)
Господа, помогите, пожалуйста, с небольшой задачей.
Есть основной сервер с обновляющимися данными, я с него периодически отправляю запросы с этими данными на другой сервер-клиент. И на сервер-клиенте должна обновляться страница.
Так вот, как сделать, чтобы из текста пост-запроса генерировалась страница с этим текстом? (Подозреваю, что это элементарно)
Аноним 18/04/17 Втр 18:58:48  975725
Какой уровень прошаренности должен быть для вкатывания во фриланс?
Аноним 18/04/17 Втр 19:00:29  975727
Вот я еблан, не могу понять как эту задачу решить. Нашел вот этот вариант.

<?php

$bankSum = 10000;
$bankPercent = 1.1;

for ($age=16; $age < 1000; $age++) {
$time = $age - 16;
$bankSum = $bankSum * $bankPercent;
if ($bankSum >= 1000000) {
break;}
}

echo "через $time лет сумма на счёту в банке составит $bankSum, этому некто будет $age";

Помогите понять, почему мне кажется, что это неправильно? И как округлить число там полученное? Через Round не выходит.
Аноним 18/04/17 Втр 19:18:03  975758
Сап, пыханы! У меня всего один вопрос. Вот скажите мне почему вас не любят другие программисты? В частности, откуда пошло выражение "пхп-макака"?
Аноним 18/04/17 Втр 19:20:03  975760
>>975758
Низкий порог вхожения -> много вкатывальщиков -> много говнокода
Аноним 18/04/17 Втр 19:21:52  975763
>>975760
В общем-то, я так и думал. Спасибо, выкатываюсь.
Аноним 18/04/17 Втр 19:29:45  975768
>>975727

round($bankSum)

Попробуй переменные выносить за кавычки. То-есть:

echo "через " . $time . " лет сумма на счёту в банке составит " . round($bankSum) . ", этому некто будет " . $age;
Аноним 18/04/17 Втр 19:37:36  975788
>>975768
О, спасибо, получилось. Не туда round пихал.
Аноним 18/04/17 Втр 19:47:43  975806
>>975768
А есть какая-нибудь практическая польза от того, что я выношу переменные за скобки? Пока я понял только, что это для того, чтобы формулы не наёбывались.
Аноним 18/04/17 Втр 19:48:13  975808
>>975806
За ковывки.
Быстрофикс
Аноним 18/04/17 Втр 19:48:44  975809
>>975808
Сука, ЗА КАВЫЧКИ.
Аноним 18/04/17 Втр 20:05:18  975832
>>975806
Вопрос отметается, я понял.
Аноним 18/04/17 Втр 20:06:23  975835
>>975806

Да, так такие функции как round() будут работать. Пока они в кавычках - считаются текстом и просто выводятся на экран.
Тебе самому потом проще будет, некоторые текстовые редакторы не хотят делать подсветку переменных внутри кавычек.

Я бы на твоём месте ещё поменял этот участок кода:

> for ($age=16; $age < 1000; $age++)

Внутри цикла у тебя используется if как "СИГНАЛ СТОП", но проще будет изменить само условие цикла, тогда не потребуется делать дополнительных проверок.

for ($bankSum = 10000, $age = 16, $time = 0, $bankPercent = 1.1;
$bankSum <= 1000000; $bankSum *= $bankPercent)
{
$time++;
$age++;
}

То-есть все переменные обозначили в условии, затем цикл повторяется столько раз, пока сумма не будет равна миллиону
Аноним 18/04/17 Втр 20:12:41  975841
>>975835
А, ну я просто перед этим прочитал в уроке, что, мол, код становится запутанным, если несколько переменных вставлять в цикл и поэтому так танцевал. Так что спасибо за информацию.
Аноним 18/04/17 Втр 20:12:58  975842
>>975763
Только обобщать не надо на всех сразу. Я и джавистов/шарпистов видел таких, которые любую вордпресную пхпмакаку заставят себя почувствовать богом программирования, и никакой порог вхождения повыше им не помог.
Аноним 18/04/17 Втр 23:35:39  976042
Аноны, у меня довольно идиотский вопрос. Когда создается объект, который наследуется от другого класса, то создаются два объекта? Или как вообще это работает?
Аноним 18/04/17 Втр 23:54:46  976054
Прошу помощи:

Работаю с Йи2 на винде и локальном сервере Апачи. Возникла необходимость из скрипта (модели) создавать папку и в ней же создавать файл. Вот только файл не создается (создаю с помощью мкдир) и происходит фатальная ошибка. Что интересно, я из модели могу создать и папку и файл везде, кроме папки локального сервера. То есть в корневой директории Йи2 - уже не могу. За пределами - пожалуйста. Помогите решить.
Аноним 19/04/17 Срд 00:18:06  976064
>>976042
Нет, один объект создается. Просто методы и свойства наследуются от какого-либо класса.
Аноним 19/04/17 Срд 00:36:44  976070
>>976054
Уточню: в ходе проверок выяснилось, что могу создать подпапку из модели бекэнда (я использую адвансед шаблон) в любом каталоге, кроме каталога фронтэнд и, соответственно, его подкаталогов. В чём проблема?
Аноним 19/04/17 Срд 00:37:16  976071
>>976064
Смотри:
<?php

class Base
{
private $test = 0;

function setPrivate($n)
{
$this->test = $n;
}

function getPrivate()
{
echo $this->test;
}

}

class Derived extends Base
{
}

$d = new Derived();
$d->setPrivate(4);
$d->getPrivate();
Аноним 19/04/17 Срд 00:50:52  976075
>>976054
>>976070
Уточню ещё раз: если модель переместить в common я всё так же могу создавать папки в бекэнде и не могу создавать во фронтэнде.
Аноним 19/04/17 Срд 00:55:25  976078
>>976042

При наследовании создается новый класс на основе старого (условно говоря, берутся методы и поля из предка и к ним добавляются поля и методы наследника) Объект всегда создается один.

Это примерно то же, как если бы ты вручную скопипастил старый класс, переименовал его и дописал новые поля и методы. Только разница в том, что при наследовании сохраняется информация о том, какой класс от какого унаследован (а в случае копипасты связи между классами нет).

Ты не путаешь понятия класса и объекта? Класс - это описание, какие поля и методы будут в созданных объектах. Наследуются именно классы, а не сами объекты.

Аноним 19/04/17 Срд 00:57:53  976080
>>976054
>>976070
>>976075
Опять уточню: даже если модель переместить во фронтэнд, я все равно могу создавать папки в бэкэнде и не могу во фронтэнде.
Аноним 19/04/17 Срд 01:04:16  976083
>>976054

А текст и подробности ошибки? Что в логах написано?

>>975768

Ты вредный совет даешь. Какая выгода ставить кучу точек и кавычек, если можно без них?

Писать стоит либо так:

echo "Прошло $time секунд\n";

либо так, если нужны выражения:

printf("Прошло %d секунд\n", $time + 100 - $start);

Подробности про printf в мануале.

Писать в шапке много переменных плохо, код становится хуже читаем. Вот это стоит вынести наружу:

$bankSum = 10000;
$age = 16;
$bankPercent = 1.1;

Да и менять значения удобнее, когда они в начале, а не в середине программы.
Аноним 19/04/17 Срд 01:06:17  976085
>>976078
Спасибо за объяснение, но я немного другое не могу понять >>976071
Вот если класс derived унаследовал от base свойство тест (приватное для родительского класса), то почему я могу его изменить из?
Аноним 19/04/17 Срд 01:06:53  976086
>>976085
из Derived
Аноним 19/04/17 Срд 01:09:36  976088
>>975727

А ты задачу про таблицу умножения решил? Тебе надо лучше разобраться с циклами. Вот тебе задача в помощь:

- выведи таблицу умножения для чисел от 1 до 5:

1 x 1 = 1
1 x 2 = 2
1 x 3 = 3
...
5 x 5 = 25

- выведи в конце сумму всех получившихся чисел (1 + 2 + 3 ... + 25 = ?)

>>975725

зайди на fl.ru и посмотри задачи

>>975719

Ты что-то путаешь. "Я отправляю данные на сервер-клиент" - на сервер или на клиент? Ты вообще понимаешь ,что значит сервер и что значит клиент?

Клиент - это тот, кто запрашивает какие-то данные
Сервер - тот, кто обслуживает клиентов (serve - обслуживать) и выполняет их запросы, дает им ответы

Аноним 19/04/17 Срд 01:13:13  976092
yiiй.png (61Кб, 1875x944)
>>976083
Скрин ошибки.

Последние логи и, кажется, они не регистрируют эту ошибку.

AH00112: Warning: DocumentRoot [C:/Apache24/docs/dummy-host2.example.com] does not exist
[Wed Apr 19 00:48:10.490833 2017] [mpm_winnt:notice] [pid 1860:tid 424] AH00455: Apache/2.4.25 (Win64) PHP/7.1.2 configured -- resuming normal operations
[Wed Apr 19 00:48:10.490833 2017] [mpm_winnt:notice] [pid 1860:tid 424] AH00456: Apache Lounge VC14 Server built: Dec 17 2016 11:15:57
[Wed Apr 19 00:48:10.506459 2017] [core:notice] [pid 1860:tid 424] AH00094: Command line: 'D:\\user\\apache\\bin\\httpd.exe -d D:/user/apache'
[Wed Apr 19 00:48:10.506459 2017] [mpm_winnt:notice] [pid 1860:tid 424] AH00418: Parent: Created child process 6296
AH00112: Warning: DocumentRoot [C:/Apache24/docs/dummy-host.example.com] does not exist
AH00112: Warning: DocumentRoot [C:/Apache24/docs/dummy-host2.example.com] does not exist
AH00112: Warning: DocumentRoot [C:/Apache24/docs/dummy-host.example.com] does not exist
AH00112: Warning: DocumentRoot [C:/Apache24/docs/dummy-host2.example.com] does not exist
[Wed Apr 19 00:48:11.378009 2017] [mpm_winnt:notice] [pid 6296:tid 396] AH00354: Child: Starting 64 worker threads.
[Wed Apr 19 00:48:12.378949 2017] [mpm_winnt:notice] [pid 7036:tid 400] AH00364: Child: All worker threads have exited.
Аноним 19/04/17 Срд 01:22:49  976099
>>975666

В CSS есть псевдокласс :empty - https://developer.mozilla.org/ru/docs/Web/CSS/:empty - может можно его задействовать.

Только наверно HTML придется переделать, например добавлять префикс n: через псевдоэлемент ::before.

>>975668

В твоем случае проще всего написать функцию, которая получает на вход массив с данными и выдает на выходе либо кусок SQL кода с экранированными данными через $pdo->quote(), либо кусок SQL кода с плейсхолдерами и массив значений плейсхолдеров.

Попробуй посмотреть как в Doctrine DBAL сделано:

док-я: http://docs.doctrine-project.org/projects/doctrine-dbal/en/latest/reference/data-retrieval-and-manipulation.html#insert
код: https://github.com/doctrine/dbal/blob/master/lib/Doctrine/DBAL/Connection.php#L680

Важно написать код так, чтобы SQL-инъекция (подмена запроса) была невозможна, чтобы при любых входных данных вид запроса соответствовал задуманному.

А в общем, для построения произвольных запросов из кусков есть паттерн Query Builder: http://docs.doctrine-project.org/projects/doctrine-dbal/en/latest/reference/query-builder.html

>>975591

Может какое-нибудь кеширование не включено?

>>975525

Тесты с вопросами на знание той или иной технологии.

>>975278

Как ты собрался писать на PHP без знания PHP?

>>975196

Я им писал пару месяцев назад, недавно ответили, что проверят, в чем проблема.

>>975191

mb-функций нет на ideone, есть на 3v4l.org
Аноним 19/04/17 Срд 01:41:40  976106
>>975191

Так понять сложно, попробуй натыкать echo которое будет выводить значения переменных, и посмотреть, где значения отличаются от ожидаемых.

Также у тебя там 2 свитча для математических действий, попробуй объединить в один.

>>975183

Сам ты сталкер. Query Builder используется для сборки запроса из кусков с условиями, если запрос всегда одинаковый, то смысла его использовать нет.

То же касается и DQL, смысл тут его использовать если можно без него обойтись.

Ты вместо аккуратного запроса нагородил кучу скобок и стрелок и наверно думаешь что Query Builder придумали как замену SQL.

>>975159

Очень полезная фича. Раньше тип указывали через phpdoc вроде @return SomeClass, а теперь можно нормально на уровне языка указать.

>>975117

Во-первых, я не советую исплоьзоваь NativeSQL с маппингом. Если тебе нужен маппинг - используй DQL или Query Builder , если тебе нужно посчитать количество то можно выполнить обычный SQL запрос без маппинга, напрямую через DBAL.

Во-вторых, ассоциации в доктрине загружаются лениво. То есть они не загружаются из БД пока к ним не обратишься (если только ты не указал для них жадную загрузку).

Соответственно запрос вида

$em->find('Thread', $threadId);

выберет только одну запись из БД, и не будет выбирать посты, авторов, картинки и что там еще может быть связано с этой записью.

Для подсчета числа постов ты можешь использовать

SELECT COUNT(*) FROM posts WHERE thread_id = ?

Но еще эффективнее будет просто в треде сделать поле "число постов" и брать данные из него. Это назвается денормализация - добавление избыточных данных в базу ради повышения производительности.

Далее, тебе надо для треда получить 3 последних поста. Тут есть много вариантов:

1) через DQL делаем запрос вида

SELECT p FROM posts WHERE IDENTITY(posts.thread) = ? ORDER BY postedTime DESC
с опцией setMaxResults(3)

Это будет преобразовано в SQL запрос вида SELECT .. ORDER BY postedTime DESC LIMIT 3

Разумеется, нужен индекс по (thread_id, postedTime) для оптимальной выборки.

Этот метод позволяет выбрать 3 последних поста оптимально, но только для 1 треда. Нужно для 10 тредов - нужно 10 запросов.

2) потому можно при желании применить денормализацию и хранить в БД ссылки на 3 последних поста каждого треда. Либо в самой таблице тредов в виде id, либо в виде таблицы связи многие-ко-многим треды <-> посты. В Доктрине она оформляется как ассоциация hasMany по моему.

Это позволяет одним запросом выбрать 3 последних поста для N тредов сразу.

Тебе важно разобраться, как именно работает Доктрина. Какие она может генерировать запросы в разных случаях.

Ну например в такой ситуации:

1: $thread = $em->find('Thread', $threadId);
2: $posts = $thread->getPosts();
3: foreach ($posts as $post)

делается 2 SQL запроса:

В строчке 1 - SELECT FROM threads WHERE id = ?
В строчке 3 (да, в 3, а не в 2, так как у нас ленивая коллекция) - SELECT FROM posts WHERE thread_id = ?

Если понимать, как работает Доктрина,то станет видно что любые обращения к коллекции $posts вызовут загрузку всех постов треда в память. даже если они все нам не нужны.

>>975117

>>Тогда будет один запрос. Но на мой взгляд, это только утяжелит таблицу, и приведет к дублированию данных без особой выгоды
> Это действительно так плохо? Это решение решило бы проблему.

Ну давай взвесим:

1) если не дублировать данные. таблица будет раза в 2 меньше, но надо сделать 2 запроса которые можно объединить в 1 с помощью UNION
2) если дублировать данные, таблица будет больше, при добавлении поста надо делать больше вставок, зато мы делаем 1 запрос вместо 2.

По моему пункт 1 выгоднее. У нас ведь была цель избавиться от потенциально большого числа запросов (если цепочка постов длинная), а два запроса или один - разницы особой нет.

Аноним 19/04/17 Срд 01:59:58  976108
>>974992

Ссылка "назад" ведет на доску.

Для прокрутки нужен яваскрипт, причем его надо будет переделывать, если в скриптах борды что-то поменяется.

>>974639

Удобный формат же. Например данные для Яндекс-Маркета в нем предоставляются.

>>974624

В Доктрине один объект = 1 строка таблицы. Если таблиц несколько, то нужно несколько объектов со связями между ними.

flush() сбрасывает в базу все изменения во всех объектах. То есть тут тебе делать ничего не надо, доктрина сама все найдет и сохранит.

>>974620

Вообще для фильтров удобно сделать объект, представляющий состояние фильтра, а не гонять туда-сюда многомерные массивы непонятной структуры.

Для произвольных выражений, может быть, придется строить дерево выражения из узлов. Стоит подумать, прежде чем за это браться. Но на массивах это будет в разы запутаннее. Это вообще как-то неправильно, использовать массив там, где лучше подходит объект.

>>974607

Полезно представить настройки фильтра в виде объекта. И передавать уже этот объект куда надо:

- в модель для поиска записей
- в шаблон для вывода формы с подставленными значениями

А городить многомерные массивы - это запутывать код.


Аноним 19/04/17 Срд 02:09:09  976109
>>974588


> [(Дд|Dd)]
Ты не понял, как работают квадратные скобки. В твоем случае это читается как "любой один из символов: скобка, буква Д, вертикальная черта". То есть в квадратных скобках ( или | это обычные символы.

> \\s\\S
Это позволяет между буквами "д" и "у" вставить любые другие буквы, например твоя регуялрка среагирует на фразу

аД кУ пиР мирА коК

Пример: https://ideone.com/QJ7KLx

Также ты забыл флаг u, без которого корректная работа с русскими буквами вообще не гарантируется.

>>974585

Можно состояние фильтра представить в виде объекта. И передавать везде этот объект. Или при желании усложнить - в виде коллекции объектов.

class PhoneFilter
{
private $isHdmi = false;
private $hasIps;
...
}

>>974463

Можно работать на винде. Можно поставить вируталку с дебианом.

Линукс изучить полезно в любом случае.

>>974448

Неправильная конструкция. Ведь getcwd получает текущий каталог процесса (не каталог где расположен файл), он может быть любым.

Надо использовать например __DIR__ или заданный в конфиге путь. Желательно все, что связано с путями, вынести куда-нибудь в класс, чтобы это было собрано в одном месте и можно было менять, не трогая остальной код.



Аноним 19/04/17 Срд 02:15:19  976111
>>974448

Почитай теорию, что такое "текущий каталог процесса":

http://heap.altlinux.org/issues/textbooks/linux_intro/Filesystem_use.html
https://ru.wikipedia.org/wiki/%D0%A0%D0%B0%D0%B1%D0%BE%D1%87%D0%B8%D0%B9_%D0%BA%D0%B0%D1%82%D0%B0%D0%BB%D0%BE%D0%B3
http://php.net/manual/ru/function.chdir.php

У каждого процесса есть "текущий каталог", он предназначен в первую очередь для того, чтобы при вызове команды не надо было указывать полный путь к файлам, с которыми она работает. Соответственно внутри твоей программы этот каталог может быть любым (он зависит от того, какой каталог был текущим когда программа была запущена). Полагаться на него нельзя.

Вот еще теория:

https://ru.wikibooks.org/wiki/UNIX#.D0.A4.D0.B0.D0.B9.D0.BB.D0.BE.D0.B2.D0.B0.D1.8F_.D1.81.D0.B8.D1.81.D1.82.D0.B5.D0.BC.D0.B0


Аноним 19/04/17 Срд 02:17:51  976114
>>974431

Можно отдельный класс для отдачи файлов. Но не все можно в него помещать.

Вот допустим функция для определения полного пути к файлу. Если она нужна не только при скачивании, а где-то еще то не логично ее помещать в класс для отдачи файлов, а логично положить в какой-то вспомогательный класс, которым все могут пользоваться.

> А нормально ли будет сделать пхп-файл в котором константами будут прописаны все эти пути? Или надо как-то хитрее делать?
Лучше класс с методами, возвращающими нужные пути. Так гибче.

Аноним 19/04/17 Срд 02:27:40  976116
>>976085

Потому что приватное свойство доступно только внутри одного класса. То есть в куске кода между class Base и завершающей скобкой. Если ты хочешь чтобы оно было доступно в потомках, надо было использовать protected.

Идея в том, что мы можем специально запретить доступ к нему напрямую и предоставлять доступ только через методы. Представь, если Base и Derived пишут 2 разных человека и тот кто пишет Base не хочет чтобы в поле записали что-то неправильное. Он делает поле приватным разрешает доступ только через методы. Или например этот человек, который пишет Base, не хочет заставлять второго изучать его класс целиком, а просто предоставляет готовый метод, который надо вызвать. Соответственно он помечает с помощью private то, что второму можно не смотреть.

Это называется инкапсуляция, почитай http://learn.javascript.ru/internal-external-interface

Инкапсуляция позволяет разделять код на слабосвязанные части, классы, что упрощает его понимание, когда объем кода большой.

> Вот если класс derived унаследовал от base свойство тест (приватное для родительского класса),
Свойство он унаследовал, а право доступа к нему - нет. Доступ к нему имеют только методы, унаследованные из Base.

Ограничение доступа идет именно на уровне исходного кода, то есть доступ к private возможен только из кода, находящегося внутри определения класса. Соответственно автор класса получает гарантию что никто не может обойти это ограничение снаружи. Это позволяет ему давать гарантии насчет своего класса, что он работает так, как описано в документации. То есть автор класса, если он не позволяет никому править его код, имеет полный контроль над тем, как будет работать класс, и наследование не поможет обойти заданные им правила.

Вообще, "гарантии" - это важное условие для написания надежных программ.
Аноним 19/04/17 Срд 02:34:37  976121
>>976085

Кстати есть еще интересный случай, когда у нас 2 приватных поля с одинаковым именем:

class Base
{
private $a;
}

class Derived extends Base
{
private $a;
}

Изучи, как это работает. Подвох тут в том, что это не одно поле, которое переопределено, а именно 2 разных поля - Base#$a и Derived#$a. методы, описанные в Base видят первое, а методы в Derived - второе.

И опять же, это значит что класс-наследник не может обойти ограничения доступа к приватному полю.

>>976092

Это общий лог Апача. Наверно у Юи есть свой лог и он перехватывает и пишет ошибки туда. Поищи.

А вообще, причина видна, хоть твой скриншот ее не показывает, но можно там увидеть "No such file or directory" то есть ты пытаешься создать папку внутри несуществующей папки.


Аноним 19/04/17 Срд 03:31:51  976132
>>976121
Папка существующая - это 100%.
Аноним 19/04/17 Срд 07:52:58  976155
>>976088
Я знаю что такое клиент. Ну ок, отправляю с одного сервера на другой сервер, где хостится сайт.
Аноним 19/04/17 Срд 10:12:49  976183
>>976099
> кеширование не включено?
Включено, включено. Я же понимаю все. Но вообще он реально тяжелый, та же юи в 3 раза быстрее летает (а у юйки весьма средняя скорость по больнице я бы сказал). Люмен пашет шустрее юи уже, но это ж не сам ларавел. Под скоростью, конечно, имеется ввиду не только скорость обработки запроса, но и потребляемые ресурсы. И конечно все это будет не критично, если нет проблем с расширением серваков, но я как-то такой подход не одобряю. Да и не осуждаю сам ларавел, нишевая тема, удобная в некоторых местах, востребованная сейчас (юи только на легаси уже, нового почти ничего нет) просто вот не взял бы я его на свой проект, на мой взгляд есть масса решений гораздо лучше и эффективнее.
Аноним 19/04/17 Срд 10:15:28  976184
>>976109
>аД кУ пиР мирА коК
>
>Пример: https://ideone.com/QJ7KLx

Переделал https://3v4l.org/7SKVK

И вот граммар нази еще https://3v4l.org/0G8MP
Аноним 19/04/17 Срд 10:24:58  976188
>>976106
>Если понимать, как работает Доктрина,то станет видно что любые обращения к коллекции $posts вызовут загрузку всех постов треда в память. даже если они все нам не нужны.
Такое поведение будет и при getRepository()?

>По моему пункт 1 выгоднее. У нас ведь была цель избавиться от потенциально большого числа запросов (если цепочка постов длинная), а два запроса или один - разницы особой нет.
Но нужно сделать не два запроса, а ещё столько же сколько детей. Получается мы делаем эти запросы либо при вставке либо при получении. Только есть разница, что мы вставляем только один раз при добавлении поста, а получаем каждый раз когда пользователь получает цепочку. С такой перспективы кажется лучше 2-ой вариант.
Аноним 19/04/17 Срд 13:53:42  976299
Поцоны, помогине с регуляркой. Допустим, в массиве текста есть такие склееные слова "васяПетя". И мне нужно эти слова расклеить.

Нагуглил регулярку: ([а-я])([А-Я]) -проверил по regex101.com - находит такое совпадение.

Пишу скрипт:

<?php
$str = 'текст васяПетя текст';
$result = preg_match_all('/([а-я])([А-Я])/', $str, $found);
echo "Matches: $result<br>";
print_r($found);
?>

Нихера не находит. Как его расклеить-то? Помогите, анончики.
Аноним 19/04/17 Срд 14:19:55  976314
>>976299
Твоя регулярка найдет только по одному символу в подмаске. Тебе нужно указать что может встретиться от 1 и более символов, с помощью метасимвола +.

'/([а-я]+)([А-Я]+)/'

https://secure.php.net/manual/ru/regexp.reference.meta.php
>+ квантификатор, означающий 1 или более вхождений
Аноним 19/04/17 Срд 15:14:32  976335
>>976314
> Тебе нужно указать что может встретиться от 1 и более символов, с помощью метасимвола +.
Спасибо за отклик, это уже лучше! Но почему пикрелейтед не вытаскивает второе слово?
Аноним 19/04/17 Срд 15:50:59  976347
>>976335

Флаг g (global) поставил? Без него на regex101 ищется только первое совпадение.
Аноним 19/04/17 Срд 15:52:45  976348
>>976347
Дык, да https://regex101.com/r/YQ0v8E/1
Аноним 19/04/17 Срд 15:56:12  976350
>>976188

getRepository возвращает объект репозитория и ничего не загружает из базы вообще.

> Но нужно сделать не два запроса, а ещё столько же сколько детей.
Нет, я говорил про вариант, где для построения цепочки нужно ровно 2 запроса:

1) для постов, до которых можно дойти по стрелкам в графе WHERE post = A (на которые ссылается A)
2) для постов, до которых можно дойти, идя против направления стрелок в графе WHERE reference = A (которые ссылаются на A)

Посмотри на граф со стрелками и может быть станет понятнее.

Я предлагаю сделать таблицу, где мы для каждого поста (узла в графе) указываем все посты, в которые можно дойти по стрелкам из него.

Аноним 19/04/17 Срд 15:56:57  976351
>>976335
А-Я найдет ведь только буквы в верхнем регистре. По сути чтобы "раслеить" этого достаточно. Но раз уж хочешь чтобы находило целиком, то ([А-Я][а-я]плюсик) во второй части. Наверно. Не проверял.
Аноним 19/04/17 Срд 16:03:50  976353
>>976351
А по хорошему тебе достаточно было бы найти из твоего склеенного говна позицию вхождения "яП", а не все сразу, что твоя первая регулярка и делает, притом там даже скобочки не нужны. С этим уже можно будет работать.
Аноним 19/04/17 Срд 16:05:42  976354
>>976351
А, точно же елки-палки>>976353
Да не, там разные слова склеены, текста дофига. Но спасибо!
Аноним 19/04/17 Срд 16:29:57  976361
>>976350
>2) для постов, до которых можно дойти, идя против направления стрелок в графе WHERE reference = A (которые ссылаются на A)
Но если пост A ссылается на пост B то нужно будет сделать запрос WHERE reference = B, и так далее.

При условии что структура такая:

>post | reference | depth
>A | B | 1
>A | C | 1
>A | D | 2
>A | E | 2
>B | D | 1
>B | E | 1
Аноним 19/04/17 Срд 17:49:50  976391
Делаю файлообменник, узнал про константу directory separator. Ее везде использовать вместо слеша в функция move_uploaded_file и подобных?
Аноним 19/04/17 Срд 18:21:43  976403
Что должно содержаться в ссылке для активации аккаунта, которая приходить на почту? Хэш подойдет?
Аноним 19/04/17 Срд 21:27:16  976518
Опчик проверь опечаточник https://3v4l.org/p736Z
Аноним 20/04/17 Чтв 02:11:25  976668
>>975806
Переменные в кавычках должны парсится, вне кавычек нет. Парсинг занимает время у php. Если у тебя большие тексты, в которых переменные, то быстрее скрипт будет работать, если ты их вынесешь из кавычек.
Аноним 20/04/17 Чтв 02:12:42  976669
>>976668
Алсо сами кавычки на одинарные надо заменить, в них парсинга не происходит.
Аноним 20/04/17 Чтв 02:26:48  976671
>>976668

Если ты пишешь "быстрее", пиши насколько. Иначе ты ничем не отличаешься от маркетологов которые берут цифры с потолка в рекламе.

Анонам советую не слушать таких пустословов. ставьте такие кавычки, которые нравятся.

По твоей логике и комментарии к коду писать не стоит, ведь из парсинг занимает время у PHP? И переменные наверно не стоит называть длиннее, чем одной буквой?

Аноним 20/04/17 Чтв 04:04:53  976687
>>976518

Твоя программа реагирует на любую латиницу, а надо реагировать только на латинские буквы в русских словах, и на русские в латинских. То есть на слова со смешанным алфавитом. Одно из решений - написать регулярку, которая будет искать такие слова, другое - разбить текст на слова и обрабатывать их по очереди.

>>976403

А зачем ты вообще присылаешь ссылку на почту? Попробуй начать рассуждать с этого.

>>976391

Можно, она для этого и придумана, но неудобно. Прямой слеш поддерживается и в Линуксе, и в винде, проще использовать его.

Вообще, иногда полезно иметь класс с вспомогательными функциями для работы с путями, вроде склеивания путей.

Вот примеры таких библиотек (нашел на packagist.org по слову path):

- https://github.com/donutclub/php-path
- https://github.com/vweevers/php-path
- https://github.com/webmozart/path-util

>>976361

Кажется, я понял, что ты имел в виду.

Тебе надо получить все узлы в графе, которые достижимы из заданного, если идти по стрелкам в любом направлении? Тогда да, для ускорения выборки надо заранее строить их списки для каждого поста, и списки могут быть огромные. Там у тебя легко могут сотни постов набраться (нужно конечно делать тесты чтобы узнать).

> Получается мы делаем эти запросы либо при вставке либо при получении. Только есть разница, что мы вставляем только один раз при добавлении поста, а получаем каждый раз когда пользователь получает цепочку.
Ну да, либо мы строим полный список заранее, либо делаем много запросов.
Аноним 20/04/17 Чтв 04:05:13  976688
>>976184

> \\W*\_*
Это лучше записать как [\\W_]*

А так, верно.

Правописание

удобнее записать пары регулярка - замена так:

[
'/xxx/' => 'yyy',
'/xxx/' => 'yyy'
]

Сразу видно, что чему соответствует.

"Зделаем" не найдется.

В остальном все хорошо.

>>976155

Тебе нужно какое-то хранилище, так как ты отправляешь данные с первого сервера на второй, и второму их надо где-то сохранить, чтобы позже отдавать своим клиентам. Обычно базу данных используют для этого, хотя возможны и другие варианты.

>>976132

Возможно ты пытаешься создать папку, которая уже существует. Или возможно все-таки родительская папка не существует. Почему бы не проверить с помощью is_dir() ?

Если линукс, могут быть проблемы с правами на одну из папок в пути.
Аноним 20/04/17 Чтв 04:05:33  976689
>>974427

8: https://jsfiddle.net/Lpguk5kx/

Ок, верно.

10: https://jsfiddle.net/18504y5y/3/

> text-decoration-color: #f0c000;
Это что-то новое из CSS3, я-то в общем не против, только надо смотреть уровень поддержки в браузерах: https://caniuse.com/#search=text-decoration

Пока очень мало, потому советую сделать по-старому, бордером. Ну и сайт запомни, пригодится.

Так, в общем верно.

Вообще, для задания размера картинки используется запутанная (но работающая) схема. Я бы для надежности добавил max-width 100% на figure. Оно и без него работает, но с ним как-то определенности больше становится.

Ну и отступ слева наверно стоит убрать у figure, его же вроде нет на исходной картинке.

11: https://jsfiddle.net/3qm145b4/2/

> top: -100px;
Этого на реальном сайте ведь маловато, если только там нет overflow hidden.

Вообще, хорошо сделано.

> На jsfiddle немного глючит из-за наличия посторонних элементов на странице.
Да, я знаю, но если выделить мышкой на слово "Radio" и нажать Tab, то все работает.

12: https://jsfiddle.net/qozy8zsL/7/

> .tabs-wrapper input + label
Вот это нехорошо, оно ведь подействует не только на заголовок вкладки, но и на любую пару input + label в теле вкладки. Нужно делать селектор более точечным. Например использовать класс .tab-head тут

Лучше сразу писать селекторы без побочных эффектов, чем потом искать косяки в верстке.

Так, в общем, верно.
Аноним 20/04/17 Чтв 04:05:52  976690
>>974373

В задаче есть массив, и он задает правило, какие буквы на какие заменяются. Проблема в том, что заменяются не все буквы, а только от а до й. Нужно шифровать и другие буквы. Также, надо дописать код для расшифровки (можно использовать array_flip).

Также в помощь тебе мануал по strtr: http://php.net/manual/ru/function.strtr.php

Для расшифровки нам надо сделать обратную замену (например менять 1 на "а"). Для этого можно перевернуть массив кодов с помощью array_flip. Попробуй это сделать и вывести получившийся массив через var_dump.

>>974246

Не работает с utf-8 и нелатинскими буквами.

>>974071

mb_strtolower
mb_strtoupper

>>974075

Квадратные скобки не требуются.

>>973866

Если ты включил режим исключений (PDO::ERRMODE_EXCEPTION, погугли) то проверять ничего не требуется. Если произойдет ошибка (например база недоступна или неверно написан запрос) то выбросится исключение и программа прервется.

В этом и плюс исключений, что тебе делать ничего не надо для их обработки. Урок https://github.com/codedokode/pasta/blob/master/php/exceptions.md

Это в mysqli надо каждый вызов проверять, а тут не надо.

Если же ты хочешь проверить, что запрос что-то нашел, хотя бы 1 запись, то это стандартно - if (count(...) > 0)


>>973871

Ничего не надо возвращать, при ошибке выбросится исключение
Аноним 20/04/17 Чтв 04:06:10  976691
>>973704

Полный путь от корня диска хранить не надо, так как тогда проект не перенести в другую папку. лучше хранить путь относительно папки с файлами. Чтобы при переносе ничего не надо было менять.

В коде сделать функции которые из этого относительного пути генерируют URL и полный путь.

При скачивании в URL в конце должно стоять исходное имя файла: /download/13/файл%20файл.txt

> Также неизбежно ли отказываться от конструктора класса-модели
Удобнее конструктор без аргументов.

> и создавать отдельный метод setProperties
Можно и его, а можно свойства напрямую или через сеттеры менять.


>>973774

Названия у тебя очень неудачные и трудно понимать код. ЧТо такое TempHandler например? Непонятно. Что такое TempInterface? Комментариев тоже нет.

> $object->tempChanges()
наверно tempChanges надо в интерфейсе описать? Так как сейчас у тебя там ни одного метода не описано и значит вызывать ты тоже ничего не можешь.

> if (!empty($order->getDirty)) {
Скобки забыл

> $temp->name = 'change';
тут наверно константа нужна

> $temp->value = json_encode($tempArray);
Если можно, лучше бы в памяти хранить сам массив, а преобразовывать в JSON только в момент сохранения в БД.

https://github.com/MakeevD/MicroCrm/blob/master/app/Helpers/SearchOrders.php
тут тоже много странного, например почему фильтр записей наследуется от модели, почему куча статических методов которые явно должны быть нестатическими?

Фильтр можно было сделать в виде объекта с полями, соответствующими отдельным критериям поиска.

>>973748

Если в тайпхинте стоит интерфейс, то тебе не надо знать, какой именно класс тебе передали. Ты и не можешь это сделать, так как новые классы (реализующие интерфейс) могут быть написаны уже после написания твоего кода.

Ты должен просто вызывать методы, которые описаны в интерфейсе, а не гадать, какой именно там класс.

Подход вида if ($x instanceof Class1) не работает, так как позже могут допсать новые классы, реализующие интерфейс.
Аноним 20/04/17 Чтв 04:06:28  976692
>>973506

зависит от ситуации, что это за данные, что в них может быть, что мы хотим с ними делать.

>>973465
>>973345


> https://ideone.com/DV6lCb

> [8]
Тут скобки не нужны

> \\)\\(\\-\\s
Тут ты задаешь определенный порядок, в котором идут символы. Но они могут ведь идти в любом порядке. надо написать "один из указанных символов", с помощью квадратных скобок, и добавить после этого звездочку.

> '/(\\-|\\s|\\(|\\)|)/';
Зачем тут последняя вертикальная черта? Лишняя ведь.

> /(\\+\\s
7)/'
Тут стоит добавить еще что это должно быть в начале

>>973548

Я бы тут использовал IN, а недостающие пустые записи вставил на стороне PHP, если так можно делать.

>>973383

> select from A where id > (select min(id) from B)
> Сколько раз выполнится вложенный запрос? Столько, сколько строк в A? Если да, то можно ли как-то это оптимизировать?
Зависит от используемой базы данных. Некоторые умны, чтобы выполнить вложенный запрос 1 раз, некоторые нет. Проверить в MySQL план выполнения можно через EXPLAIN : https://www.google.ru/search?q=habr+explain+sql&newwindow=1&gbv=1

Оптимизировать можно с помощью переменной в PHP, выбрав первым запросом число, положив в переменную и поставив во второй запрос. В языке SQL тоже есть переменные (@x), но возможности работы сохранения данных из запроса в переменную разные в разных базах данных.

В MySQL ( https://dev.mysql.com/doc/refman/5.7/en/select-into.html ) синтаксис такой:

SELECT MIN(id) FROM table INTO @minValue;
SELECT FROM A WHERE is > @minValue;

В Postgres есть что-то такое: https://www.postgresql.org/docs/9.2/static/plpgsql-statements.html#PLPGSQL-STATEMENTS-SQL-ONEROW
Аноним 20/04/17 Чтв 04:06:45  976693
>>973182

Для подсветки текста у нажатой кнопки.

>>973135

Эти кнопки делаются на основе input radio или checkbox. Мы делаем кнопку в виде тега label, который передает клик на input, и в зависимости от состояния инпута мы подсвечиваем текст кнопки.

Пример:

<input type="checkbox" class="btn-checkbox" id="checkbox1"><label for="checkbox1" class="btn-label">Текст</label>
.btn-checkbox:checked + .btn-label { background: yellow; }

Но тут есть недостаток - надо указывать атрибуты id, for. Избавиться от них, можно положив инпут внутрь label.

Изучи селекторы + и ~.

На хабре есть статьи, гугли. Например

- https://habrahabr.ru/post/144104/
- https://habrahabr.ru/post/154719/
- https://habrahabr.ru/company/imagecms/blog/204030/
- http://dimox.name/custom-checkboxes-and-radio-buttons-using-css-only/


>>973151

тут нет связи между label и input - их надо либо вложить друг в друга либо связать через id.

Аноним 20/04/17 Чтв 10:07:15  976751
>>976688
>Тебе нужно какое-то хранилище
Ну да, можно и через базу, но это не главное.
Возможно, вы не совсем поняли мой вопрос, пожалуй упрощу его:
Как записать в файл данные из POST-запроса? Просто напишите пример кода, пожалуйста.
Аноним 20/04/17 Чтв 13:19:39  976826
313131313.png (14Кб, 768x211)
Антуаны, курю ваш гайд
Поясните, зачем нужны фигурные скобки на пикрилейтед? И без них же зоебись работает
Аноним 20/04/17 Чтв 16:00:05  976909
Анонсы, у меня к вам есть один тупой теоретический вопрос. Будьте добры ответьте, если не затруднит. Возможно ли любую рандомную верстку сделать адаптивной тупо скриптами без человека? Ну теоретически.
Аноним 20/04/17 Чтв 20:00:55  977026
>>976687
>А зачем ты вообще присылаешь ссылку на почту? Попробуй начать рассуждать с этого.
Чтобы удостовериться что почта существует, конечно же. В таком ключе становиться очевидно, что не так важно что должно содержаться в ссылке. Просто я часто видел в таких ссылках содержится какой-то хэш-код.
Аноним 20/04/17 Чтв 21:18:58  977047
http://ideone.com/sdVGXW
как заставить ничью работать нормально?
Аноним 20/04/17 Чтв 22:21:42  977068
>>976691

Относительно папки с файлами это значит включая ее в этом пути? /downloads/2010/10/10 или /2010/10/10
Аноним 20/04/17 Чтв 22:39:31  977073
>>977047
>PHP Notice: Undefined variable: playedDice2 in /home/UwjuzN/prog.php on line 13
>Молодой человек, это не для вас написано
Аноним 21/04/17 Птн 00:29:22  977122
>>977073
<3
Аноним 21/04/17 Птн 09:09:42  977180
Опчик проверь задачу "Клавиша Shift" https://3v4l.org/0N8g0

Анончики как в регулярках в утверждениях указать чтобы проверялась наличие не только одной точки/знак вопроса/восклицательный знака, но и все возможные варианты, вот такие например .../??/!!!!

> preg_split('/((?<=[.?!]))/iu', $text, 0, PREG_SPLIT_NO_EMPTY)
Аноним 21/04/17 Птн 18:48:04  977418
Чому этот код вешает мне браузер?
$newPow = function($b, $e) use (&$newPow){
if($e){

return $b * $newPow($b, $e);
} else {
return ;
}
};
echo $newPow(4, 4);
Аноним 21/04/17 Птн 18:54:29  977421
Господа, вот вопрос. Я хочу сделать пакет для packagist / composer. Но мне нужны две версии. Под PHP 7.0 (с хинтингом скалярок) и под PHP 5.6

Ближайшее время (год) я буду поддерживать оба. Кодобаза будет почти одна, только я хинтинг из PHP 5.6 уберу. Какой best practice по проворачиванию такого (две параллельные ветки в пакаджисте), при условии что я не хочу версии 1.0 & 2.0, я хочу чтобы версия была одна между двумя бранчами?
То есть есть ли у меня возможность каким-то образом называть теги так, чтобы packagist вел именно параллельные версии? Вроде есть только постфикс RC для релиз кандидатов. Само собой в обоих бранчах будет свой композер.json (отличается только require для php) и настройки для CI.
Аноним 21/04/17 Птн 20:07:14  977444
>>977418

Браузер он вряд ли может подвесить, скорее сервер. Попробуй поставить там echo внутри функции и выводить, с какими аргументами она вызвана.

Возможно что у тебя тут получается бесконечная рекурсия. Тогда PHP будет выполнять скрипт, пока не закончится память. Но в современных ОС используется swap, и прежде чем PHP умрет, он загоняет остальные программы в своп, что и создает ощущение что все тормозит.

https://ru.wikipedia.org/wiki/%D0%9F%D0%BE%D0%B4%D0%BA%D0%B0%D1%87%D0%BA%D0%B0_%D1%81%D1%82%D1%80%D0%B0%D0%BD%D0%B8%D1%86

Как с этим бороться? Проверь такие настройки с помощью phpinfo():

- max_execution_time - ограничение на время выполнения
- memory_limit - ограничение на потребление памяти

Если у тебя установлен xdebug то доступна настройка xdebug.max_nesting_level ограничивающая глубину рекурсии. https://xdebug.org/docs/basic

Аноним 21/04/17 Птн 20:15:16  977446
>>977418

Также можно открыть диспетчер задач от администратора и прибить процесс php (или apache), который потребляет много памяти и ест процессор.

>>977421

Ты изучал описание как композер работает с версиями? https://getcomposer.org/doc/articles/versions.md

Ты можешь попробовать сделать одну версию основной, а другую под тегом вроде php7 и ставить ее явным указанием ветки

"mnylibrary": "dev-php7"

Но вообще я бы советовал тебе отказаться от этой идеи. Много труда потратишь. Проще подождать пока php7 наберет долю. А проверять тип аргумента можно костылем вроде

TypeHints::assertIsString($x);

Увы, кроме ссылки на документацию, подсказать наверно ничего не могу. Ну еще есть вариант сделать 2 репозитория конечно.

> при условии что я не хочу версии 1.0 & 2.0, я хочу чтобы версия была одна между двумя бранчами?
Вот это кстати хорошая идея, сделать 2 версии. Можно для второй указать требование php7, а для первой php5, и в таком случае композер сам выберет нужную версию в зависимости от версии PHP (если указать в зависимости "mylbrary": ">= 1.0").

Я бы конечно не советовал ставить зависимости на последние версии PHP, это сужает область применения твоей библиотеки.

Аноним 21/04/17 Птн 21:48:43  977493
Решаю тут задачку из учебника опа: дан текст, содержащий слова на русском и английском языках. В некоторых словах часть русских букв заменена на похожие на них латинские, и наоборот. Напиши программу, которая находит все такие слова, выводит их и выделяет квадратными скобками первую замененную букву. Решив, понял, что наговнокодил

<?php
error_reporting(-1);
$text = "МУУ ГООУО ОПП по Удмуртинской Области объявляет конкурс на поставку кaнцелярских тoваров на сумму 100500р.";
$regexp = "/([а-яё])([a-z])([а-яё\s.])([a-z])([а-яё]*)/ui";
$match = [];
if (preg_match($regexp,$text,$match)){
echo "Опечатка с слове: {$match[1]}{$match[2]}{$match[3]}: к[а]нцелярских";
}

скажите,что надо исправить, что бы работало правильно(лол). Сформулировал, конечно, еще как,но думаю, все поняли, о чем я прошу:3
Аноним 21/04/17 Птн 22:56:33  977512
Можно ли проверять или получать залогиненного пользователя прямо по среди шаблона, при условии если используется менеджер завимостей? Например в шапке? Или всегда нужно делать это в контроллере?
Аноним 21/04/17 Птн 23:56:50  977535
Мне нужно в phpunit сделать для одного теста очень много входных данных через dataProvider (допустим, 6 000). Но я не хочу иметь 6 000 точек. Можно ли как-то указать PHPUnit считать 20 наборов входных данных за одну точку?
Аноним 22/04/17 Суб 01:46:47  977575
>>977535

Если данных так много, то возможно их лучше поместить в отдельный csv файл. Его в том числе можно редактировать в excel/openoffice.

> Можно ли как-то указать PHPUnit считать 20 наборов входных данных за одну точку?
Тогда надо группировать данные и подавать на вход теста массив из 20 элементов.

>>977512

Можно. Но лучше наверно передать в шаблон самого пользователя, чем сервис, проверяющий залогиненность. Часто бывает такое, что какая-то переменная выводится в шапке и нужна почти на всех страницах, и тебе надо обеспечить передачу таких переменных.
Аноним 22/04/17 Суб 02:38:43  977580
>>977575
> подавать на вход теста массив из 20 элементов
Это изврат, но мне пришлось так и сделать
Аноним 22/04/17 Суб 11:11:08  977633
Вопрос по формам в Симфони.

Использую в форме такое описание

->add('categories', 'collection', [
'type' => 'app_executor_category',
'allow_add' => true,
'allow_delete' => true,
'by_reference' => false,
])

app_executor_type:

$builder
->add('categoryCode', 'text', [
'property_path' => 'category',
]);

$builder->get('categoryCode')->addViewTransformer(
// трансформер поиска категории по коду
);

Схема app_executor_category - id, ex_id, category_id.

Суть проблемы. При передачи в форму меньшего кол-ва элементов categories чем было в предыдущем сохранении - получал записи в БД c null. Добавил 'allow_delete' => true, эта проблема ушла.

Осталась следующая: если в следующем сохранении в массиве элементы поменяются местами
categories[0][categoryCode]=sports&categories[1][categoryCode]=food
в
categories[0][categoryCode]=food&categories[1][categoryCode]=sports

получаю ошибку duplicate entry из sql, потому что в categories пападают записи под существующими id, но в первой лежит id второй категории, а во второй - первой, и когда они сохраняются - при любом раскладе будет дублированная запись в бд.

Аноним 22/04/17 Суб 13:31:30  977716
>>977180
Вот еще Yoda Speak - https://3v4l.org/nGbTu
Аноним 22/04/17 Суб 14:38:27  977748
14927606109240.jpg (31Кб, 480x480)
вопрос ОПу: я вот решил задачку Сдача, http://ideone.com/n03Gaa
обязательно ли усложнять задачу, добавляя купюры 200 и 2000 и добиваясь минимума выдаваемых купюр или можно дальше продолжать решать задачки. просто перешел по ссылке по решению рюкзаков и монет и испугался словосочетания "реккурентная формула"
Аноним 22/04/17 Суб 15:47:18  977802
Мне нужна Ваша помощь. Вылетает такая ошибка при восстановлении бэкапа 1с-битрикс. Менял эту хуйню в php.ini, но она не сохраняется. Чё делать?

Сайт работал в кодировке UTF-8. Конфигурация сервера не соответствует требованиям.
Для продолжения установите настройки PHP: mbstring.func_overload=2 и mbstring.internal_encoding=UTF-8.
Аноним 22/04/17 Суб 15:55:01  977810
Как заставить себя пройти регулярные выражение? Уже 3 месяца как забил:с
Аноним 22/04/17 Суб 17:49:31  977862
>>977802

Что значит "не сохраняется"? Ты отредактировал файл, открыл заново и там нет твоих изменений? У тебя может прав нужных нет?

После редактирования php.ini надо перезапустить apache или php-fpm, в зависимости от того, что используется.

Проверить конфигурацию можно с помощью phpinfo()

Ну и конечно func_overload - это максимальный быдлокодинг (проблема в том, что с ним часть функций начинают поддерживать utf-8, а часть - нет, и очень легко запутаться). К тому же в документации написано, что эта опция устарела (ну наконец-то): http://php.net/manual/ru/mbstring.overload.php

>>977810

Не знаю, я же не психолог. Но могу сказать, что если ты не можешь освоить регулярки, то ты в более сложных темах дальше вряд ли разберешься.

А что именно тебе кажется сложным? Регулярка это ведь всего лишь шаблон для поиска текста, где можно использовать специальные символы вроде "любая буква".

Если ты читаешь наш учебник, попробуй погуглить и почитать другие статьи. Ключевые слова - "регулярные выражения pcre", "регулярные выражения в php"
Аноним 22/04/17 Суб 19:17:22  977933
лиля.jpeg (6Кб, 236x213)
>>976099
>>mb-функций нет на ideone, есть на 3v4l.org

не понимаю и туда запилил Лиличку https://3v4l.org/h3E9h, тоже только знаки вопроса вылазят. Допоможить!
Аноним 22/04/17 Суб 19:19:28  977935
>>977933

https://github.com/codedokode/pasta/blob/master/php/strings-utf8.md

ты почему-то решил что $string[0] вернет первую букву строки. Но это не так, $string[0] это только первый байт из строки, а буква состоит из нескольких байт и при попытке его отобразить получается вопросик.

Используй mb_substr.
Аноним 22/04/17 Суб 19:28:54  977943
Снимок.PNG (484Кб, 1253x590)
>>974941
>можно ли такое вообще сделать.
Всё ограничивается только твоей фантазией.
Аноним 22/04/17 Суб 20:38:56  977980
>>977935
спасибо, понял промах
Аноним 22/04/17 Суб 21:08:32  977989
>>977935
исправил
https://3v4l.org/630Ol
Аноним 22/04/17 Суб 21:08:42  977990
>>977493

Тут надо искать слова, содержащие буквы из разных алфавитов. То есть слова, где рядом есть буквы из 2 разных алфавитов. Получается такое выражение:

(любые буквы, 1 русская буква, 1 латинская буква, любые буквы)
или
(любые буквы, 1 латинская буква, 1 русская буква, любые буквы)

У тебя примерно то же по сути, но добавлено лишнее и не рассматривается вариант, когда слово состоит из латинских букв и содержит одну русскую.


Аноним 22/04/17 Суб 21:28:23  978004
>>977990
спасибо :3
Аноним 22/04/17 Суб 23:16:37  978077
>>977535
Если у тебя 6 тыс наборов данных, то ты что-то делаешь не так. Обычно все граничные случаи покрываются максимум 10 наборами.
Аноним 22/04/17 Суб 23:27:31  978084
Ребят, в командной строке вылезает
Invalid request (Unexpected EOF). Как это фиксить и чем чревато?
Аноним 23/04/17 Вск 03:12:11  978207
>>978077
там пермутации же. Мне нужно покрыть все варианты входных данных.
Допустим, первый параметр может быть в четырёх состояниях, второй — тоже, это уже 16, и так далее. Какой конкретно сбойнёт никто не скажет. Тесты для этого и нужны, чтобы все случаи покрыть. Если тесты пишутся под конкретную ошибку имлементации, возникшую в прошлый раз, это плохие тесты. Потому что после рефакторинга они пропустят новую ошибку
Аноним 23/04/17 Вск 08:35:08  978228
Всем привет. Ребят, посоветуйте пожалуйста какие-нибудь мануалы, где я буду по очереди писать гостевуху, борду, ещё что-нибудь такое. Просто плохо учусь на абстрактных примерах, нужна практика.
Синтаксис в целом знаю, учебник опа проходил несколько лет назад, недавно освежил свои знания на кодакадемии.
Аноним 23/04/17 Вск 09:39:22  978236
>>978228
Писать гостевуху и борду достаточно глупые вещи. Делай задачи из ОП поста, сначала список студентов, потом файлообменник.
Аноним 23/04/17 Вск 09:58:11  978237
>>978236
Борда и гостевуха более простые задачи.
Аноним 23/04/17 Вск 10:54:02  978248
поясните про yii2, его вне снг вообще используют? и как сильно он востребован в россии?
Аноним 23/04/17 Вск 14:45:51  978417
ок2.jpg (216Кб, 600x627)
Привет, ОП. Ты тот кто мне нужен.
Что есть: Я одминю юнкисы лет пять, не вылажу из консоли, хорошо умею в баш, умею в питон, понимаю как работает лапм, могу поднять и сконфижить мускуль с апачем. Фронтенд тоже в целом представляю как работает.
Чего нет: Я совершенно не работал с ПХП. Я только в теории понимаю ООП и СКЛ, опыт работы с ними эпизодический.
Чего надо: Поддерживать и в идеале отрефакторить местную самописную систему учёта. Поднять уровень пхп, рассмотреть сценарии применения.
Я пролистал по диагонали Трахтенберга, но без практики всё вылетает из головы. Хочу пробежаться по кодакадеми, или аналогу, чтобы руки набить, прежде чем к реальному коду прикасаться. Или сразу за студентов приниматься?
Аноним 23/04/17 Вск 18:25:23  978588
Где нубу почитать как работает веб-сервер?
Чтоб и не нудное чтиво на полгода, но и знания базовые получить
Аноним 23/04/17 Вск 18:50:30  978610
>>978588
В ОП-посте есть ссылка, нужно было его читать перед тем как задавать вопрос.

https://github.com/codedokode/pasta/blob/master/soft/web-server.md
Аноним 23/04/17 Вск 22:44:19  978759
14494316878270.png (206Кб, 500x415)
Имею БД students, хочу создать в ней следующую таблицу:
CREATE TABLE student
( name VARCHAR(30),
surname VARCHAR(30),
gender VARCHAR(30),
group VARCHAR(5),
mail VARCHAR(20),
score SMALLINT UNSIGHNED,
yearofbirth YEAR,
placeofbirth VARCHAR(10),
password VARCHAR(40));

и вылезает вот это:
ERROR 1064 (42000): You have an error in your SQL syntax; check the manual that
corresponds to your MySQL server version for the right syntax to use near 'group
VARCHAR(5),
mail VARCHAR(20),
score SMALLINT UNSIGHNED,
yearofbirth' at line 5

Как фиксить?
Аноним 24/04/17 Пнд 00:02:27  978818
>>978759
Ну так почитай, что в ошибке. Слово 'group' зарезервировано MySQL. Я знаю 2 способа обойти:
- назвать столбец как-то по-другому, например student_group
- поменять group на `group`
Аноним 24/04/17 Пнд 00:53:34  978868
>>978759
`group` в кавычках пиши, unsigned без h
Аноним 24/04/17 Пнд 01:28:33  978886
147656327217351[...].jpg (32Кб, 604x340)
>>978818
>>978868
Спасибо, помогло
Аноним 24/04/17 Пнд 05:21:21  978922
Скажите а можно ли в свойство класса передавать сразу какой нибудь обьект, без конструкторов и методов? У меня при этом действии вываливается все с непонятной ошибкой
Аноним 24/04/17 Пнд 11:55:02  979026
>>978922
Можно, конечно.
Конкретный код покажи, чтобы понять, в чем у тебя ошибка.
Аноним 24/04/17 Пнд 12:03:50  979027
Ребят, а расскажите, что вы используете в качестве песочницы для пхп, чтобы делиться кодом? Во фронт-енд треде все юзают кодпен, а вы?
Аноним 24/04/17 Пнд 14:19:18  979099
>>979027
https://3v4l.org/
Аноним 24/04/17 Пнд 14:56:42  979133
>>979099
Какое-то немодное и некрасивое название. Ребята из опенспейса не поймут(
Аноним 24/04/17 Пнд 15:00:14  979137
Выделение009.png (69Кб, 449x228)
кто обучался по данному курсу? интересует ваше мнение. мне пока нравиться очень, все рассказывают подробно, систематизированно. Из недостатков пожалуй два заметил:
1. Уроки по 20-50 минут в среднем, многое затянуто, по самому ларавелю смотрю первую часть, но пока ее досмотришь, дипломный проект сделаешь к пенсии. Думаю начать сразу с практической части.
2. В некоторых моментах особенно ларавел коверкается произношение заморского. Ретурн, фолсе и много еще замечательного. Не граммарнацик, но все же.

гений торрент преступлений
Аноним 24/04/17 Пнд 18:04:46  979230
Мне нужно передать аяксом файл и ещё одну переменную в php-скрипт. Файл "собираю" через такую конструкцию:
var data1 = new FormData();
$.each( files, function( key, value ){
data1.append( key, value );
});


И отправляю с параметрами:
data: data1,
dataType: 'json',


А принимаю с
if( isset( $_GET['uploadfiles'] ) ){
и так далее. Так вот, нифига не догоняю, как послать две переменных: файл и строку. Обычно я использую на php-стороне такой код для разбора:
$value = array ();
$i = 0;
while(list ($key, $val) = each ($_POST)){
$value[$i] = $val;
$i = $i + 1;
}
$date1 = $value[0];

но тут он не прокатывает.
Как это можно разрулить?
Аноним 24/04/17 Пнд 22:48:08  979382
>>979026
class Student{
public $data = new StudentTableGateway;

}

Ошибка такая:

Parse error: syntax error, unexpected 'new' (T_NEW)
Аноним 24/04/17 Пнд 23:26:31  979414
2016-10-20-1714[...].png (47Кб, 896x433)
>>979382
Так нельзя. Вообще, зависимости нужно не хардкодить, а передевать сверху, в качестве аргумента конструктора: https://github.com/codedokode/pasta/blob/master/arch/di.md
IDE умеют генерировать поля на основе переданных аргументов, если тебя не устраивает многословность PHP.
Алсо зачем в сущности объект TableGateway, это ты ActiveRecord и TableGateway смешать в кучу пытаешься?

>>978248
На реддите о нём почти никто ничего не пишет, на апворке тоже мало предложений по Yii.
Аноним 24/04/17 Пнд 23:31:40  979418
На сайте есть несколько форм, данные из каждой добавляются в разные таблицы БД. Думаю для простоты сделать один файл в котором будут обрабатываться все формы, а чтобы определить какую форму стоит обрабатывать, добавить в каждую форму скрытое поле. Так правильно делать?
Аноним 24/04/17 Пнд 23:37:44  979432
Этого достаточно для защиты от sql-инъекций?

$name = trim(htmlspecialchars($_POST["name"]));

какие нужно ещё использовать функции?
Аноним 25/04/17 Втр 00:10:24  979455
>>979414
>Алсо зачем в сущности объект TableGateway
Я хотел реализовать паттерн TDG, но как использовать его с самой моделью я не понял, видимо мне Dependency Injection нужно курить
Аноним 25/04/17 Втр 01:03:54  979491
Regex101 требует только регулярное выражение. Куда мне тогда скрипт писать про номера?
Аноним 25/04/17 Втр 01:33:58  979501
Почему на regexp101 ограничители красным подчёркивает? Как он вообще работает?
Аноним 25/04/17 Втр 02:18:44  979508
>>979432

То, что ты написал, вообще никакого отношения к SQL-инъекциям не имеет. Тебе надо сначала разобраться, что это такое и почему возникает уязвимость. Сейчас же ты просто поставил какие-то функции наугад, не думая, что они делают. Так не годится, начни с урока: https://github.com/codedokode/pasta/blob/master/security/sql-injection.md а потом еще почитай мануал по использованным тобой функциям.

>>979418

Не стоит. У каждой формы скорее всего свои правила обработки данных и общий код для них написать не получится. Алсо почитай про MVC и про работу с формами

https://github.com/codedokode/pasta/blob/master/forms.md
https://github.com/codedokode/pasta/blob/master/arch/mvc.md

>>979414

Это за рубежом, а в русскоязычных странах он довольно популярен.

>>979382

В PHP в качестве значения поля по умолчанию нельзя указывать что угодно, а только константные значения (числа, строки, массивы, другие константы и тд).

>>979230

А ты понимаешь, что делает каждая строчка в твоем коде? Тебе надо не искать готовое решение, а изучить то, что ты используешь. То есть для начала прочти мануал по:

- FormData
- jquery.ajax()
- мануал PHP про загрузку файлов http://php.net/manual/ru/features.file-upload.php

А так, если ты не представляешь, как этот код работает, как ты его дальше развивать будешь?

>>979133

это значит eval
Аноним 25/04/17 Втр 02:19:02  979509
>>978417

В ОП посте есть учебник, полистай его, и найди те задачи, которые ты с ходу не можешь решить, и решай их. Там же есть глава по ООП.

Также, желательно прочесть мануал https://secure.php.net/manual/ru/langref.php

Далее, решай студентов из ОП поста, к ним много подробных комментариев.

Кодеакадеми - там очень базовый, ознакомительный уровень, его хватит для написания скрипта на 20 строк, но не для большой системы. И это же относится ко многим книгам "для начинающих".

>>978237

Примерно того же уровня что и студенты. Тоже простая система с парой таблиц в БД и формами.

>>978207

Не, на практике покрыть все случаи обычно нереально. Потому тестируют отдельные кейсы, вроде "если пользователь ввел неправильный email, форма должна отобразитоь ошибку".

Тесты обычно пишут по требованиям, про ТЗ. Если там написано "пользователь должен заполнить имя, email и придумать пароль для регистрации", то мы из этого делаем позитивные/негативные сценарии (заполнил, не заполнил, заполнил неправильно) и тестируем.

Полностью покрыть все комбинации входных данных обычно нереально, или неоправданно долго/дорого. Тесты не гарантируют что все 100% корректно, их задача - отлавливать случайные ошибки в добросовестно написанном коде.

>>978084

В веб-сервере для командной строки? Скорее всего никак не надо, это просто браузер отменил запрос и разорвал соединение.
Аноним 25/04/17 Втр 02:19:14  979510
Получаю из нескольких полей формы данные в массиве, теперь мне нужно добавить этот массив в базу данных, причём каждое значение массива в отдельную строку. Это можно как-то сделать не прокручивая весь массив в цикле, после каждой итерации выполняя занесение данных в базу?
Аноним 25/04/17 Втр 02:19:20  979511
>>977989

> $biggest=mb_strlen(max($text_arr));
Неправильно. max() выстраивает строки по возрастанию байт, которыми они кодируются, и берет ту, в которой первый байт максимален. Соответственно для строк: ['abram', 'z'], max() вернет 'z' потому что в utf-8 код для 'z' больше чем для 'a'.

Если интересно, вот правильное сравнение строк по алфавиту, но тут оно не нужно: https://github.com/codedokode/pasta/blob/master/php/collation.md

> $text_arr
Плохое название, не надо в переменной писать ее тип, и не надо использовать подчеркивание. Правильнее назвать $lines (строки).

В остальном верно.

>>977748

Не то чтобы обязательно, но на этой задаче можно научиться реализовывать алгоритмы на языке программирования. Советовал бы выделить время и помучаться.

"рекуррентная" - значит что это последовательность, где следующий член последовательности выражается через предыдущий (или предыдущие). https://ru.wikipedia.org/wiki/%D0%A0%D0%B5%D0%BA%D1%83%D1%80%D1%80%D0%B5%D0%BD%D1%82%D0%BD%D0%B0%D1%8F_%D1%84%D0%BE%D1%80%D0%BC%D1%83%D0%BB%D0%B0

Ну например рекуррентная формула для последовательности натуральных чисел 1, 2, 3...:

x[1] = 1; // последовательность начинается с единицы
x[n] = x[n - 1] + 1; // следующий член на 1 больше предыдущего

В программе у тебя есть проблема: слишком большие блоки в if и к тому же они еще и сильно вложены. Лучше сделать так:

if ($total_sum < $amount){
echo "Выдача невозможна: недостаточно средств в терминале\n";
exit();
}

....

> $quantity_5000=
> $quantity_1000=
> $quantity_500
Это копипаста, так не пойдет, надо обойтись без нее. Нельзя просто бездумно копировать код, надо подумать, как избавиться от повторов. Хотелось бы чтобы номиналы купюр задавались в массиве в начале программы и их можно было бы менять, не трогая код. А то если у нас будет 10 номиналов купюр, ты 10 блоков кода что ли будешь копипастить?
Аноним 25/04/17 Втр 02:19:47  979512
>>977716

ок, верно.

>>977633

А что у тебя там за связи? Многие-ко-многим между categories и executor? Ты не пробовал сделать как тут описано ( http://symfony.com/doc/2.7/form/form_collections.html )? В частности, посмотри пункт "Doctrine: Ensuring the database persistence" в конце.

Также, в Доктрине прописаны связи между category и executor как многие-ко-многим?

>>977180

> Анончики как в регулярках в утверждениях указать чтобы проверялась наличие не только одной точки/знак вопроса/восклицательный знака, но и все возможные варианты, вот такие например .../??/!!!!
Использовать второе утверждение, проверяющее что дальше идет не знак препинания:

(?<=[.,])(?![.,]) - соответствует позиции после точкой или запятой, за которой нет другой точки или запятой.

Решено верно.

>>977068

Можно включать, можно не включать.

>>977026

А зачем нужен хеш? Чтобы доказать, что пользователь действительно получил ссылку, а не пытается подтвердить несуществующий ящик, мы генерируем сложный случайный код (не хеш, с чего ты решил, что это результат хеш-функции?), и просим пользователя его предъявить как подтверждение владения почтовым ящиком. Злоумышленник не может угадать код.
Аноним 25/04/17 Втр 02:20:17  979513
>>976909

Вряд ли, хотя может быть для какой-то части сайтов получится написать алгоритм.

>>976826

Чтобы отделить выражение от остальной части строки: http://php.net/manual/ru/language.types.string.php#language.types.string.parsing

Они не обязательны.

>>976751

Нет, я пример кода писать за тебя не хочу.

>>974013

Не делай кеширование, если у тебя товары обновляются часто. Посылай новый запрос при каждой введенной букве (с небольшой задержкой, если быстро печатают).

> Т.е., допустим, по фокусу на инпут идёт один запрос на сервер
На фокус ничего делать не требуется, только при нажатии клавиш.

> откуда в потоке приходят записи и колбэком заносятся в общий селект.
Не надо изобретать велосипед, уже есть селекты на JS которые работают с аякс-запросами: select2, chosen.

>>973867

Если это объект из DOM или SimpleXmlElement, то там есть методы, поищи в мануале.

>>973811

В базе ты хранишь путь вместе с датой. А расположение папки download задаешь в конфиге или в коде.

>>973499

В CSS есть анимация и transition (переходы).
Аноним 25/04/17 Втр 02:20:46  979514
>>972880

Хороший вопрос про https://www.owasp.org/index.php/AJAX_Security_Cheat_Sheet#Always_return_JSON_with_an_Object_on_the_outside

Сории, что сразу не ответил. Здесь речь об уязвимости которая позволяет обойти ограничения на получение данных с другого домена. Допустим есть домен good.example.com, на котором залогинен пользователь, и на нем есть URL, который отдает важную информацию о пользователе в формате JSON: good.example.com/userinfo

Получить информацию можно только из браузера пользователя, так как для доступа к ней нужна авторизация через куки.

Злоумышленник владеет сайтом на домене example.evil и хотел бы при заходе пользователя получить эту информацию с good.example.com. Просто так отправить аякс-запрос со страницы он не может, так как браузер не позволяет получить ответ с другого домена. Отправить запрос он может (как GET, так и POST - через подключение картинки или отправку формы), а прочесть ответ - нет.

Однако, злоумышленник может подключить JS скрипт с другого домена на своем сайте, при этом браузер отправит запрос и выполнит содержимое ответа как JS-код. То есть злоумышленник пишет у себя на сайте

<script src="//good.example.com/userinfo"></script>

При этом браузер отправит запрос, получит JSON-ответ такого вида:

['secret_info', 'secret_info2', ...]

и попытается его выполнить как JS-код. Так как JSON почти является подмножеством JS, то JSON почти всегда является валидным JS-кодом. И не вызовет ошибки. Но и данные злоумышленник, казалось бы, не получит.

Однако, тут есть лазейка. В JS массив - это объект с конструктором window.Array. Когда ты пишешь var x = [1, 2];, в реальности происходит вызов конструктора как new Array(1, 2). Злоумышленник может перед подключением скрипта заменить конструктор на свою функцию, которая перехватит данные:

window.Array = function(data) { ... };

И браузер при выполнени JSON-кода, содержащего массив, вызовет функцию злоумышленника и передаст в нее содержимое массива. Таким образом, злоумышленник обойдет ограничения на доступ к данным с другого домена и похитит данные пользователя. Там вполне могут быть например какие-то персональные данные, или данные, нужные для входа на сайт. Ну или, злоумышленник может деанонимизировать пользователя по его логину на другом сайте.

В качестве меры защиты раньше предлагалось вставлять в начало ответа синтаксически некорретный кусок кода, чтобы он вызвал ошибку при попытке выполнить его как яваскрипт. Но тогда ответ конечно перестанет быть JSON-ом.

Вот пример, когда эта уязвимость была в gmail: http://blog.jeremiahgrossman.com/2006/01/advanced-web-attack-techniques-using.html и позволяла узнать имя и email пользователя.

Вот еще по теме:

- http://stackoverflow.com/questions/3146798/why-do-people-put-code-like-throw-1-dont-be-evil-and-for-in-front-of
- http://stackoverflow.com/questions/3503102/what-are-top-level-json-arrays-and-why-are-they-a-security-risk
- http://haacked.com/archive/2008/11/20/anatomy-of-a-subtle-json-vulnerability.aspx/
- http://www.ninoishere.com/json-hijacking/
- http://ejohn.org/blog/re-securing-json/
- https://bugzilla.mozilla.org/show_bug.cgi?id=376957

Использование объекта вместо массива { ... } делает JSON не валидным JS кодом.

В Хроме сейчас создание массива через литерал [1, 2] не приводит к вызову подмененного конструктора, если его переопределить.

Стандарт ES5: http://es5.javascript.ru/x11.html#x11.1.4

> 1. Пусть array("массив") будет результатом создания нового объекта, как если бы этот объект был создан выражением new Array(), где Array является стандартным встроенным конструктором с этим именем.

То же самое там написано и про литерал объекта - вызывается встроенный конструктор Object.

Вообще, первопричина такой уязвимости - это возможность обходить кросс-доменные ограничения через подключение скриптов. Если тебе интересно, есть еще такие лазейки, существующие с давних времен:

- подключение iframe с другого домена (тут по идее разграничение должно работать, но кто знает что там можно найти)
- подключение картинок, стилей, скриптов с другого домена
- отправка форм и XHR на другой домен

Для других типов, веб-шрифтов, например, выдается ошибка при попытке подключить шрифты с другого домена.
Аноним 25/04/17 Втр 02:21:16  979515
>>967259

Не знаю, может и не подойдут если рассчитаны на конкретную версию API.

>>967048
>>967050

> function inclineWord($number, $type) {
Тут идея с массивом не очень удачная, так как у тебя получилась фукнция, умеющая склонять только 3 слова, лучше было просто передавать в функцию возможные формы слов, а она выберет нужную. А так ты зачем-то туда еще и точку засунул. Какое отношение точка имеет к выбору формы слова? Это не задача этой функции, ставить точки.

Также, у тебя переменная $type привязана по смыслу к функции numberToText, что тоже плохо, лучше если функции независимы друг от друга, а не спутаны вместе.

> $words[0] =
Вместо явного указания индекса лучше использовать $words[] = ...

> $numbers = array();
> $numbers[0] = floor($number / 1000000);
> $numbers[1] = floor(($number % 1000000)/1000);
> $numbers[2] = $number % 1000;

Можно еще так писать:

$numbers = [
floor($number / 1000000),
floor(($number % 1000000)/1000),
...
];

> function numberToText($number) {
> if ($number <> 0){
Тут if слишком огромный и занимает почти всю функцию, надо поменять в нем условие на противоположное.

> $finalArray[$i]
Тут лучше просто [] вместо [$i]

> function getFinalWord
Название плохое, лучше getWordFormNumber

> function checking($number){
Название неправильное, непонятно что значит.

> function spellSmallNumber($number){
название не соответствует содержимому


>>966920

Слишком много кода для простой задачи. Например это повторяется 2 раза: $credit*$percent+$service

> $total=$total+$pay;
используй тут +=

> if($credit+$service<$pay) {
Тут не учтен процент от долга.
Аноним 25/04/17 Втр 02:21:34  979516
>>979455

При использовании TDG класс Student не знает о TDG, а вот TDG знает про Student и создает его при надобности.

Ты не путаешь модель студента и класс StudentTDG? Это 2 разных класса, первый представляет информацию об одном студенте, второй - умеет работать с базой данных.

>>979491

Ты тестируешь регулярку на regex101, и когда она будет работать верно, переносишь ее в скрипт и тестируешь все еще раз.

Ты не обязан использовать regex101, но по моему на нем отлаживать регулярку быстрее, чем каждый раз править и запускать код.

>>979501

Ограничители там уже стоят и их писать не надо.

Вот пример выражения: https://regex101.com/r/jmRKQL/1/

Вверху пишешь регулярку, внизу текст и он подсвечивает все фрагменты текста, соответствующие регулярке.

Добавь флаг m если хочешь, чтобы ^ соответствовал началу каждой строки, а не только началу текста.
Аноним 25/04/17 Втр 02:22:40  979517
Кого я пропустил - напомните о себе

>>979510

Есть синтаксис INSERT в MySQL (нестандартный) для вставки нескольких строк, погугли.

Аноним 25/04/17 Втр 03:02:27  979524
>>979516
>а вот TDG знает про Student и создает его при надобности
То есть модель передаёт только нужные свойства и в ней хранятся только они? А если я захочу написать какой нибудь метод не связанный с работой в бд, мне его в TDG делать? Ведь я думал что TDG на то и нужен чтобы не мешать работу бд с другими методами, которые будут в модели.
Запутался я вобщем
Аноним 25/04/17 Втр 03:39:29  979525
>>979524

Для чего по твоему нужен TDG?
Что такое по твоему модель? Для чего она нужна?

Мне кажется ты что-то перепутал из-за непонимания этих терминов.
Аноним 25/04/17 Втр 04:07:51  979526
Пробел в регулярных выражениях как будет? Пробелом и будет, вроде [0-9 ]?
Аноним 25/04/17 Втр 06:49:20  979539
>>979526
>Пробел в регулярных выражениях как будет? Пробелом и будет
Да, но ещё есть \s, который найдет любой пробельный символ (пробел, табуляцию, перенос на новую строку и т.д.).

> [0-9 ]
Это найдет либо цифры, либо пробел.



Аноним 25/04/17 Втр 06:58:11  979545
>>979512
>А зачем нужен хеш? Чтобы доказать, что пользователь действительно получил ссылку, а не пытается подтвердить несуществующий ящик, мы генерируем сложный случайный код (не хеш, с чего ты решил, что это результат хеш-функции?), и просим пользователя его предъявить как подтверждение владения почтовым ящиком. Злоумышленник не может угадать код.
Было бы удобно не создавать новых полей или таблиц которые содержат этот код, а просто отправить хеш тот самый который получаем из пароля.
Аноним 25/04/17 Втр 09:38:33  979602
>>979508
>А так, если ты не представляешь, как этот код работает, как ты его дальше развивать будешь?
Методом велосипедов и костылей!
Окей, почитаю. Правда, я туповатый и мало что понимаю в прочитанном.
Аноним 25/04/17 Втр 10:47:16  979628
>>979602
Я не догоняю.
На js стороне мы создаём форму и добавляем в неё данные из файла. Можно заодно добавить что-нибудь вроде data1.append("key", "652");
>Объекты FormData позволяют вам легко конструировать наборы пар ключ-значение, представляющие поля формы и их значения
Окей, отправляем. Но php-скрипт же ведёт себя так, будто просто получил файл. И я не понимаю, как в нём получить отправленную информацию, кроме той, которая в $_FILES.

То есть, по отдельности всё клёво: вот мы создаём объект из ключей-значений и отправляем его. Дальше магия - и вот мы получаем файл, как если бы отправили его тупо кнопкой, а не аяксом. И вот вам пять переменных, типа $_FILES['userfile']['name'], с ними и работайте.
Аноним 25/04/17 Втр 12:40:11  979726
>>979628

В отладчике в браузере Ctrl + Shift + I на вкладке Network посмотри какой POST запрос отправляется и что в нем передается , передается ли параметр, какие заголовки Conent-Type/Content-Length?
Аноним 25/04/17 Втр 14:20:01  979803
Можно ли в функцию заносить массив, вроде function Func($array) {
}?
Аноним 25/04/17 Втр 15:27:57  979863
>>979726
Request Payload
------WebKitFormBoundaryN6lXi0VCJBkSobQb
Content-Disposition: form-data; name="0"; filename="del_object_v1.png"
Content-Type: image/png


------WebKitFormBoundaryN6lXi0VCJBkSobQb
Content-Disposition: form-data; name="key"

652
------WebKitFormBoundaryN6lXi0VCJBkSobQb--

Это? Вижу, что параметр key передаётся. Но как его принять на стороне php-скрипта? Я пробовал пару вариантов, но они не сработали.
Аноним 25/04/17 Втр 18:09:46  979970
>>979863

Можешь еще скинуть кусочек кода, HTML, JS, чтобы я мог у себя запустить и проверить?

Просто пока мне кажется, что тут все в порядке и данные должны приходить в массив $_POST.
Аноним 25/04/17 Втр 19:47:22  980017
Разъясните мне тупому как все таки работают замыкания в ЖС. Вот решение первой задачки по жс в учебнике ОПа:
"use strict";
var sequense = function (start, step) {
var n = start;
return function(){
return n = n + step;
}
}
var generator = sequense(4, 1);
console.log(generator());
console.log(generator());
Работает вроде как надо. Только вот я все равно не понял, почему значение n сохраняется раз и навсегда, а не перезаписывается каждый раз при обращению к generator.
Аноним 25/04/17 Втр 20:18:17  980038
>>980017
Алсо ОП, поправь все таки условия в задачнике, т.к. в твоей тестирующей проге добавлены еще условия. Алсо2, на мой код пишет такое:
Функция sequence(startingValue, step)
encountered a declaration exception
Что бы это значило? Сам код:
function sequense(start = 0, step = 1) {
var flag = false;
return function () {
if (flag) {
return start = start + step;
} else {
flag = true;
return start;
}

};
}
var generator = sequense();
console.log(generator());
console.log(generator());
console.log(generator());

Аноним 25/04/17 Втр 22:45:47  980166
Почему у меня + никак не воспринимает?
Аноним 25/04/17 Втр 22:58:47  980178
>>980166
АА, блджад, теперь он 8 не видит. Ну что это?
Аноним 25/04/17 Втр 23:30:18  980190
Что такое подмаска?
Аноним 25/04/17 Втр 23:44:02  980195
>>980017
Прочувствуй разницу:
>return n = n + step;
и
>return n + step;

n сохраняется для функции sequence, а не для generator, но увеличивается, так как на него есть ссылка. Получается, ты передаешь start и step при вызове generator, но используется только step (и n берется из функции).
Аноним 25/04/17 Втр 23:50:49  980202
>>980178
Всё он видит. У тебя какое условие? 8 и десять цифр от 0 до 9. А скармливаешь "8-495-1-234-567", оно не подходит под условие!

В общем, если человеческим языком перевести:
покажи "+7" или "8 и десять цифр в промежутке от 0 до 9"
Аноним 26/04/17 Срд 00:21:14  980230
>>980195
Все равно туплю. По моей логике, в generator() у меня лежит ф-я sequence. И значит при вызове фи генератор я по ссылке вызываю фю секвенс с параметрами. Так вот, почему сохраняется состояние переменной n от прошлого вызова, а не каждый раз по новой устанавливаются?
Аноним 26/04/17 Срд 00:21:57  980231
>>980202
То есть нужно для семи так де писать? То есть 7[0-9]{10}|8[0-9]{10}?
Аноним 26/04/17 Срд 00:37:49  980243
>>980231
Подумай, как включить туда ещё пробелы, скобки и дефисы
>>980230
Тут-то и есть эта тонкая грань. Когда ты пишешь так
>var generator = sequense();
ты запускаешь функцию sequense,
но когда ты пишешь дальше
>generator();
это равносильно вот такому:

>var n = 4; --> сохранилась внутри sequence
>generator(); --> n = 4 + 1 = 5;
>generator(); --> n = 5 + 1 = 6;
>...
>function generator(n, step) {
>return n = n + step;
>}

То есть n передается из функции sequence в функцию, которая вернулась в generator, как будто ты еще находишься внутри sequence. Блджад, это реально трудно объяснить... Captain OP, to the rescue!
Аноним 26/04/17 Срд 00:45:57  980248
>>980243
Ошибся:
>function generator(start, step) {
>return n = n + step;
>}
n конечно же передается сверху сверху, а step из аргументов.
Аноним 26/04/17 Срд 01:04:55  980254
>>980017

Это называется замыкание (closure) - когда к функции в момент ее создания привязываются переменные снаружи ее. И она может позже обращаться к этим переменным.

https://learn.javascript.ru/functions-closures
https://learn.javascript.ru/closures

Когда ты вызываешь функцию, то создаются объявленные внутри нее через var переменные. Когда ты выходишь из этой функции, они уничтожаются (так как к ним больше никто не может обратиться и незачем их хранить в памяти).

При этом, если функцию вызвать несколько раз, то каждый раз при входе в функцию создается новая копия переменной:

function test() { var n = 1; }
test(); // входим в функцию, создаем переменную n, выходим, переменная уничтожается
test(); // то же самое, создается новая копия n и уничтожается
test(); // еще раз то же самое

Но когда тело функции обращается к переменным, созданным снаружи функции, получается более интересная штука - замыкание. В момент создания функция сохраняет ссылку на переменную и может потом к ней обращаться:

function createFn() {
var n = 1;
var fn = function () { n++; return n; }
return fn;
}


var f1 = createFn();
// входим в функцию createFn, создаем переменную n (назовем ее n_1)
// создаем новую функцию, сохраняем ее в переменную fn и в момент создания функция привязывается (запоминает ссылку на копию переменной) к этой копии переменной n_1
// выходим из функции, переменная n_1 не уничтожается, так как ссылка на нее сохранена в fn и к ней могут быть обращения
// новая созданная функция сохраняется в f1


var f2 = createFn();
// входим в функцию createFn, создаем второй экземпляр переменной n (n_2)
// создаем новую функцию, функция привязывается к этому экземпляру переменной n_2
// выходим из функции, переменная n_2 не уничтожается так как ссылка на него есть в функции
// сохраняем созданную функцию в f2

console.log(f1());
// вызываем f1, она увеличивает n_1 на 1 и возвращает ее (получается 2)

console.log(f2());
// вызываем f2, она увеличивает n_2 на 1 и возвращает ее (тоже 2)
console.log(f1()); // выводит 3
console.log(f1()); // выводит 4
console.log(f2()); // выводит 3

Если повторить:

- локальные переменные, созданные внутри функции, доступны только внутри нее (в том числе в создаваемых внутри нее функциях)
- локальные переменные удаляются, как только они становятся недоступными, исчезают все ссылки на них (в обычном случае - при выходе из функции)
- функция при создании сохраняет ссылки на внешние переменные, которые в ней использованы. Эти переменные не удаляются, так как она удерживает ссылки на них. Функция с привязанными к ней внешними переменными называется замыканием.

Вот еще пример кода с замыканием. Попробуй определить, что выведет программа?

function createTwoFunctions() {
var n = 1;
var f1 = function () { n++; return n; }
var f2 = function () { n++; return n; }
return [f1, f2];
}

var fns = createTwoFunctions();
var f1 = fns[0];
var f2 = fns[1];
console.log(f1()); // что выведется?
console.log(f1());
console.log(f2());
console.log(f1());

Аноним 26/04/17 Срд 01:16:22  980258
>>980038

Скорее всего ты используешь синтаксис значений аргументов по умолчанию, который введен в ES6 и который не поддерживает твой браузер.

>>980178

Плюс это спецсимвол и надо писать \+ или [+] чтобы он не имел специального значения.

Попробуй расшифровать свое выражение:

-- первый вариант --

[+] = должен встретиться знак плюс
[[\s]? = затем может быть, а может и не быть (так как знак вопроса), один из символов в квадратных скобках, то есть скобка [ или пробельный символ \s
+ = предыдущее выражение встречается 1 или более раз подряд
[7] = далее должна идти цифра 7

-- или второй вариант --

8[0-9]{10} = должна идти цифра 8, за ней ровно 10 цифр

-- конец --

То есть твое выражение ищет либо кусок вроде +7 либо номер из 11 цифр подряд, где первая = 8

Было бы полезно, если бы ты словами написал что ты имел в виду, и привел бы выражение, а мы бы сказали правильно ты его написал или нет. Попробуй сначала именно словами сформулировать, что в каком порядке должно идти.

Аноним 26/04/17 Срд 01:24:35  980260
>>980190

Регулярное выражение еще называют "шаблоном", совпадение с которым надо найти, или "маской" (которая в данном случае значит "шаблон"). "Подмаска" - это часть регулярки, заключенная в круглые скобки.

>>980230

Ты немного неправильно рассуждаешь.

"в generator() у меня лежит ф-я sequence". Неправильно. В переменную generator ты поместил анонимную (без имени) функцию, которую создала и вернула функция sequence(). sequence() при каждом вызове создает и возвращает новую функцию.

А когда ты пишешь generator() со скобками, то ты делаешь вызов функции, ссылка на которую была сохранена в переменную.

В JS функции - это объекты и ссылки на них можно сохранять в переменные, передавать и возвращать из функций.

Если бы ты написал

var generator = sequence;

То тогда бы ты положил в переменную generator ссылку на функцию sequence. но у тебя написано sequence(), а круглые скобки обозначают вызов функции.

Вот еще пример интересного выражения:

var x = fn(1)(2)(3);

Ну-ка, расшифруй, что это значит? Как это выполняется? Допиши функцию fn так, чтобы в x оказалась сумма значений в скобках (1 + 2 + 3 = 6).

Аноним 26/04/17 Срд 01:26:41  980261
>>980038

Флаг тут не требуется. Проще либо уменьшить start заранее на 1 шаг либо сохранять его перед увеличением.
Аноним 26/04/17 Срд 01:30:06  980262
>>980260

И еще кое-что. Ты наверно заметил, что я стараюсь писать не "в переменную мы поместили функцию", а "в переменную мы поместили ссылку на функцию". Почему? Потому что функции - это объекты и они не копируются при присваивании, копируется ссылка на тот же самый объект:

var a = {}; // создается новый объект, ссылка помещается в a

var b = a; // не создается вторая копия объекта, просто в b помещается ссылка на тот же объект, на который ссылается a

a.x = 1; // добавляем поле в объект
console.log(b.x); // выведет 1

То же самое с функциями:

var a = function () {};
var b = a;

Здесь создается одна функция (один объект) и обе переменные на этот один объект ссылаются.

Это по моему написано у меня в предисловии к задачам, советую тоже почитать.

Аноним 26/04/17 Срд 03:46:31  980278
https://github.com/codedokode/pasta/blob/master/db/databases.md#Задачка-про-кинотеатр/

https://pastebin.com/Fm1f7srw
Тут глючит: http://sqlfiddle.com/#!9/3935de/9

В 4-й части (где группировка по 09-15, 15-18 и т.д) не додумался, как выводить сам временной промежуток. Есть ещё идея добавлять union на каждый временной промежуток:
select '9-15', count ...
union
seect '15-18', count ...
...

Но нельзя ли одним запросом?

>>976689
Вопросов по замечаниям нет, спасибо за проверку, сажусь за макет.
пэхэпэфаг 26/04/17 Срд 04:00:21  980280
15y8v50Zp9o.jpg (4Кб, 219x88)
сап 2щ. накидайте инфы по sapi плес, касательно пхп. в чем профитосы юзать тот или иной сабж. в частности mod_php vs cgi vs fastcgi vs php-fpm. особо никогда не интересовался детально этим. работает и збс. но щяс встал (вопрос) быстродействия сайта. if кто-то располагает годным соусом что почитать на эту тему, подкиньте - любые статьи, маны, псевдокниги сойдут. ну или так вкратце накидайте +/- че к чему.
Аноним 26/04/17 Срд 10:35:13  980362
>>980280
Бери php-fpm, никаких vs тут уже не надо обсуждать в 2017.
Аноним 26/04/17 Срд 10:46:47  980368
>>980254
Подожди. Т.е. по факту, это ф-я внутри ф-и заставляет сохранять значение переменной, так?
Аноним 26/04/17 Срд 11:37:19  980386
>>980278
Переделал задачу с промежутками и исправил ошибки: https://pastebin.com/1C9451Si
Но есть проблема - если за определённый промежуток (допустим 15-18) не было ни одного сеанса за всё время, то строка '15-18'|0|0 не выведется.
Аноним 26/04/17 Срд 13:00:01  980423
>>980368
Не сказал бы, что функция заставляет. С каждой функцией связаны LexicalEnvironment - набор её переменных, грубо говоря и Scope - свойство, указывающее не внешнее окружение. Если функция не находит в своём LexicalEnvironment какой-то переменной, то она лезет на уровень выше через свойство Scope и ищёт переменную во внешнем LexicalEnvironment. Вот здесь подробней и правильней описано: https://learn.javascript.ru/closure

>>980278
>>980386
Ещё раз переделал, с учётом сеансов без купленных билетов: https://pastebin.com/Ym8kuDTv
Аноним 26/04/17 Срд 15:10:58  980510
>>979970
Действительно, через $_POST['key'] сё нормально передаётся. То ли я ступил, то ли что-то было в другом месте неправильно, когда проверял.
Аноним 26/04/17 Срд 15:18:56  980512
>>979803
Можно.
Аноним 26/04/17 Срд 16:09:17  980535
Опчик проверь задачу числа прописью - https://3v4l.org/psi7R
Аноним 26/04/17 Срд 16:18:15  980537
>>980535
Ой там в 93 строке написано "0" заместно "Ноль", фикс карочь
Аноним 26/04/17 Срд 19:03:26  980629
Как лучше организовать проверку введенного пользователем? Просто if-ами, сделать специальный класс с методами, или отдельные функции? Что делать, если данные не верны (empty, другого типа чем ожидалось, слишком длинное/короткое, не входит в требуемый диапазон и т.п.): откатывать с ошибкой, или пытаться исправить/привести к должному виду и показать пользователю? Стоит ли использовать assertion? Нужно подходить к каждому конкретному случаю индивидуально в текущем контексте, или стараться унифицировать это все? Приветствуются мнения как опытных так и задумывавшихся над этим вопросом новичков (я сам новичок, а тема как мне кажется важная).
Аноним 26/04/17 Срд 20:25:30  980673
>>979137
Где такой курс, скинь ссылку.
Аноним 26/04/17 Срд 21:45:32  980731
14913462746802.gif (1770Кб, 450x260)
>>980673
http://therutor.org/torrent/537606/frejmvork-laravel-s-nulja-do-profi-videokurs-2016-pcrec/

если в гугле забанили
Аноним 26/04/17 Срд 22:26:08  980772
Не, ну ребята это какой то пиздец. Всю жизнь почти на линуксе сидел. Под него кодил. Под ним кодил. Тут в одном из проектов понадобилось тянуть данные с Microsoft Analysys Services(OLAP Cube)
Проект написан на PHP и подключится к этой всей байде через PHP нету возможности. Было решено написать программу на C# которая бы отдавала данные, все это отлично работает, если запускать напрямую из консоли. Но когда PHP пытается вытащить данные, запустив консольный скрипт и получив результат, то ничего в ответ не приходит. Ну что за блять гемморой с виндой?Кто знает в чем может быть причина?Под винду вообще не писал ничего и с C# первый раз вообще работаю.
Уже даже не знаю что делать.
Аноним 26/04/17 Срд 22:43:59  980787
Можно ли в регулярных выражениях внутри скобок писать ещё одни скобки? [0-9[(\s-)+]]. Работать такое будет?
Аноним 26/04/17 Срд 23:53:38  980830
Безымянный.png (24Кб, 760x312)
Можно ли сделать, дабы он считал исключительно цифры? И почему выделяется после номеров?
Аноним 26/04/17 Срд 23:56:12  980832
Безымянный.png (23Кб, 772x325)
>>980830
А с этими он не работает.
Аноним 27/04/17 Чтв 00:12:31  980846
Безымянный.png (41Кб, 840x359)
А вот тут они и вовсе не работают.
Аноним 27/04/17 Чтв 00:54:51  980856
https://ideone.com/FmOH9F
Я вот сделал, но он мне выдаёт месиво. Это так должно быть или я накосячил?
Аноним 27/04/17 Чтв 01:27:49  980866
4.png (6Кб, 380x169)
5.png (21Кб, 869x257)
Почему не работает xpath?
Аноним 27/04/17 Чтв 10:11:45  980909
>>980629
> Просто if-ами, сделать специальный класс с методами, или отдельные функции?
Я в студентах просто сделал класс валидатор с одним методом, а там через метод проверял ифами поля объекта-студента, точнее с помощью регулярок. Ошибки записывал в массив errors, который и возвращал. Хотя логичней было бы возвращать исключения, наверное.
Аноним 27/04/17 Чтв 10:38:12  980915
>>979137
Смотерл по Yii2. Хер его знает, меня удивило что автор закрывает скрипты тегом ?> - это же ужас. Хотя в остальном вроде норм, но не могу сказать насколько его практики в целом полезны.
Аноним 27/04/17 Чтв 10:47:43  980920
largep1704lrlau[...].jpg (87Кб, 625x454)
>>980772
>Проект написан на PHP и подключится к этой всей байде через PHP нету возможности.
?
Аноним 27/04/17 Чтв 14:40:32  980992
>>979545

Ну так этот хеш может сгенерировать и злоумышленник, так как он знает свой пароль.

>>980038

А какое именно условие надо поправить?

>>980278

> join seans s2 on s2.start > s.start
Тут должно быть >=

> список фильмов, для каждого указано общее число посетителей за все время, среднее число зрителей за сеанс и общая сумма сбора по каждому, отсортированные по убыванию прибыли.
А любопытно, для подведения итога тут не подойдет GROUP BY WITH ROLLUP вместо UNION?

> число посетителей и кассовые сборы, сгруппированные по времени начала фильма:
Тут можно сделать так:

SELECT CASE
WHEN HOUR(start) BETWEEN 9 AND 14 THEN '9-15'
WHEN HOUR(start) BETWEEN 15 AND 17 THEN '15-18'
END CASE as group,
...
GROUP BY group

Также, можно заджойниться на созданную на лету таблицу:

SELECT ...
FROM seans
JOIN (
SELECT '9:00' AS start, '15:00' AS end
UNION
SELECT '15:00' AS start, '18:00' AS end
) AS periods ON seans.start >= period.start AND seans.start < period.end
...
GROUP BY periods.start

Так, в общем, верно решено. Только названия стоит получше придумывать и ключевые слова писать большими буквами.
Аноним 27/04/17 Чтв 14:41:03  980993
>>980280

Надо понимать, что эти SAPI - это способы организовать взаимодействие между веб-сервером и PHP, который может работать как внутри веб-сервера так и как отдельный процесс. В обоих случаях нужно иметь какой-то протокол для передачи запроса от сервера к PHP и результата его обработки от PHP к серверу. Сам PHP взаимодействует только с веб-сервером по одному из описанных ниже протоколов, а веб-сервер уже взаимодействует с клиентом (браузером) по HTTP.

Эти протоколы позволяют обрабатывать запросы не только с помощью PHP, но и с помощью программы на любом другом языке. При этом программе не надо уметь работать с HTTP - это ответственность веб-сервера.

Первые веб-серверы могли отдавать только статические страницы (то есть файлы с диска), и CGI был простым способом добавить динамические возможности (добавление комментариев, учет числа просмотров страницы).

CGI - самый древний и медленный, но самый простой интерфейс. При поступлении запроса веб-сервер запускает новый процесс PHP, передавая информаицю о запросе (заголовки, URL) в переменных окружения. В PHP она будет доступна в массиве $_SERVER ( http://php.net/manual/ru/reserved.variables.server.php ). PHP выводит ответ (например HTML код) на стандартный вывод (а веб-сервер пересылает его в браузер), ошибки выводятся в стандартный поток ошибок (веб-сервер сохраняет их в свой лог). После обработки одного запроса программа завершается.

В случае поступления POST запроса тело запроса передается программе на стандартный поток ввода.

Он очень простой, так как обработчик легко написать на любом языке, хоть bash-скриптом. Также его поддержка не требует сложного кода в веб-сервере. Но он очень медленный, так как на каждый запрос интерпретатор PHP запускается с нуля, инициализируется.

- про CGI http://students.uni-vologda.ac.ru/pages/pm97/cgi/cgi.html - в статье есть пример программы в 7 строк, показывающей, как просто использовать CGI.

- Про переменные окружения: http://www.linux-ink.ru/static/Docs/Courses/adv-user-guide/adv-user-guide/ch02s02.html
- про потоки ввода/вывода: http://xgu.ru/wiki/%D0%A1%D1%82%D0%B0%D0%BD%D0%B4%D0%B0%D1%80%D1%82%D0%BD%D1%8B%D0%B5_%D0%BF%D0%BE%D1%82%D0%BE%D0%BA%D0%B8_%D0%B2%D0%B2%D0%BE%D0%B4%D0%B0/%D0%B2%D1%8B%D0%B2%D0%BE%D0%B4%D0%B0

Далее, mod_php - это модуль, который позволяет встроить интерпретатор PHP внутрь Апача. Соответственно теперь он запускается вместе с сервером, и работает намного быстрее. Это не универсальный интерфейс, как CGI, который позволяет использовать любой веб-сервер и любой язык программирования.

Из минусов - поддерживается только Апач и PHP (nginx - уже нет), также, если у тебя много сайтов, и хочется чтобы каждый из них работал от отдельного пользователя, с отдельными ограничениями ресурсов, то реализовать это затруднительно. Если PHP вдруг упадет, он утянет за собой процесс веб-сервера, в котором выполняется.

Далее, FastCGI - это специальный протокол для передачи запросов от веб-сервера к программе, например, интерпретатору PHP. При его использовании интепретатор PHP запскается как отдельная программа, открывает порт и принимает запросы от веб-сервера по протоколу FastCGI, обрабатывает их (запуская PHP-программу, которую указал сервер) и постылает ответы обратно. Опять же, перезапускать на каждый запрос PHP не надо, потому работает быстро. Также, можно для каждого сайта запустить свой процесс PHP, от своего пользователя, со своими ограничениями. Благодаря тому, что PHP работает в отдельном процессе, от отдельного пользователя, при проблемах он упадет один, не задевая веб-сервер.

Он быстрый, но потреял простоту, которая была у CGI и с ним скрипт на bash из нескольких строчек использовать не получится.

http://lectureswww.readthedocs.io/5.web.server/fcgi.html

php-fpm - это менеджер процессов PHP, работающих по протоколу FastCGI. Выше я описал протокол, но чтобы его использовать, надо как-то запускать процессы PHP, поддерживать определенное их количество, перезапускать, если они падают, логгировать ошибки. Всем этим и занимается php-fpm. Мануал: http://php.net/manual/ru/install.fpm.php

http://xandeadx.ru/blog/php/866

Также, может тебе будет полезно почитать про архитектуру серверов вообще: https://gist.github.com/codedokode/ffd520440a970c07c1c6

>>980362

Глупый совет. Обсуждать как раз надо, чтобы понимать в каких случаях что лучше подходит. А ты подаешь все так, как будто есть всегда плохой и всегда хороший вариант, непонятно правда почему об этом до 2017 года не знали.
Аноним 27/04/17 Чтв 14:41:32  980994
>>980386

Тогда джойнить надо на созданную из SELECT/UNION таблицу.

>>980423

Да, правильно.

>>980535

> if (($isFemale == 1) || ($isFemale == 2)) {
Тут странно, я думал что $isFemale может быть только 0 или 1. Нелогично как-то.

> inclineWord($millions % 10,
Непонятно, почему так. Это же неправильно наверно.

В остальном верно.


>>980629

Урок по формам https://github.com/codedokode/pasta/blob/master/forms.md

Валидация делается функцией, подумай, что подается ей на вход и что она вернет на выходе.

> Просто if-ами, сделать специальный класс с методами, или отдельные функции?
как удобнее. Главное, чтобы была функция/метод, которую можно вызвать.

> Что делать, если данные не верны (empty, другого типа чем ожидалось, слишком длинное/короткое, не входит в требуемый диапазон и т.п.)
Вернуть сообщение об ошибке

> или пытаться исправить/привести к должному виду
Можно и так, но тогда надо предусмотреть возврат исправленных значений и информирование о произведенных исправлениях.

> Стоит ли использовать assertion?
Нет, assert() используется чтобы сказать "это утверждение всегда верно, иначе в программе ошибка и надо ее завершить"

> Нужно подходить к каждому конкретному случаю индивидуально в текущем контексте, или стараться унифицировать это все?
В фреймворках это унифицируют, вот Symfony Validation, если не боишься:

http://symfony.com/doc/current/components/validator.html

>>980787

Нет, ты не понимаешь смысл квадратных скобок. Они значат "ровно один любой из перечисленных символов". Соответственно круглые скобки внутри будут восприниматься просто как символы скобок. Также как и звездочка, вопрос или плюс. Мануал http://php.net/manual/ru/regexp.reference.character-classes.php

>>980830

Выделяется потому, что регулярка выделяет не только номер, но и символ перевода строки перед ним. Добавление ^$ и флага m должно помочь.

> Можно ли сделать, дабы он считал исключительно цифры?

да, напиши "1 цифра, за ней любые символы", возьми это в скобки и повтори 10 раз.

[0-9(\W)+] - тут скобки и плюс обозначают соотв. символы, и не имеют специального значения. Мануал http://php.net/manual/ru/regexp.reference.character-classes.php

>>980846

тут слишком длинное выражение

Вместо +? надо писать звездочку.

Также, у тебя ^ относится только к первой альтернативе, и $ ко второй.

^ab|cd$ читается как (^ab)|(cd$) а не ^(ab|cd)$

>>980856

Регулярка слишком длинная, вывод программы невозможно читать. Надо его привести в понятный человеку вид.

>>980866

В loadHTML указывается не доменное имя, а строка с HTML кодом. Открой мануал по loadHtml.
Аноним 27/04/17 Чтв 14:42:56  980996
Если я вас пропустил - напомните о себе

Тот, кто не напомнит, умрет через 3 дня.
Аноним 27/04/17 Чтв 14:43:19  980998
>>980994
>В loadHTML
Там изначально был loadHTMLfile, просто на скрине проебался. Все равно не работает.
Аноним 27/04/17 Чтв 14:49:13  981001
>>980998

В loadHtmlFile наверно указывается иям файла, а не доменное имя узла в сети.
Аноним 27/04/17 Чтв 14:50:38  981002
7.png (4Кб, 394x51)
>>981001
Ну зачем ты лечишь, я взял пример отсюда http://stackoverflow.com/questions/13718500/using-xpath-with-php-to-parse-html/13718745#13718745

Аноним 27/04/17 Чтв 14:51:44  981003
>>980993
Ты сам все подал так, как оно и есть. Кроме php-fpm сейчас нет нормальных решений. Можно, конечно, работать в ивент лупах, много подобного есть, но это уже другое архитектурное решение вообще. Никто уже не работает ни с cgi, ни с mod_php, потому что апач дружит с php-fpm. Поговорить о них можно, конечно, и узнать всякое интересное, но выбора уже не стоит, ибо во всех случаях лучше всего фпм и подходит. Может только есть свои реализации php-fpm у кого, суть однако той же остается.
Аноним 27/04/17 Чтв 14:56:26  981006
>>981002

Там указан URL, а не доменное имя. Плюс, я не вижу там никакой обработки ошибок.

>>981003

А mod_php чем плох? Не надо держать и настраивать отдельного демона php-fpm.

Аноним 27/04/17 Чтв 14:59:38  981007
>>981006
>Там указан URL, а не доменное имя
О чем ты вообще, он должен брать содержимое любой ссылки. Кроме того, и там, и там URL.
Аноним 27/04/17 Чтв 15:44:08  981024
>>981006
Тащемта речь про задел на хайлоад. В отрыве от этого и mod_php и php_fpm однохуйственны практически.
Так в целом хорошо сконфигурированный апач с модпхп и так же хорошо сконфигурированный энжинкс с пхпфпм покажут сопоставимые результаты, разница будет в копейках.
Жопа приходит в следующих случаях:
Абсолютно нормальная ситуация для почти всех веб сервисов - отдача статики. Апач в этом месте начинает срать под себя. Это изи обходится подключением энжинкса для конкретно этой цели. Но уже тут возникает вопрос, если нет разницы нахуя мутить переусложненную схему?
Это еще не все. Апач при попытке масштабирования начинает уже просто водить тебе хуем по губам со словами "php мой, мая прелесть, саси". В итоге модпхпшный боярин снова идет на поклон к энжинксу, чтобы тот хоть как-то развел по углам ебанутых апачей. Но вот это уже оказывается попросту существенно медленее простого энжинкса и кучки фпм, так еще и переусложнено на кой то хуй. Снова.
Выбиратьть апач с модпхп в дейтсвительности нужно только в том случае, если позарез нужны его расширения, в иных случаях он ничем не выигрывает.
Аноним 27/04/17 Чтв 16:08:10  981044
>>981024
Напомнило это все отзывы от рубимакак, которые базарят за скорсть RoR.
Руби на Рельсах обычно стоят со своим интерпретатором, там все свое, интерпретатор и фреймворк, тогда как РНР стоит обычно на кучу сайтов да еще в связке с апачем.
Аноним 27/04/17 Чтв 16:44:49  981063
а можно ли как-нибудь вызвать функцию mb_Strtoupper в ideone? Просто при запуске вываливается фаталочка: call to undefined function...
Аноним 27/04/17 Чтв 17:59:50  981086
>>980992
Да, WITH ROLLUP подошёл и помог избавиться от лишнего запроса с джойном:
http://sqlfiddle.com/#!9/3935de
https://pastebin.com/BFvfR3U8

Правда в мануале написано, что ORDER BY и WITH ROLLUP взаимно исключающие: https://dev.mysql.com/doc/refman/5.7/en/group-by-modifiers.html
Поэтому для сортировки нужно оборачивать запрос с ROLLUP в ещё один запрос вида SELECT * FROM (...) ORDER BY column
Однако такой подход будет при сортировке также учитывать строку 'Итого'.
Аноним 27/04/17 Чтв 22:07:53  981206
>>981063
Никак. Лучше скачай PHP на комп.
Аноним 28/04/17 Птн 02:09:45  981348
image.jpg (394Кб, 920x768)
Не знаю туда ли я зашел, но если есть база,например, mysql с таблицами, и нужно чтобы юзеры могли просматривать определенные строки, редактировать, добавлять новые строки в нее. Как это реализовать? В html таблицы выводить результат? Для полей нужны выпадающие списки, где-то ограничить ввод. Какие варианты есть?
Аноним 28/04/17 Птн 02:10:22  981349
>>980992
>Ну так этот хеш может сгенерировать и злоумышленник, так как он знает свой пароль.
Не может, так как он не знает соль. Но если мы позволяем не активированному пользователю логиниться и сохраняем хэш в куки, тогда злоумышленник может получить его от туда. Значит остается только вариант с созданием дополнительных таблиц/колонок?
Аноним 28/04/17 Птн 13:59:01  981471
>>981007

Нет, не любой. Ты все путаешь.

Во-первых, 2ch.hk - это не URL. Прочти мой урок про URL, это будет полезно: https://github.com/codedokode/pasta/blob/master/network/urls.md

Во-вторых, PHP не поддерживает любые URL, а поддерживает "потоки", за счет чего в некоторых функциях можно вместо имени файла указать похожий на URL идентификатор:

http://php.net/manual/ru/intro.stream.php
http://php.net/manual/ru/wrappers.php

То, что ты указал, для PHP - это просто имя файла.

>>981024

Локально у себя может быть проще поставить Апач вместо того, чтобы возиться с настройкой нгинксов. Также, у Апача больше настроек и опций конфигурации, чем у нгинкса. Есть вещи, которые нгинкс в принципе сделать не может. Хотя я не думаю, что это плохо - это все же в первую очередь веб-сервер для быстрой раздачи контента.

>>981063

На ideone не включили почему-то нужное расширение. Используй альтернативы, например 3v4l.org

>>981086

Ага, значит тут он не очень подходит. Но зато ты теперь знаешь про эту возможность.

Конечно можно было извернуться как-то так

ORDER BY IF(id = 'Итого', 1, 0) ASC, ...

но наверно это уже лишнее усложнение.

>>981348

Нужна программа на PHP, которая будет брать данные из базы и генерировать на их основе HTML код, а также обрабатывать запросы на изменение данных.

>>981349

Лучше сделать отдельное случайное число, чтобы не создавать потенциальных уязвимостей. Если тебе жалко места, можно сделать отдельную таблицу кодов подтверждений и чистить в ней старые записи.
Аноним 28/04/17 Птн 14:12:00  981482
>>981471
>Локально у себя может быть проще поставить Апач вместо того, чтобы возиться с настройкой нгинксов.
Зачем ты об этом говоришь, если изначальный вопрос только про быстродействие сайта был?
Аноним 28/04/17 Птн 17:50:07  981572
IMG8705.JPG (197Кб, 1280x552)
Кто нибудь сталкивался с подобной ошибкой? Возникает только в таблицах с json столбцом.
Аноним 29/04/17 Суб 01:08:28  981752
>>966608 (OP)
Можно передавать со страницы на страницу в скрытом поле формы id определённой записи в базе данных? Это не нарушение безопасности?
Аноним 29/04/17 Суб 01:16:32  981754
Как организовать защиту от накрутки голосования. Думал сделать таблицу в базе данных, куда заносить ip проголосовавшего и id вопроса по которому он голосовал, и потом проверять, выводить результат или же выводить форму для голосования. Но ip всегда можно изменить. Потом решил вешать куки на каждого проголосовавшего, и проверять ещё и по куки, но его тоже можно стереть. Как вообще поступить?
Аноним 29/04/17 Суб 01:19:28  981756
>>981754
Регистрация с подтверждением по паспорту в офисе Ростелеком.
Аноним 29/04/17 Суб 01:54:12  981763
>>981756
А другие варианты? Ничего кроме учёта поведенческого фактора (сколько пробыл на странице с голосованием и т.д.) в голову не приходит.
Аноним 29/04/17 Суб 09:58:38  981825
tema.png (14Кб, 1353x150)
Как работает список стран внизу страницы на сайте Тёмы? Я так понимаю, это foreach с тремя if. Анон, можешь объяснить принцип работы псевдокодом или живым примером?

http://www.tema.ru/travel/mexico/
http://www.tema.ru/travel/usa.1990-1991/
http://www.tema.ru/travel/belarus.2015-2/
Аноним 29/04/17 Суб 17:29:52  982041
ОП, как в Симфони 2.7 в формах указать для кастомного поля что оно необязательное?

Добавляю в форму
->add('radius', 'app_map_radius', [
'required' => false,
])

В кастомном типе 'app_map_radius' описываю содержимое этого поля как

$builder
->add('dist', 'text', [
'constraints' => [
new NotBlank('from'),
],
])
->add('from', 'app_latlon', [
'constraints' => [
new NotBlank('radius'),
],
]);

Все преобразуется и выдает как мне нужно, но проблема в том, что теперь он требует "from" когда radius не передаю.

Как сделать 'app_map_radius' необязательным? empty_data => null не помогает.
SQL календарь Аноним 29/04/17 Суб 18:22:43  982072
https://pastebin.com/2u1nhFnH (можно сразу вставлять запрос в консоль MySQL, sqlfiddle опять глючит)

Чувствую, что ход моих мыслей нужно объяснить.
Первым делом я заметил, что для каждого месяца нужно ровно 7 столбцов и до 6-и строк включительно: http://calendar.yuretz.ru/

На основании этого можно сгенерировать таблицу, содержащую числа от 1 до 42-х (так как 6 * 7 = 42): https://pastebin.com/uwHjYikh

Теперь осталось добавить пару условий и ещё одну переменную-счётчик - для ячеек. Этот счётчик начинает увеличиваться сразу же, а счётчик дней в месяце - только тогда, когда значение счётчика ячеек превысит @firstDayNumber. В этой переменной находится номер 1-го числа в неделе (пик). И ещё, если значение счётчика дней превысит количество дней в месяце - то добиваем оставшиеся ячейки таблицы нулями. Ну а потом оборачиваем это в ещё один SELECT, чтобы отобрать только нужные столбцы и избавиться от строки, состоящей из нулей (она может быть, а может не быть).
Аноним 29/04/17 Суб 21:41:40  982165
Скажите, php и javascript это разные языки с разными функциями или просто аналоги друг друга? То сеть, можно ли сделать на javasript то же, что и на php?
Аноним 29/04/17 Суб 21:55:37  982170
Господа, я себе всю голову выебал и изгуглился вдоль и поперек.
Помогите, пожалуйста.
Суть проблемы: у меня есть два скрипта, один из них выполняется 95-120 секунд, а второй выполняется 0.1 секунду.
Почему-то, когда я запускаю первый скрипт, и следом второй. Второй скрипт не отвечает до тех пор, пока не исполнится первый. Как это пофиксить? Чую, что где-то в конфиге апача, но повторяюсь, всю голову выебал и перегуглил, но нихуя не могу понять даже, куда копать. Халп, помогите, господа.
Аноним 29/04/17 Суб 22:20:33  982177
>>982165
жаваскрипт изначально был как дополнение для "лица" сайта. То есть он работал на стороне клиента - подвигать там формы, снежок посыпать, то есть анимация и прочее и прочее.
ПХП работал на стороне сервера.
Сейчас жаваскрипт может работать и на стороне сервера и на стороне клиента.
Все зависит от того, что хочешь делать, у каждого языка своя задача.
Аноним 29/04/17 Суб 23:15:21  982201
аноночик у меня есть такой вопрос
вот у меня есть структура сайта где начиная с index передаются параметры по нажатию ссылки а там уже инклюдится кусок кода в зависимости от нужды но в итоге это всё выглядит как index.php?id=1&sydi=to&vot_tydi=eto
как бы я подозреваю что я даун
и хотелось бы узнать как должно это работать не у макаки
даже не могу нагуглить нормально это. где искать то хотя бы?
Аноним 29/04/17 Суб 23:33:45  982210
Зачем нужен похопе, если есть божественный жс?
Аноним 30/04/17 Вск 00:06:47  982226
>>982177
>на стороне сервера и на стороне клиента.
А пхп могёт в это?
Аноним 30/04/17 Вск 00:25:48  982234
14627355808983.gif (3418Кб, 533x367)
>>982210
Да иди нахуй, жс это вообще худшее что есть в вебдеве.
Аноним 30/04/17 Вск 00:47:24  982236
База данных голосования. Есть таблица с вопросами и есть ещё одна таблица с вариантами ответов в которой каждый вариант связан с ответом которому он принадлежит через id. Есть задача вывести все вопросы на странице, при этом показывать количество проголосовавших, а также при наведении мышкой, показывать варианты ответов с результатами. Как сделать это в несколько запросов это понятно, но можно ли сделать в один? То есть, в один запрос надо получить строку из таблицы вопросов, все связанные с ним варианты ответов из таблицы вариантов ответов, а также общее число проголосовавших (в таблице вариантов ответов есть поле, в котором хранится голосовавшие за каждый вариант, нужна их сумма). Я вот не понимаю, как одним запросом подсчитать сумму проголосовавших.
Аноним 30/04/17 Вск 02:24:19  982256
Вывожу записи из БД по id, хочу проверить правильность выводимой записи, то есть чтобы в GET параметр поступал только существующий id. Этого достаточно? Или нужно что-то ещё? И нужно ли как-то чистить $_GET["id"] если я использую подготовленные запросы в PDO?

$id = $_GET["id"];

if(!is_numeric($id)) {
exit("Не существующая страница");
}

if($id<=0 || $id>=$max){
exit("Не существующая страница");
}

//$max = максимальный id в базе данных
Аноним 30/04/17 Вск 08:40:09  982278
>>982226
Нi.
Аноним 30/04/17 Вск 11:21:47  982306
>>982256

Можно просто $id = intval($_GET['id']);

> И нужно ли как-то чистить $_GET["id"] если я использую подготовленные запросы в PDO?
Нет, если ты все значения поставляешь через плейсхолдеры.

>>982236

Число проголосовавших считается через аггрегатные функции COUNT DISTINCT/GROUP BY. Ответы можно получить через GROUP_CONCAT, но там конечно свои подвохи.

Вообще, я не уверен, что тут есть смысл все делать одним запросом.

>>982201

Если это страницы сайта, то обычно используют так называемые seo friendly URL. Урок https://gist.github.com/codedokode/772a4ccc03e41d6b7cba

>>982170

Может быть много вариантов. Например, если ты используешь сессии, то там есть блокировка сессии на время использования, которая вызывает такой эффект.
Аноним 30/04/17 Вск 11:22:08  982307
>>982072

Ты тут полагаешься на использование увеличивающейся переменной, но ведь можно сделать проще и надежнее. Сгенерировать таблицу с числами от 1 до N с помощью формулы SELECT id * 7 + column:

1 2 3 4 5 6 7
8 9 10 11 ...

И затем из чисел в ней вычесть смещение, соответствующее номеру первого дня.

Тогда может быть даже не понадобится внешний запрос, достаточно будет условия WHERE firstColumn <= @daysInMonth

Все-таки эта штука со счетчиком - она полагается на строго определенный порядок увеличения счетчика, обхода строк.

>>982041

required по моему влияет только на установку атрибута html5 required на инпуте, и все (если я не путаю).

Есть такая вещь: http://symfony.com/doc/2.7/form/data_based_validation.html

И такая: http://symfony.com/doc/2.7/form/use_empty_data.html

Они не годятся?

>>981825

Там просто надо выбрать страны, сгруппированные по месяцам. И взять текущий месяц +/- 2 месяца вперед/назад.

>>981754

Никак, если приз ценнее, чем затраты на накрутку голосования, то его будут накручивать.

Разве что как-то подтвержать личность. Потому что даже номера телефонов можно скупить в большом количестве, если это стоит меньше, чем приз.

>>981752

Скорее всего нет.

>>981572

Возможно программа phpMyADmin просто не поддерживает JSON колонки? Может на их форуме или в интернете поискать эту ошибку?
Аноним 30/04/17 Вск 13:29:50  982358
>>982307
Я это понимаю, что там сортировка по дате, но как убрать лишнее и чтобы расположение было такое?

1 страница
1 новость
2 новость
3 новость
4 новость
5 новость

2 страница
1 новость
2 новость
3 новость
4 новость
5 новость

3 страница
1 новость
2 новость
3 новость
4 новость
5 новость

4 страница
2 новость
3 новость
4 новость
5 новость
6 новость

5 страница
3 новость
4 новость
5 новость
6 новость
7 новость


http://ideone.com/xDwbbM
SQL календарь Аноним 30/04/17 Вск 14:04:21  982369
>>982307
https://pastebin.com/exhuPHzp

Да, вычитание смещения это хитро, я бы не додумался до такого.

> Тогда может быть даже не понадобится внешний запрос, достаточно будет условия WHERE firstColumn <= @daysInMonth
Тут ведь firstColumn это alias? У меня в MySQL WHERE не заработал, только HAVING: http://stackoverflow.com/a/942592

И вроде WHERE firstColumn <= @daysInMonth не будет отсеивать полностью пустые строки (как на пике). Поэтому я отсеиваю через NOT (Пн = 0 AND Вск = 0)
Аноним 30/04/17 Вск 15:23:55  982403
149307790018437[...].jpg (54Кб, 500x620)
Я всё таки не до конца понять суть неймспейсов и автоподключения. Через компосер сделал автоподключение namespace Students;
И теперь вместо реквестов
require_once (__DIR__. Students\models\Auth.php');
require_once (__DIR__. Students\models\Helper.php'); Пишутся эти ЮЗИ
use FrontController\Helper;
use FrontController\Auth;
В чем сокральный смысл? Хуярить слешами НазванияПространтв/НазванияКласса тоже не круто.
Вместо Auth::login();
Писать Students/Auth::login();

Либо require, либо use, либо Students/
Разве это всё не одно и тоже? Ну за исключением того, что можно использовать одинаковые названия методов, классов(что блядь приведёт к неминуемой путанице ИМХО).
Аноним 30/04/17 Вск 15:36:30  982408
>>982403
> Либо require, либо use
Как раз таки use необязательно писать если ты запрашиваешь класс из того же неймспейса. К примеру, у тебя такая структура проекта:
src/
-- класс Foo c namespace App,
-- класс Bar c namespace App,

То теперь в файле Bar тебе не нужно писать use App\Foo, так как и Foo и Bar в одном и том же неймспейсе. А require'ы ты обязан писать всегда. Ещё в IDE или редакторах с плагинами за тебя use будет генерировать сам редактор, а require за тебе никто генерировать не будет.
Аноним 30/04/17 Вск 15:40:51  982413
Подскажите,а можно ли в цикле foreach, указывать уникальные номера переменным,как это можно с помощью обычного for?

Вроде:
for($i=0;$i<=2;$i++){
$var{$i} = $i;
}

, только с foreach
Аноним 30/04/17 Вск 15:43:07  982416
>>982408
Может просто засунуть весь проект в один namespace? Ну тогда правда всем php файлам нужно будет написать в шапке namespace ИмяПространстваИмен; И с подгрузкой файлов не придётся трахаться, всё верно? Только с левыми библиотеками придётся дёргать обратный слэш.
Или если каталог файлов такой
Project/controllers/MainController
Project/models/MainModel

И объвить неймспейс Project....то будет ли работать без USE и дроча с папками и слэшами - MainModel в MinController?
Аноним 30/04/17 Вск 15:51:53  982423
>>982226
могет, если ты скомпилируешь код в исполняемый файл и заставишь пользователя его открыть.
Рабочей средой жаваскрипта был браузер и веб-страничка. У пхп - интерпретатор, веб-сервер, куда заходят.
Сейчас Жаваскрипт можно также запускать на веб-сервере, чтобы он выполнял логику,
Аноним 30/04/17 Вск 16:30:06  982437
>>982416
> Ну тогда правда всем php файлам нужно будет написать в шапке namespace
Даже это необязательно. Попробуй положить все классы в папку src, а в composer.json пропиши автозагрузку так: https://pastebin.com/2WxRxWXB

И нет, если ты раскидаешь классы по разным папкам, но оставишь их с одним неймспейсом, то композер этого не поймёт.
Аноним 30/04/17 Вск 16:40:32  982442
ПОМОГИИИИ >>966608 (OP)
Подскажите,а можно ли в цикле foreach, указывать уникальные номера переменным,как это можно с помощью обычного for?

Вроде:
for($i=0;$i<=2;$i++){
$var{$i} = $i;
}

, только с foreach
Аноним 30/04/17 Вск 16:42:48  982444
Сап, пыхобояре, в общем пишу курсач на пыхе, до этого никогда не кодил на ней, подскажите как здесь: https://pastebin.com/fMTRiZsJ запилить, чтобы при первом клике на кнопку сортировки он сортировал по убыванию, а при втором по возрастанию. Пытался сделать булевскую переменную, проверять ее в каждой кнопке и в зависимости от значение менять в запросе ASC на DESC и наоборот, но почему-то нихуя не вышло. Короче, направьте на путь истинный, с меня как всегда
Аноним 30/04/17 Вск 16:55:26  982456
>>982444
Забыл добавить свой вариант проверки, вот он:
$check = true;
if(isset($_POST['sort_by_id']))
{
if ($check) {
$query1 = "SELECT FROM films ORDER BY Film_id ASC";
}
else {
$query1 = "SELECT
FROM films ORDER BY Film_id DESC";
}
Аноним 30/04/17 Вск 18:22:41  982505
Есть какая-то проблема у меня с терминологией: не мог вот сразу понять, что модель - это всего лишь один сраный класс, в котором свои свойства и методы. Нет, блеять, понапишут, что там бизнес-логика, хуёгика. Так сложно было написать, что это класс со свойствами-методами, а в контроллере надо просто его экземпляр создать на соответствующем экшне, чтобы при переходе на указанный URL и создавался этот экземпляр класса и отображался нужный вид?
Траханные вы в жопу программисты косноязычные твари, ненавижу вас, пидарасов.
Аноним 30/04/17 Вск 19:49:39  982580
Ебать я на винде с сокетами истрахался, открыл для себя факт, что если запустить из консоли пхп-скрипт, он обращается к другому php.ini (не тому, что указан для апача) и расширения не подтягивает, я пока разобрался, двое суток уебал на это.
Аноним 30/04/17 Вск 20:15:41  982603
>>982580
А причем тут винда?
Аноним 30/04/17 Вск 21:25:02  982643
>>982603
Притом, что в ней все через жопу.
Аноним 30/04/17 Вск 21:26:41  982647
>>982643
Анус ставишь?
Аноним 30/04/17 Вск 22:52:35  982667
>>982647
Отъебись, троляка
Аноним 30/04/17 Вск 23:17:43  982671
>>966608 (OP)
Какой вы браузер юзаете для разработки хром или лису?
Аноним 30/04/17 Вск 23:42:40  982679
>>982667
Нене, ты давай сперва объясни мне схуяли по твоему cli конфиг у пхп именно на винде богомерзкой отличается от серверного? Не можешь? Вот и нахуй иди. Это нормальная практика, а ты дебил блядь очередной.
Аноним 30/04/17 Вск 23:49:53  982684
14885498481580.jpg (101Кб, 951x657)
>>982671
bump
Аноним 30/04/17 Вск 23:55:51  982687
>>982437
>Даже это необязательно. Попробуй положить все классы в папку src, а в composer.json пропиши автозагрузку так: https://pastebin.com/2WxRxWXB
ЭМММ, создать папку scr и засунуть туда model/config/components во всех папках есть файлы с классами...
Аноним 30/04/17 Вск 23:56:47  982688
>>982505
Ето так
Аноним 30/04/17 Вск 23:57:24  982689
>>982679
В том, что винда говно, он прав однозначно. На ней танцев с бубнами гораздо больше, касаемо создания среды разработки для веба как минимум.
Аноним 01/05/17 Пнд 01:38:31  982714
>>982689
this. На рабочем компе что-то настраивается и конфигурируется с полпинка, а на шинде: тут в path добавь, сюда скачай, здесь в конфиг пропиши, аазазазаза, порт заблокирован, сладенький. Я ебал это говно. На сахарной убунточке за 20 минут настраиваю ламп и приблуды к нему, здесь заебался бубном трясти.
Аноним 01/05/17 Пнд 01:55:23  982724
А ^ надо перед слешем же ставить ^\\D ?
Аноним 01/05/17 Пнд 13:07:51  982824
>>982358
Бамп вопросу.
Аноним 01/05/17 Пнд 16:26:19  982972
>>982684
Нет РНР, ебать лохи.....
Аноним 01/05/17 Пнд 16:29:34  982974
>>982689
Apache_MySQL+PHP устанавливается минут за 5-10 не из консоли. А чтобы из консоли - надо её знать, тогда ещё меньше времени на установку.
Аноним 01/05/17 Пнд 17:42:08  983047
Оп, подскажи как мне сделать "...Чтобы пользователю при ошибке не надо было повторно вводить пароли и прикладывать файлы, нужны более сложные подходы (например сохранять отправленные файлы и пароли в какое-то временное хранилище на сервере)." из задачи про файлообменник. Интересует поле отправки файла. Как я понял после гугления, значение value нельзя ставить по соображениям безопасности. Ну допустим я где-то сохранил сырой файл который не прошел валидацию, а как его в шаблон вернуть-то. Фэйковый инпут без type='file', если файл уже на сервере, а потом вместо отправки брать уже готовый файл?
Аноним 01/05/17 Пнд 18:38:45  983088
>>982689
>>982714
В данном случае говно его руки и невежество. Он не из-за винды ебался два дня, а из-за того, что нихуя не знает про настрйоку пхп. Касается это не только апача. В итоге у него лыжи не едут и он во всем винит систему, так и не поняв в чем суть за два то дня, как удобно, ебал я таких дурачков.
Аноним 01/05/17 Пнд 20:06:44  983174
600px-125659665[...].png (551Кб, 600x600)
Рейтаните решение задачи FuzzBuzz

https://github.com/grigoryMovchan/sandbox_php/blob/master/fizzbuzz.php
Аноним 01/05/17 Пнд 20:52:18  983245
>>983174
Выдает FizzBuzz для 0 и ничего для 100 (т.е. не решено, плохо даже если пишешь ради смехохуечков)
Падает с fatal error при $firstWordTrigger = 0 (не проверяет входные параметры)
Вью не подготовлен для вызова скрипта через консоль.
Ну и $index вместо $i для итератора тоже будет вызывать бугурты.

Я так и не понял. Ты хотел максимально плохо или максимально переусложненно сделать.
Аноним 01/05/17 Пнд 21:05:37  983260
>>983174
> https://github.com/grigoryMovchan/sandbox_php/blob/master/fizzbuzz.php#L55+L63
Не понимаю, почему так много условий используется, можно же проще: https://ideone.com/prz7YP

>>983245
Двачну насчёт консоли, тоже это смутило. Классы есть, а гибкости и удобства нет.
Аноним 01/05/17 Пнд 21:24:14  983268
>>983174

Тут неправильно используются классы. К примеру, объект FizzBuzz - что он представляет? Какую сущность он моделирует?

Если ты пытался показать, что ООП для тебя выглядит сложными и запутанным, то ты недостаточно старался - есть более солидные работы, например https://github.com/EnterpriseQualityCoding/FizzBuzzEnterpriseEdition

Хочу также добавить, что люди считают ООП "переусложненным" только потому, что не понимают, что это и как правльно его использовать. Точно так же как начинающий, не осиливший функции, может удивляться, зачем разделять код на отдельные файлы и функции, когда можно просто написать все стеной вперемешку с HTML в одном файле.
Аноним 01/05/17 Пнд 21:25:27  983269
>>983174

Ну и вот статья на Хабре на похожую тему: https://habrahabr.ru/post/153225/
Аноним 01/05/17 Пнд 21:40:34  983280
>>982403
>require_once (__DIR__. Students\models\Auth.php');
> Auth::login();

и

> Students/Auth::login();

Неужели разницы не видно? Просто берешь и пишешь "Папка"/класс вместо богомерзких реквайрев.

Алсо, у тебя уже ошибка там, кавычку пропустил. Так что неймспейсы это круто, мечтаю о нормальной реализации в js.
Аноним 01/05/17 Пнд 21:47:44  983284
>>982413
>>982442
foreach лучше использовать только для перебора некоторой коллекции (например массива), чтобы применить к каждому элементу некоторые действия.

>$i = 0;
>foreach ($items as $item) {
> $var[$i] = $item;
>}
Аноним 01/05/17 Пнд 22:27:59  983299
>>982403

Читал ли ты мой урок? https://github.com/codedokode/pasta/blob/master/php/autoload.md

Неймспейсы не были придуманы ради автозагрузки. Они нужны для защиты от конфликта имен (появления 2 классов с одинаковыми названиями). Один из вариантов борьбы - делать длиннющие имена классов, другой - добавить неймспейсы, которые позволяют эти длиннющие названия записывать короче.

А потом уже оказалось, что эту особенность можно использовать для автозагрузки, поставив соответствие между неймспейсами и папками. Но такая возможность была и раньше, раньше просто подчеркивания в имени класса заменяли на слеши.
Аноним 02/05/17 Втр 00:31:17  983344
14374123505580.jpg (28Кб, 393x395)
>>983174
Я его джаст фор фан для треда в бэ сделал. Но раз оно тут вызвало такой ажиотаж, то постараюсь допилить в меру ваших замечаний.

Новый репозиторий https://github.com/grigoryMovchan/FuzzBuzz

>>983245
>Выдает FizzBuzz для 0 и ничего для 100
Падает с fatal error при $firstWordTrigger = 0 (не проверяет входные параметры)
Исправил

>Вью не подготовлен для вызова скрипта через консоль.
Никогда не запускал скрипты через консоль. Попытался запилить. Работает. Реализация хромает, но уже хочется.

>>983260
>Не понимаю, почему так много условий используется, можно же проще
ИНКАПСУЛЯЦИЯ @ ПОЛИМОРФИЗМ troll.jpg

>>983268
>Какую сущность он моделирует?
Отделил логику от представления

Аноним 02/05/17 Втр 00:32:03  983345
>>983344
>но уже хочется.
но уже хочется спать.
Аноним 02/05/17 Втр 00:42:10  983348
Что-то я торможу, как подключить библиотеку jQuery, если она используется на разных страницах, при этом сайт не как ЦМС, и не использует роутинг. Надо подключать на каждой странице по отдельности?
Аноним 02/05/17 Втр 00:47:25  983350
>>983348
переписать на шаблоны, чтобы хотя бы шапка и подвал отдельно подключались и не дублировался код из них
Аноним 02/05/17 Втр 02:04:50  983367
Есть радиокнопки

<input type="radio" class="variant" value="1">
<input type="radio" class="variant" value="2">
<input type="radio" class="variant" value="3">

Надо сделать определённое действие при выборе любой из них, как описать это на jQuery? Не могу понять как описать именно выбранную радиокнопку.
Аноним 02/05/17 Втр 07:31:00  983406
>>983088
Лол, имбецил дурной, ты думаешь разработка ведется из под маков/линухов просто так? Спермораб за свою винду топит.
Аноним 02/05/17 Втр 07:42:42  983408
>>983406
Когда в следующий раз будешь уже на линухе пытаться выполнить консольный скрипт с конфигом от сервера на него тоже лопату говна вкинь, дебич. А вообще пиздуй из профессии. Раз уж ты не можешь понять где ты обосрался тебе любая ось и окружение мешать будет.
Аноним 02/05/17 Втр 07:49:40  983410
>>983408
Я не тот человек, просто макнул тебя лицом в говно, еблан. Если ты привык жрать сперму - не хвались этим.
Аноним 02/05/17 Втр 08:09:49  983415
>>982358
БАМП.
Аноним 02/05/17 Втр 08:13:21  983416
>>983410
Винда говно, сути дела не меняет, такое поведение норма для пхп на любой системе, стажер ебаный.
Аноним 02/05/17 Втр 18:02:16  983654
А в реальных проектах обработка, валидация форм и вывод ошибок на сервере происходит, или же лучше на жс чтобы меньше нагрузки было?
Аноним 02/05/17 Втр 18:22:06  983664
>>983654
И там и там.
Аноним 02/05/17 Втр 18:56:28  983676
2249369378[1].jpg (44Кб, 373x480)
>>983367
Просто добавь айди
Аноним 03/05/17 Срд 01:17:49  983901
Безымянный.png (17Кб, 864x252)
Квадратные скобки в регулярных выражениях обозначают либо то, либо другое. Следовательно, прикл будет искать мыло состоящее только из цифр или букв? Найдёт US@email, но не найдёт US1@EMAIL?
Аноним 03/05/17 Срд 08:55:31  983986
>>983901
Не найдет, но не по-этому.
Аноним 03/05/17 Срд 09:14:12  983992
>>983986
Хули ты тут загадки загадываешь!
Аноним 03/05/17 Срд 13:30:45  984117
>>983992
Сумимасе, семпаи!

>Match a single character present in the list [a-zA-z0-9_+.-]+ between one and unlimited times

Вообще охуели не читать сперва хоть что-нибудь по теме?? Мусор ебаный.
Аноним 03/05/17 Срд 13:32:33  984119
>>983901
Найдёт и то, и то, но если только ты доменную зону допишешь: US@email.com и US1@EMAIL.COM, например. Иначе не найдёт ни того, ни другого - там стоит же точка, она обязательна.
Аноним 03/05/17 Срд 13:42:50  984125
>>984119
Помоему ты не прав, точка там в качестве символа только. Эта регулярка должна найти и два изначальных варианта, и два тобою предложенных и даже ".@.". Что в целом делает ее некорректной для работы с мылом.
Аноним 03/05/17 Срд 13:43:11  984126
>>983901

us1@email - найдет
us1@EMAIL - нет так как после @ там в регулярке стоят только маленькие буквы, и нет флага i, указывающего что регистр букв игнорируется при сравнении.

> Квадратные скобки в регулярных выражениях обозначают либо то, либо другое.
Они обозначают "один любой из указанных в квадратных скобках символов".
Аноним 03/05/17 Срд 13:54:32  984130
>>984125
Ну и да, апперкейз в домене не найдет конечно же, проглядел. Так что эти варианты не заедут.
Аноним 03/05/17 Срд 18:45:24  984247
>>984125
А, да, точно, давно не работал с регулярками, соррян.
И про i тоже не подумал.
Аноним 03/05/17 Срд 18:58:02  984254
Привет, подскажите по задаче сапер javascript.
Как сделать:
>>https://github.com/enotocode/minesweeper.mvc/blob/master/app/BrowserView.js
>>Тут у тебя жестко прописаны id и селекторы элементов, и мы не можем поместить на 1 странице несколько BrowserView (привязанные к одной или разным моделям), жаль. Ну и ссылка на document жестко прописана.

Мне видятся два варианта. Как сделать лучше, есть ли другие решения?
1. генерировать уникальный случайный айди при создании экземпляра класса BrowserView, он будет добавляться к id всех сгенерированных dom-объектов, чтобы иметь доступ к ним через getElementById.
2. хранить сгенерированные элементы внутри экземпляра класса и изменять их свойство innerHTML.
Аноним 03/05/17 Срд 20:48:02  984329
>>966608 (OP)
Сап, аноны. Я полный ноль в php, знаю только css, html и немного ruby on rails. Нужно сделать мини-приложение (есть бд в postgresql, нужно редактировать и выводить данные на страницу) с ГУИ. Подскажите самый простой и быстрый способ.
Аноним 03/05/17 Срд 21:53:04  984395
>>984254

Можно сделать так:

- убрать id из кода. Ну например, передавать в конструктор View просто нужный DOM элемент:

var div1 = ...
var view1 = new BrowserView(div1);

А если он сам его создает, то не присваивать ему id, а только сохранять ссылку на него.

Элементы внутри можно искать не по id, а например по названиям классов или дата-атрибутам. У тебя id используется для поиска ячеек, но ячейку в таблице можно искать по номеру столбца и строки как table.rows.cells[j]

Второй вариант - это из старых времен когда не было функций вроде querySelector, использовать иерархические id. То есть мы делаем у элементов id вида

<div id="view1/someElement/somePart">
<div id="view2/someElement/somePart">

который содержит в себе id родительского элемента + слеш + название подэлемента. Если слеш использовать в id нельзя, можно взять другой символ. В твоем случае можно было бы присваивать ячейкам id вроде view1/10/20.

Сейчас так не делают, проще искать элементы внутри view по имени класса или атриутам. А в твоем случае - по номеру строки/столбца.

Аноним 03/05/17 Срд 22:17:14  984406
>>984329
дай денег веб-макаке
Аноним 03/05/17 Срд 22:19:54  984409
>>984406
не дам
Аноним 03/05/17 Срд 22:37:54  984426
>>984329
Ставишь руби с рельсами, делаешь КРУД.
Всё.
Аноним 04/05/17 Чтв 02:00:19  984508
Можно ли использовать в функции __construct встроенные функции и циклы? Пишу класс для валидации формы, и хочу чтобы прямо в __construct выполнялись определённые функции вроде trim, strip_tags и т.д., цикл тоже нужен, так как приходит массив со значениями, так можно делать?
Аноним 04/05/17 Чтв 10:34:36  984588
Cj7MiDARmI.jpg (96Кб, 1600x900)
>>966608 (OP)
Привет ребятки, помогите настроить пхп шторм. Как пофиксить отступы между строками и нумерацией строк? Что это за хуита? (выделил желтым) Как убрать бары которые выделил красным и серым?
Алсо реквест годных тем для оформления, а то смотрится эта штука как-то совсем не очень.
вбросьте свои оформления

Аноним 04/05/17 Чтв 10:36:14  984590
>>984588
Левый бар убрал, осталось разобраться с правым.

Как же по уебищному тут темы смотряца, ужос.
Аноним 04/05/17 Чтв 10:40:34  984593
>>984588
С отступами тоже чек, осталось правый бар с этой полосочкой всратой убрать. И ГЛАВНОЕ ТЕМУ ТЕМУ, ЧТОБЫ БЫЛО КРАСИВО КАК В АТОМЕ ИЛИ СУБЛИМЕ!11111111
Аноним 04/05/17 Чтв 10:41:43  984594
>>984593
СУБЛИМЕ монокай тема, гугли. По дефолту в шторме вообще даркула доступна, хорошая тема, откуда ты этот вырвиглаз достал чет хз.
Аноним 04/05/17 Чтв 10:43:51  984595
Cj7MiDARmI.jpg (96Кб, 1600x900)
>>984594
Вот как выглядит даркула. Ваще капец.
Аноним 04/05/17 Чтв 10:45:10  984596
>>984595
А мне норм, не раздражающая.
Аноним 04/05/17 Чтв 10:47:16  984597
>>984596
На вот тебе монокай.
http://www.phpstorm-themes.com/theme/monokai
Аноним 04/05/17 Чтв 10:55:37  984601
>>984597
А вообще он и так доступен должен быть.
Аноним 04/05/17 Чтв 10:56:52  984603
Cj7MiDARmI.jpg (96Кб, 1600x900)
>>984597
Блин как-то плохо смотрится.
Аноним 04/05/17 Чтв 11:20:55  984611
blob (33Кб, 497x438)
>>984603
У тебя хуйня какая-то. вот дефолтный из шторма. Правда выглядит он все равно как говно.
Аноним 04/05/17 Чтв 12:45:59  984660
>>982358

Если речь о MySQL, то в 2 запроса: первым выбираешь текущую и N предыдущих записей, вторым - N следующих. Объединить их можно при желании через UNION, а можно на стороне PHP.

Если у тебя дан массив в PHP, то аналогичным образом, с помощью отсеивания и сортировок. Ну то есть оставляешь только элементы с текщей или меньшщей датой, сортируешь, берешь N, и так далее.

>>982369

> IF(id + 1 < 1 OR id + 1 > @daysInMonth
Еще можно использовать использовать (id + 1) BETWEEN 1 AND @daysInMonth

> Тут ведь firstColumn это alias? У меня в MySQL WHERE не заработал,
Тогда можно использовать id, WHERE id + 1 <= @daysInMonth или как-то так.

> И вроде WHERE firstColumn <= @daysInMonth не будет отсеивать полностью пустые строки
Мне кажется, будет, если понедельник больше, чем число дней в месяце то вторник и далее можно не проверять. Или ты про первую строку, которая может оказаться пустой? Для нее можно поставить условие вроде id + 7 >= 1 или как-то так.

Ну в любом случае, твое решение верное, думаю ты теперь хорошо разбираешься в написании сложных SQL запросов.

Дальше можешь изучать индексы, оптимизации и денормализацию. Так как на практике часто скорость выполнения "правильных" запросов недостаточная и требуется их ускорить.

>>982413
>>982442

Не надо писать $var{$i}, и вообще не надо использовать "переменные переменные", надо использовать массив в таких случаях.

>>982416

Если у тебя мало классов то можно, но когда их станет много, будет неудобно что они все в одной папке.

Про то как что работает лучше всего прочесть в офиц. мануале.

Ну и прочти мой урок про PSR-4, название controllers - не соответствует PSR-4.

>>982444

Если (сортировка не задана или равна ASC) {
сортировать по возрастанию, а в ссылке с этой колонкой выводить DESC
} иначе {
сортировать по убыванию, а в ссылке писать ASC
}

Почему для сортировки используется POST? Изучи, когда применяется POST и когда GET. передавать лучше не параметр вида sort_by_id, а 2 параметра с именем поля и направлением, или объединить их в одно поле вида +date, -date.
Аноним 04/05/17 Чтв 12:46:21  984661
>>982505

Мне кажется, ты не понял что такое модель. Потому что это не то, что ты описал. Читал мой урок про MVC? https://github.com/codedokode/pasta/blob/master/arch/mvc.md

>>982643

В Дебиане например 2 комплекта php.ini файлов (отдельно для cli и для Апача) - а в винде один. Так что спорный вопрос.

>>982724

^ это привязка к началу строки и естественно она может стоять только в начале регулярки.

>>983047

Создаешь на сервере временное хранилище файлов. При загрузке файла сохраняешь его туда. При ошибке выводишь форму, а около поля для файла выводишь информацию о сохраненном на сервере файле (можно например сохранить его идентификатор в скрытом поле, главное чтобы злоумышленник его не подобрал, чтобы он не был предсказуемым). Выводишь галочку удаления сохраненного файла, если это возможно. При желании можно сделать кастомный инпут, который будет выглядеть как будто файл уже выбран. Можно пойти дальше, выодить превьюшку для картинок и тд.

При отправке формы надо соответственно проверять:

1) приложен ли к ней файл
2) передан ли идентификатор сохраненного файла
3) выбрана ли опция удаления сохраненного файла

Это конечно все стоит абстрагировать и вынести в отдельный класс, чтобы подключение было максимально простым.

После успешной проверки формы файл перемещается из временного хранилища в постоянное.

Также, надо будет написать сборщик мусора, удаляющий невостребованные файлы после определенного срока.

Другой вариант - отправлять форму через аякс без перезагрузки страницы.

>>983344

В браузере проще ставить Content-Type чем выводить теги. Также, до сих пор нет пояснения, какую сущность моделирует класс FizzBuzz.

>>983367

Выбранное значение можно получить через $('.variant:checked').val() если я не путаю. http://api.jquery.com/val/

Что значит "как описать"? Если ты имел в виду "как повесить обработчик на выбранную кнопку" то у тебя серьезное непонимание того, как работает jQuery. Это не CSS, там нельзя писать селекторы и правила для них.

Надо повесить обработчик на родителя кнопок с фильтром через on() и в нем уже проверять, какая кнопка нажата.
Аноним 04/05/17 Чтв 12:46:47  984662
>>983654

Если не проверять на сервере, тебе в базу мусора могут напихать.

>>984508

Можно с точки зрения PHP там почти любой код писать, единственное что конструктор в отличие от обычной функции, не может ничего вернуть через return.

Но с точки зрения архитектуры может оказаться неправильно что ты что-то делаешь в конструкторе. Его задача ведь просто подготовить объект к работе и заполнить поля начальными значениями.

И я не очень понял какое отношение strip_tags или trim имеют к валидации.

Аноним 04/05/17 Чтв 12:52:01  984667
>>984662
Вообще от логики зависит. Если по логике ты принимаешь грязные значения в конструкторе, а работать хочешь уже с чистыми, то в конструкторе их обработка как раз в тему.
Аноним 04/05/17 Чтв 15:55:15  984740
>>984661
>Мне кажется, ты не понял что такое модель. Потому что это не то, что ты описал. Читал мой урок про MVC? https://github.com/codedokode/pasta/blob/master/arch/mvc.md
Блеять, вот не надо снова.
Модель - это всего лишь класс в файле, который лучше назвать по имени этого класса. В нём всё определено - свойства и методы для работы (они могут получать данные из БД или с помощью конструктора).
В контроллере прописан конкретный URL, по которому сработает нужный экшн с созданием экземпляра модели (или с чем-то другим, что связано), получатся какие-то данные из модели и отдадутся в вид.
Вот не надо меня снова путать, ОП!
Аноним 04/05/17 Чтв 15:56:58  984744
>>984740
А типа контроллер это не класс в файле? А типа %сущность нейм% это не класс в файле? И чем они тогда от модели отличаются, мм? У них у всех и поля и методы и все на свете есть или может быть.
Аноним 04/05/17 Чтв 16:14:33  984758
>>982505
Энжой ёр ООП.

>>984661
А вообще интересно, почему именно "модель"? Это же из английского пришло?

>a three-dimensional representation of a person or thing or of a proposed structure.

>a system or thing used as an example to follow or imitate.

Курица или яйцо?
Аноним 04/05/17 Чтв 16:45:08  984778
>>984744
Ну это-то да, но не путай меня!!11
Ну просто модель - это самый главный класс в файле, от него надо плясать, в нём всё содержится, а дальше работают контроллер и вид.
Просто когда с нуля идёшь и не нюхал конкретных примеров (а я долго не мог понять так, чтобы не глядя сообразить, как надо), то так проще: модель - класс, который отвечает на саму суть, контроллер - ну тоже класс, который вспомогательный и связывает с URLом конкретным, а дальше всё мы получаем в виде.
Это очевидным становится в какой-то момент, далеко не сразу, потому что многое путает, не понятно, что тут главное.
Аноним 04/05/17 Чтв 16:46:36  984782
>>984758
Да ООП-то збс, на самом деле, там всё удобно.
Аноним 04/05/17 Чтв 17:19:44  984791
>>984778
Если бы ты мог трезвой башкой оценить как ты ебануто пишешь, то не называл бы програмистишек косноязычными. Ну если тебе так проще - хуярь на основе этих выводов.
Аноним 04/05/17 Чтв 18:11:08  984817
задача.png (97Кб, 752x690)
решение.png (26Кб, 619x452)
Помоги с задачей анон. Пробовал таким образом, в итоге результат в виде ключей:
1 2 3
1 2 3
Как сделать что бы значения то были ?
Аноним 04/05/17 Чтв 18:14:55  984818
>>984817
array_rand возвращает ключ (индекс), а не значение. А что нужно написать что бы обратиться к элементу массива $word1 с индексом str1?
someApprentice 04/05/17 Чтв 18:21:17  984823
Я реализовал Sphinx, но текущий хостинг не дает опции его подключить. Поэтому, я планирую найти другой а на этом хранить только изображения и webm.

Есть какое-нибудь решение чтобы объединить два сервера и/или хостинга в один? Или обычно просто указывают абсолютный адрес изображения в шаблоне?

Как преобразовать $_GET запрос отправляемой формы из /search.php?q=... в /search/... ? Нужно разбирать ссылку в контроллере и делать ридерект?

Хотелось бы иметь собственное доменное имя. Но мне с этим не очень понятно - Провайдер обращается к корневому доменному серверу, корневой сервер перенаправляет запрос к серверу верхнего уровня, сервер верхнего уровня к авторитетному серверу. Есть какой-нибудь способ создать собственный авторитетный сервер? Я нашел много статей об этом BIND, но действительно ли методы описанные там могут позволить дать доступ к моему домену для всего интернета?
Аноним 04/05/17 Чтв 18:37:16  984825
>>984823
Используй какую-нибудь сетевую файловую систему чтобы с одного сервака обращаться к другому, где хранится говно, как к файлам на машине, а не по сетевому адресу.
Аноним 04/05/17 Чтв 18:42:47  984827
>>984818
Я хз. К тому же в эхо я вписываю $res, а в нем уже 3 массива. Как к ним ко всем то обратиться ?
Аноним 04/05/17 Чтв 19:06:14  984834
>>984827
В $res нету никаких массивов. Там только строка склеенная из трех случайных чисел которые ты получил из array_rand. попробуй вместо $str1 $word1[$str1] в 24 строке.
Аноним 04/05/17 Чтв 19:07:55  984835
>>984778
> контроллер - ну тоже класс, который вспомогательный
Вот поэтому я (не ОП) советую людям в студентах делать контроллер не классом, а обычным скриптом, чтобы лучше понимали MVC. MVC это в первую очередь о разделении ответственностей, это идея, а не строгий набор классов. Идея отвязать данные и представление друг от друга. Проблема в том, что люди не изучают ООП на нормальном уровне (разделение ответственности, связность/связанность, DI) и сразу лезут читать об MVC, бездумно принимая всё новое как догму. А ведь MVC это лишь одна из возможных реализаций принципа разделения отвественности. В React+Redux не MVC, но данные от представления тоже отделены.
Ну и вот цитата из доков Symfony: "When creating a framework, following the MVC pattern is not the right goal. The main goal should be the Separation of Concerns; this is probably the only design pattern that you should really care about."

> а я долго не мог понять так, чтобы не глядя сообразить, как надо
Нужно понимать, а не учить, тогда и подглядывать никуда не нужно будет.

Алсо в PHP-фреймворках контроллер-класс это вообще груда независимых методов, вот тут интересные мысли про ADR: https://habrahabr.ru/post/260769/
Просто для каждого экшна неудобно передавать одни и те же сервисы, вот их и группируют в классы-контроллеры.

> Просто когда с нуля идёшь
Так большинство и учатся, тебе кто-то мешает читать про SOLID, юнит-тесты?
Аноним 04/05/17 Чтв 19:31:21  984844
Я тут пытаюсь в веб, изучаю Silex. Ну и вот, хочу сделать такой поиск, по трем разным полям. Вот как тут выпадающее меню. http://bootsnipp.com/snippets/featured/search-panel-with-filters
Ну так и вот, как мне определить, какой пункт выбран?
Аноним 04/05/17 Чтв 19:54:40  984851
>>967836
ОП, скинь пожалуйста файлы (картинки, шебм) к тредам 18-59. И сохранился ли HTML 25-го треда не с архивача, а с двача? А то отдельно парсить придётся, так как у него другая разметка. Текущий парсер парсит нормально всю информацию из всех тредов, за исключением 25-го (т.к HTML с архивача) и нескольких постов из 1-го (т.к. ошибки в HTML).

>>984823
Нужно учитывать, что файлов будет где-то на 2 ГБ (треды 1-17+60-80 весят 750 мб). Кстати если не сохранять шебмки, то должно хватить 500 мб на все треды и быть может не понадобится облачное хранилище.
Аноним 04/05/17 Чтв 20:28:36  984863
>>984823
> Как преобразовать $_GET запрос отправляемой формы из /search.php?q=... в /search/... ? Нужно разбирать ссылку в контроллере и делать ридерект?
Это неэффективно. Ты можешь через JS по клику на кнопку перенаправлять браузер на /search/query/, а в случае отключенного JS, отправлять обычный GET запрос вида /serach?q=123.
someApprentice 04/05/17 Чтв 20:32:42  984865
Продолжаю разбираться с биткойнами http://phpclub.rf.gd/pr/chain/955476

Тяжело читать, переводить и, при этом ещё, запоминать документацию. Попробую восполнить пробелы спросив что мне не понятно.

Цепь блоков состоит из узлов. Когда мы устанавливаем Bitcoin Core мы создаем новый узел?

Когда мы ставим Bitcoin Core мы всегда должны иметь все блоки которые есть в сети? Сейчас их размер составляет свыше 100Gb - будет довольно дорого арендовать VPS с таким выделенным пространством.

В примерах приведенных в документации есть пример с командой sendtoaddress https://bitcoin.org/en/developer-examples#simple-spending . Там создается новый кошелек на который переводиться сумма, но не создается кошелек с которого это сумма переводиться. При установке клиента Bitcoin Core - это же клиент? даётся какой-то стандартный кошелек с которым работает эта команда?

Наверно, лучше использовать raw transaction.

>Там интересная схема. Там при создании кошелька генерируется набор случайных байт (то есть очень большое число). Это закрытый, секретный ключ, который позволяет делать любые операции с кошельком (им подписываются транзакции). Из этого ключа необратимым преобразованием генерируется открытый ключ (по моему, просто берется хеш от закрытого ключа). Это публичный идентификатор кошелька.
Мне же не обязательно хранить закрытый ключ в БД, если я хочу чтобы у каждого пользователя был собственный кошелек? Ведь его можно получить в любой момент с помощью команды dumpprivkey.

>Соответственно когда ты создаешь новую транзакцию, и подписываешь ее закрытым ключом, участники сети, каждый, проверяют, что подпись соответствует открытому ключу и таким образом удостоверяются что ты именно владелец кошелька.
В примере с simple raw transaction ( https://bitcoin.org/en/developer-examples#simple-raw-transaction ) это делать не обязательно. Получается Bitcoin Core это всего лишь кошелек, который может работать только с его собственными адресами? Или же он может самостоятельно определить закрытый ключ чтобы отличить свои адреса от остальных, если он всё-таки содержит их?


Наверно, должен быть способ чтобы перенести закрытые и открытые ключи на другой носитель. Как они хранятся?
someApprentice 04/05/17 Чтв 21:35:25  984890
>>984825
>Используй какую-нибудь сетевую файловую систему чтобы с одного сервака обращаться к другому, где хранится говно, как к файлам на машине, а не по сетевому адресу.
>сетевую файловую систему
Можно по конкретней, пожалуйста? Запрос в гугле выдал много разной информации.

>>984851
>Нужно учитывать, что файлов будет где-то на 2 ГБ (треды 1-17+60-80 весят 750 мб). Кстати если не сохранять шебмки, то должно хватить 500 мб на все треды и быть может не понадобится облачное хранилище.
Всё-таки, в тред можно отправить файлы общим размером 40Мб. Если максимализировать, то в каждом треде может быть вряд ли конечно, но, всё же, раз такая возможность не чем не запрещена, значит, это нужно соблюсти до и более 1000 постов с такими файлами.

>>984863
Так всегда делается при роутинге? Должны же фреймворки как-то обходиться без js.
Аноним 04/05/17 Чтв 21:37:53  984891
>>984890
У меня на работе вот эта используется:
https://ru.wikipedia.org/wiki/GlusterFS
Аноним 04/05/17 Чтв 22:43:22  984920
2.jpg (188Кб, 1157x1078)
>>984661
>Также, до сих пор нет пояснения, какую сущность моделирует класс FizzBuzz.
Сори, не понимаю вопрос. Ты бы как сделал? Не стал бы создавать отдельный класс?
Аноним 05/05/17 Птн 00:15:09  984949
Что то торможу, отправляю через аякс форму на другую страницу, с неё получаю нужные мне данные, и хочу разместить их в определённом диве. Данные приходят, и размещаются но сразу же пропадают, использую следующее событие (form - id формы):

$("#form_quest").submit(function(){
//Ну и здесь дальше всякое разное
});

Пробовал делать событие клик по кнопке отправки, тоже самое получается. Как сделать так, чтобы после отправки формы через аякс приходящие с другой страницы данные можно было разметить на странице и они там бы остались. Я так понял вся проблема из-за того что страница перезагружается, как это избежать?
Аноним 05/05/17 Птн 00:18:09  984954
>>983901
$emails = array(
'Test Example <test@example.com>',
'test@localhost',
'test@localhost.com'
);

foreach ($emails as $email) {
echo (filter_var($email, FILTER_VALIDATE_EMAIL)) ?
"[+] Email '$email' is valid\n" :
"[-] Email '$email' is NOT valid\n";
}

http://php.net/manual/ru/filter.filters.validate.php
Аноним 05/05/17 Птн 00:27:22  984957
>>984758

Вот определение из Википедии:

https://ru.wikipedia.org/wiki/%D0%9C%D0%BE%D0%B4%D0%B5%D0%BB%D1%8C

> Модель (фр. modèle, от лат. modulus — «мера, аналог, образец») — это система, исследование которой служит средством для получения информации о другой системе; представление некоторого реального процесса, устройства или концепции

Ну например, мы можем на уменьшенной модели самолета изучать, как его обтекает воздух в аэродинамической трубе. Есть также такое понятие, как "математическая модель", то есть система формул, которые описывают какие-то процессы. Например математическая модель, описывающая движение жидкости, может использоваться для расчета того, как будет течь и какое давление будет создавать реальная жидкость в трубах. Модель в программировании близка по сути к математической модели, только она создается с помощью языка программирования.

"модель" в программировании - это абстрактная система (например, набор переменных и функций, или объектов), которая соответствует какой-то реальной системе, которую мы хотим изучить. Ну например, если у тебя задача учитывать товары на складе, то возможно у тебя будет класс Товар, который является "моделью" реального товара.

Отсюда и идет название компонента "model" в MVC.
Аноним 05/05/17 Птн 00:32:20  984959
>>984834
Спасибо. Кстати, есть возможность это сделать короче ? А то у меня впечатление сложилось, что все можно было проще сделать, вот только хз как.
Аноним 05/05/17 Птн 01:42:54  984977
Получаю при помощи аякса данные с другой страницы, может прийти либо текст, либо число результат работы ПХП функции. Проблема в том, что тип переменной и в том и в том случае string, и взависимости от приходящего варианта нужно сделать разные действия. Как их различить? Есть ли в яваскрипте функция, которая определяет находится ли число в переменной с типом стринг? Или можно ли перевести в пхп переменную стринг в инт? или как сделать так, чтобы результат работы функции ПХП (она возвращает id), давал число а не строку?
Аноним 05/05/17 Птн 01:49:06  984979
>>984977
Я забыл один момент, на стороне ПХП перевожу нужную переменную в число, при помощи всех возможных методов intval, settype но если вывести полученные данные в аяксе то он всёравно пишет что string
Аноним 05/05/17 Птн 08:20:50  985027
questions.png (35Кб, 655x457)
Первая задачка по ООП в учебнике ОПа.
Нужно знать в правильном ли я направлении начал писать код
Аноним 05/05/17 Птн 08:22:01  985028
>>985027
Раздел "повторим" не проходил, так что сильно не обоссывайте
Аноним 05/05/17 Птн 09:32:38  985044
1.png (54Кб, 691x760)
2.png (25Кб, 239x399)
Черт с ним, лучше поясните за NULL у correctAnswers
Аноним 05/05/17 Птн 09:38:20  985046
Где новый хлеб? Сюда опчик заглядывает?
Аноним 05/05/17 Птн 12:04:10  985085
>>985044
Аномалия, ёпт, сам задумался.
Код выложи куда-нибудь, дай покумекать.
Аноним 05/05/17 Птн 12:05:13  985086
>>985027
Да правильно, там же код есть в самом уроке.
Аноним 05/05/17 Птн 13:04:54  985137
>>985027

У тебя ошибка. У тебя $questions передается как аргумент, и сразу же ты эту переменную перезаписываешь, кладя в нее пустой массив, и значит значение, которое хранилось там до этого, теряется.
Аноним 05/05/17 Птн 13:07:12  985139
>>985044
У тебя там опечатка: correctAnwer и correctAnswer. А PHP, конечно же, "молодец", просто налету создал несуществующее свойство. Это одна из причин, почему к свойствам никогда напрямую не обращаются, а используют геттеры/сеттеры, но ты пока не морочь себе этим голову. Алсо questions1 ужасное название переменной.

>>985137
Он потом исправил на questions1: >>985044
Аноним 05/05/17 Птн 13:49:19  985164
>>984979
JS:

https://learn.javascript.ru/types-conversion#численное-преобразование
Аноним 05/05/17 Птн 13:50:59  985165
>>985027
А почему тут просто функция, где классы методы? Поясните. ООП же.
Аноним 05/05/17 Птн 16:19:01  985262
>>985165
Для того, чтобы понять принцип и просто выывести результат.
Можно сделать эту функцию методом внутри класса, а вывести как Question::createQuestion([все аргументы]);
Ну и там должны быть все echo внутри самого метода тогда, хотя это не вполне будет правильно и будет громоздко.
Вроде как-то так, блё, я хз.
Аноним 05/05/17 Птн 17:39:08  985312
<?php
/ Выполнение запроса с передачей ему массива параметров /
$sth = $dbh->prepare('SELECT *
FROM fruit
ORDER BY ? DESC');
$sth->execute(array('red'));
$red = $sth->fetchAll();
Почему не работает данный пример? Всяко пытался, но все равно не сортирует. В чем может быть ошибка?
http://php.net/manual/ru/pdo.prepare.php
Аноним 05/05/17 Птн 17:47:05  985318
>>984660
>Если у тебя дан массив в PHP, то аналогичным образом, с помощью отсеивания и сортировок. Ну то есть оставляешь только элементы с текщей или меньшщей датой, сортируешь, берешь N, и так далее.
Можешь псевдокодом расписать?
someApprentice 05/05/17 Птн 18:38:52  985354
Писать чат на чистом js или на jquery?
Аноним 05/05/17 Птн 18:56:19  985359
ошибка.png (13Кб, 800x361)
Почему у меня айдеон не видит строковые операции ?
Аноним 05/05/17 Птн 19:03:52  985362
>>985359
Попробуй выполнить свой код на https://3v4l.org/ и посмотреть что за ошибка, если на айдеон не отображается.
Аноним 05/05/17 Птн 19:11:52  985365
>>985362
Спасибо, там все норм. С айдеоном не то что то.
Аноним 05/05/17 Птн 19:39:30  985378
phalcon дрочеры есть?
Аноним 05/05/17 Птн 19:41:47  985380
>>985378
есть
Аноним 05/05/17 Птн 20:01:16  985393
>>985378
На работе использую, такой себе фреймворк.
Аноним 05/05/17 Птн 20:32:10  985439
>>985354
Angular? Отлично работает с jquery и react.
Аноним 05/05/17 Птн 20:51:51  985451
>>985380
>есть
>>985393
>такой себе фреймворк

проясните в щщи, вопрос скорее общий. все время будет так всрато?
вроде все красиво, все раскидываеться по полочкам при желании, одм-орм внятная б.м. и вольт можно послать нахуй, но все равно что-то мешает просто брать и делать (не вхожу в поток в терминах смузисосеров). ну и доки всраты на 7/10 постоянно приходиться лезть в зефирный гит и искать хинты.

с учетом того что это по большей части кроновый пет-проект, a круд и вся ботва будет в общем и целом на фронте, стоит продолжать надрачивать на такуюто скорость с модельной метой и насиловать привычки или перекатиться на laravel пока не поздно?

из бекграунда yii, cake, silex, ci и крапаль зенда со вторым сифоном.

олсо 985393 - анон проясни плз за hr. как часто хайрите людей, быстро ли закрываете позицию и если берете под обучение сколько у среднего кандидата уходит на вкатывание.
Аноним 05/05/17 Птн 21:08:03  985460
>>985451
Вот это ПРОФЕССИОНАЛЬНЫЙ ЖАРГОН
Аноним 05/05/17 Птн 21:53:34  985503
>>985164
Проблема в том, что мне нужно получить число в определённом случае именно на стороне ПХП, так как если в аякс приходит число выполняется одно действие, а если строка то другое.
Аноним 05/05/17 Птн 22:00:12  985511
engineer.jpg (134Кб, 700x525)
>>985451
Аноним 05/05/17 Птн 22:08:53  985518
>>985451
> все время будет так всрато?
Зависит от задач, у нас на фальконе в основном API (рест), для того чтобы грамотно сделать архитектуру приложения под это, пришлось писать дополнительную библиотеку, которая работает поверх фалькона. Но это уже специфика, мне например очень не понравился их PHQL в целом, он ограничен и предназначен только для простых селектов. Но в работе ты привыкаешь его использовать, и потом натыкаешься на подводные камни когда какие-то SQL запросы просто отказываются работать из-за того что нет поддержки определенных команд. И приходится уже использовать Raw SQL, а там своя специфика работы с этим.

> ну и доки всраты на 7/10 постоянно приходиться лезть в зефирный гит и искать хинты
Это да, благо зефир это не си, и в основном у них простой и понятный код там.

> стоит продолжать надрачивать на такуюто скорость с модельной метой и насиловать привычки или перекатиться на laravel пока не поздно?
Про laravel ничего не скажу, с ним не работал. Если кроновый пет-проект я бы не стал надрачивать на скорость, просто бы делал на чем удобнее. С таким бэкграундом я думаю фалькон тебе легко зайдет, главное знать все нюансы, их там достаточно много.

> олсо 985393 - анон проясни плз за hr. как часто хайрите людей, быстро ли закрываете позицию и если берете под обучение сколько у среднего кандидата уходит на вкатывание
У нас на бэке раньше было два разработчика, теперь четыре, все из них с фальконом работают. Двух добрали вот недавно, по месяцу искали каждого (это джунов). Под обучение берем, в среднем у джуна уходит неделя на обучение, но сюда не только фреймворк входит, тут еще стандарты разработки внутри компании, канбан, ознакомление с стеком технологий (у нас хайлоад, используются различные сервера очереди, оперативные хранилища, кафка, кассандра и прочее) ну и с самим проектом на котором будут работать. У нас на собеседованиях довольно хороший отсев мамкиных вкатывальщиков, поэтому все кого взяли это вполне осиливали за неделю и теперь работают. Опять же тут зависит от прошлого опыта работы, если человек работал с симфони или зендом, то освоить фалькон ему не составит труда.
Аноним 05/05/17 Птн 22:22:57  985523
>>985318
У меня почти получилось! Как ограничить вывод массива?

Ща показывает все новости, типа так.

4 страница
1 новость
2 новость
3 новость
4 новость
5 новость
6 новость
7 новость
8 новость
9 новость
Аноним 05/05/17 Птн 22:30:31  985531
>>985518
>использовать Raw SQL
ога, уже наткнулся на ту же херню с простейшими джоинами. в целом мне нравится общая система, но сдаеться мне его точили на классик веб, хайлоад и сервисы уже со скрипом. похоже это важный аргумент против, спасибо, прояснил от души, добра.
Аноним 05/05/17 Птн 22:34:54  985533
>>985531
Джоины там нормально и с PHQL работают, просто нужно использовать ModelsManager, метод executeQuery. Таким образом результат уже будет не сущностью, а экземпляром класса ResultsetSimple или ResultsetComplex.
Аноним 05/05/17 Птн 22:44:10  985538
>>985503
Скажи, в каком виде ты отдаешь с сервера данные?
Аноним 05/05/17 Птн 22:58:53  985544
>>985538
С одной страницы отдаю данные полученные с формы через Аякс где они обрабатываются. После обработки, функция используемого для этого отдаёт обратно на страницу либо id добавленной записи, либо текст с описанием ошибок ввода данных.
Аноним 05/05/17 Птн 23:01:06  985545
Есть ли хорошие уроки по созданию сайтов на вордпресс? Большая часть уроков на ютубе заключается в том, чтобы установить на ворпдресс стандартную блоговую тему и поставить виджеты. Но на вордпресс делают сайты для бизнеса, с представлением услуг и т.д. и такие темы не обладают стандартной блоговой структурой. Где найти уроки, которые будут обучать создавать именно такие сайты?
Аноним 05/05/17 Птн 23:27:51  985554
>>985544
Получается ты отдаешь всегда string (даже если через echo пропускаешь int число), ajax парсит как строку. Отдавай например в формате json или xml. Тогда при парсинге ajax преобразует "42" в int 42. Или сам приводи к int. Пойми, что к числу в js можно привести число или строку, содержащую только число и пробельные символы. Полюбому у тебя получится или id или NaN, ёпте.
Аноним 05/05/17 Птн 23:33:52  985556
>>985544
>isNaN("12") \\ false, 12 это число
>isNaN("12a") \\ true, строка не имеет математического смысла
Аноним 05/05/17 Птн 23:44:52  985562
>>985533
>ModelsManager
ну да, я так и делаю, но чота оно так все больше на тупой orm на массивах из первых кейков смахивает. получается больше минусов чем плюсов в итоге. можно канешн впихнуть фабрику этого дерьма в basemodel или впилить behavior под себя но зачем весь этот корч, если есть возможность сменить на что то более подходящее под задачу. не хотеть, слишком уж геморно, дохуя усилий на инфраструктуру вместо логики. горит невероятно от гениального испанского решения пилить сверхпроизводительный и ультракастомизируемый оверхед-движок и не позаботиться о хотя бы вполовину такой же удобной orm для реальных задач а не васянских лендингов.
Аноним 05/05/17 Птн 23:57:34  985563
>>985554
>Или сам приводи к int
Как это можно сделать на стороне ПХП? На странице куда идёт запрос аякса, работает функция, которая выдаёт либо id добавленной записи, либо текст с описанием ошибок. Если получаю id, то пишу (int)id, в аякс всёравно приходит строка.
Аноним 06/05/17 Суб 00:10:04  985567
>>985563
Продолжу. Суть в том, что в правильность введения данных проверяется на стороне ПХП, в отдельном классе, но отправляю я туда данные при помощи аякса. Если данные введены не верно, то выводится сообщение с описанием ошибки.Если верно, то просто происходит переадресация при помощи яваскрипт по id добавленной записи. Я вроде бы нашёл выход из положения, сделал так:

if(msg.length<11) {
//делаем переадресацию
} else {
$("#errors").html(msg);
}

Понятно что это очень криво, но другого варианта не смог найти. Суть в том, что описание любой ошибки больше 11 символов, а у id не может иметь больше чем 11 цифр, поэтому если длина строки меньше 11, то значит это в любом случае id и надо делать переадресацию, если нет, то просто вывести описание ошибок. Главный вопрос, это слишком криво? так делать нельзя?
Аноним 06/05/17 Суб 00:18:03  985569
Слышал что если используешь подготовленные запросы PDO, то полученные данные из формы можно вообще не чистить. Это так? Если это так, то ведь тогда любой может постараться передать строку с очень большим количеством символом, ведь maxlength в форме может обойти очень просто. выходит, даже резать при помощи substr полученные данные из формы не надо при использовании подготовленных запросов ПДО?
Аноним 06/05/17 Суб 00:46:46  985571
>>985518
Расскажи пожалуйста, что должен знать/уметь нормальный кандидат в отличии от "мамкиного вкатывальщика"? Что вообще из себя представляет средний соискатель, который пролетает мимо работы?
Аноним 06/05/17 Суб 02:59:20  985618
>>985571
Удвою вопрос.
Аноним 06/05/17 Суб 08:44:09  985645
1.png (68Кб, 800x832)
2.png (66Кб, 564x906)
Со вчера не читал вас, всем кто отвечал спсибо. Сегодня утром внезапно осенило конкретно подорвал себе мозг новыми возможностями - в начальном курсе опа такому не учили, после чего сделал все как положенонаверное! Буду писать дальше
Аноним 06/05/17 Суб 08:47:36  985646
>>985645
ёбаный null, сука
Аноним 06/05/17 Суб 08:51:35  985647
>>985571
На HH.ru посмотри по своему городу/области. Я даже на свою мухосрань пару вакансий видел
Аноним 06/05/17 Суб 08:59:46  985650
Cyg68eOUcAQx0i5.jpg (31Кб, 317x372)
>>985511
Аноним 06/05/17 Суб 11:38:23  985716
>>985562
Я тоже с орм там поебался. Фалкон использую для апи. Несколько своих оберток и трейтов впиздюрил в модели, получше стало. Однако все равно иногда приходится ради хитрого джоина руками все делать. А так, конечно, ничего особо критичного, работать с ним можно.
Аноним 06/05/17 Суб 13:21:13  985770
>>984949
Интересный вопрос, меня тоже интересует. Я пришёл к выводу, что нужно использовать сессии и их глобал массив, но это блядь ведь хуйня ебанная, все высчитанные параметры на одной странице записывать в массив, что бы на второй все эти данные вытащить...и работаиь с ними...потом опять в массив и снова из массива на 3 странице..чето сомневаюсь что это так должно быть.
Аноним 06/05/17 Суб 13:39:10  985775
964898900.jpg (212Кб, 1266x1024)
Почему-то не работает offsetUnset в RecursiveArrayIterator. Есть массив с информацией о файле полученной через getID3. В нем есть некоторые элементы с очень длинными кракозябрами, что создает проблемы в базе данных когда я добавляю его в json столбец. Хочу удалить эти элементы.
Аноним 06/05/17 Суб 23:19:29  986117
Почему не закидывают папку vendor часто на Гитхаб для проектов на yii2?
Типа, там всё одинаковое и просто клонируй свою?
Аноним 06/05/17 Суб 23:29:02  986123
>>986117
Так композер сам все подтянет.
Аноним 07/05/17 Вск 00:50:38  986176
>>985770
Я нашёл выход, страница перезагружалась потому что форма отправлялась, я убрал тег <form></form> и просто начал работать с отдельными полями <input>, событие поставил при клике на кнопку, всё отправляется нормально и прописывается в нужном диве, ничего не перезагружается. Я вот только не знаю, можно ли по правила оставлять input без <form>
Аноним 07/05/17 Вск 02:55:10  986200
2017-05-0702-57[...].png (186Кб, 1920x1080)
Анонс поделки

https://github.com/grigoryMovchan/unmarriedLifestyle
https://github.com/grigoryMovchan/AphorismCMS

Код говно, это только прототип. Классы инкапсулированы, легко перекачу на MVC
Аноним 07/05/17 Вск 04:20:46  986208
>>986176
Это работает, ечли у тебя ожна страница, а мне нужны данные на нескольких..
Аноним 07/05/17 Вск 06:21:12  986210
zzazazaz.png (41Кб, 1042x523)
http://ideone.com/1ypuIK
Ну, как теперь?
тот дурачек с первой задачкой на ооп
Аноним 07/05/17 Вск 10:51:45  986240
>>986123
Ни хрена не подтянул, сучка, вообще не могу сделать миграции. Там проект на РНР5.04 рассчитан, а у меня РНР7.
Какие-то расширения не включены, хотя всё раскомментировано.
Вообще в гугле забанили, какая команда на винде для активизации композера для данного проекта? Я нипони.....
Аноним 07/05/17 Вск 11:44:38  986257
>>986240
Разобрался: composer install в папке проекта, а затем php yii migrate.
Всё установилось, всё работает.
Аноним 07/05/17 Вск 23:17:10  986598
>>986200
NetBeans ?
Аноним 08/05/17 Пнд 04:46:23  986675
Снимок.JPG (10Кб, 292x67)
Бля ОП, в регулярках зачем экранировать спецсимвола двойным обратным слэшэм?
С одинарным же работает.
Ты где-то пояснял, но я не могу найти где, почини плес навигацию по урокам.
Аноним 08/05/17 Пнд 04:49:25  986678
>>986675
Бля, скрин кривой. А с поиском самого обратного слэша только с экранированием через два таких же.
Аноним 08/05/17 Пнд 04:49:43  986679
Снимок.JPG (10Кб, 300x57)
>>986678
Аноним 08/05/17 Пнд 04:59:31  986681
1.JPG (19Кб, 1199x33)
2.JPG (11Кб, 356x52)
3.JPG (9Кб, 269x71)
Регулярки.
Открыл две первые ссылки из гугла, в конце инфа из урока ОПа.
И того инфа по поводу \s с трёх разных ресурсов:

>пробел или табуляция
>пробел
>пробел или перевод строки

Это проблема русскоязычных? ВЕДЬ БЛЯДЬ ПРОБЕЛ И ПЕРЕВОД СТРОКИ ЭТО НАХУЙ ДВЕ РАЗНЫЕ ВЕЩИ, ещё и табуляция в придачу. Упоминать об этом совсем не стоит? Пусть ньюфаг сам ебётся потом когда у него из-за перевода строки будут регулярки по пизде бежать)))))
Аноним 08/05/17 Пнд 11:18:41  986722
>>986598
там же написано
Аноним 08/05/17 Пнд 13:21:15  986772
Решаю задачу "Сумма прописью" и зашёл в тупик.

Вот небольшая часть листинга:
/ Возвращает соответствующую числу форму слова: 1 рубль, 2 рубля, 5 рублей /
function inclineWord($number,$word1,$word2,$word5) {
$last2Digits = $number % 100;
if (($last2Digits >= 10)&&($last2Digits <= 20)){
return $word5;
}else if($last2Digits % 10 == 1){
return $word1;
}else if(($last2Digits % 10 >= 2)&&($last2Digits % 10 <= 4)){
return $word2;
}else{
return $word5;
}
}

/
Преобразует числа от 0 до 999 в текст. Параметр $isFemale равен нулю,
если мы считаем число для мужского рода (один рубль),
и 1 — для женского (одна тысяча)
/
function smallNumberToText($number, $isFemale) {

$spelling = array(
0 => 'ноль', 10 => 'десять', 100 => 'сто',
1 => 'один', 11 => 'одиннадцать', 20 => 'двадцать', 200 => 'двести',
2 => 'два', 12 => 'двенадцать', 30 => 'тридцать', 300 => 'триста',
3 => 'три', 13 => 'тринадцать', 40 => 'сорок', 400 => 'четыреста',
4 => 'четыре', 14 => 'четырнадцать', 50 => 'пятьдесят', 500 => 'пятьсот',
5 => 'пять', 15 => 'пятнадцать', 60 => 'шестьдесят', 600 => 'шестьсот',
6 => 'шесть', 16 => 'шестнадцать', 70 => 'семьдесят', 700 => 'семьсот',
7 => 'семь', 17 => 'семнадцать', 80 => 'восемьдесят', 800 => 'восемьсот',
8 => 'восемь', 18 => 'восемнадцать', 90 => 'девяносто', 900 => 'девятьсот',
9 => 'девять', 19 => 'девятнадцать'
);

$femaleSpelling = array(
1 => 'одна', 2 => 'две'
);

if($number % 100 !=0){

}
}
Не понимаю,что надо делать с функцией smallNumberToText. Типа, сам процесс. Надеюсь,кто-нибудь да поможет:3
Аноним 08/05/17 Пнд 13:50:29  986784
>>966608 (OP)
сам аноны
нужно написать мини-веб-приложение для работы с бд
какой фреймворк выбрать?
Аноним 08/05/17 Пнд 17:07:41  986867
1288950710699.jpg (460Кб, 1024x768)
>>986784
Мой возьми

https://github.com/grigoryMovchan/mvc_template
Аноним 08/05/17 Пнд 17:24:36  986880
>>986867
Ах ты социблядина..
Аноним 08/05/17 Пнд 17:25:43  986882
14536411442520.png (46Кб, 256x256)
>>986880
Обидно. Ты меня не знаешь.
Аноним 08/05/17 Пнд 17:31:51  986888
>>986882
>telegram
>soc
Please tell me more about
Аноним 08/05/17 Пнд 17:46:22  986899
>>986888
Я там не сижу, только черпал идеи для поделок
Аноним 08/05/17 Пнд 18:41:30  986931
>>986899
Хорошо. Цитатник понравился, допиливай что задумал.
Аноним 08/05/17 Пнд 19:52:10  986952
>>986772

Нужно сделать массив и в него добавлять по очереди слова для сотен, десяток, единиц, если они есть в числе:

- если число содержит сотни, добавить слово для сотен
- если число оканчивается на 11-20, добавить соотв. слово
- если число содержит десятки, добавить слово для них

И так далее. В конце из массива собрать строку.

>>986681

Обратимся к мануалу: http://php.net/manual/ru/regexp.reference.escape.php

> \s
> любой пробельный символ

> Следующие символы считаются как "пробельные": HT (9), LF (10), FF (12), CR (13), и пробел (32). Тем не менее, если идет локале-зависимый поиск, и произойдет совпадение с символами в диапазоне 128-255, они также будут восприняты как пробельные, например NBSP (A0).

К "пробельным" относятся символы пробела, перевод строки \n, табуляция \t (она добавляет отступ), и более экзотичные спецсимволы \r (используется вместе с \n под Windows) и \f.

Также, при отстутсвии флага u к ним могут добавиться другие пробельные символы вроде символа с кодом 160 (неразрывный пробел), если он есть в кодировке, которая выбрана в текущей локали. Но это нас не должно касаться, так как мы используем юникод, а не однобайтовые кодировки.

А что происходит в юникодном режиме? Обратимся к документации по библиотеке pcre, которую используют preg-функции: http://www.pcre.org/original/doc/html/pcrepattern.html (англ) и ищем там выражение \s

> ....
> If PCRE is compiled with Unicode property support, and the PCRE_UCP option is set, the behaviour is changed so that Unicode properties are used to determine character types, as follows:
> \s any character that matches \p{Z} or \h or \v

Что такое "\p{Z}"? Это описано тут http://php.net/manual/en/regexp.reference.unicode.php и тут https://en.wikipedia.org/wiki/Unicode_character_property . В Юникоде у каждого символа есть категория: является ли он буквой, цифрой, разделителем или чем-то еще. \p{Z} - это символы из категории "разделители", они перечислены например тут:

- http://www.fileformat.info/info/unicode/category/Zl/list.htm
- http://www.fileformat.info/info/unicode/category/Zp/list.htm
- http://www.fileformat.info/info/unicode/category/Zs/list.htm

Также, про юникодные пробелы есть статья на англ: https://www.cs.tut.fi/~jkorpela/chars/spaces.html

Далее, посмотрим что такое \h и \v для pcre в ее мануале:

> The sequences \h, \H, \v, and \V are features that were added to Perl at release 5.10. In contrast to the other sequences, which match only ASCII characters by default, these always match certain high-valued code points, whether or not PCRE_UCP is set. The horizontal space characters are:

И далее там перечислены конкретные символы.

В общем, в юникодном режиме \s соответствует "старым" ASCII-символам вроде пробела, \n, \t, неразрывному пробелу + множеству юникодных пробелов и разделителей строк и абзацев.

Если ты не очень понимаешь что такое кодировки, юникод, то советую мою статью про них: https://github.com/codedokode/pasta/blob/master/cs/strings.md

Также, вот статья про юникод и регулярки на англ: http://www.regular-expressions.info/unicode.html
Аноним 08/05/17 Пнд 19:52:36  986953
>>986675

Дело в том, что бекслеш используется как спецсимвол на 2 уровнях: в движке регулярок при разборе регулярного выражения и в синтаксисе PHP при разборе содержимого строки.

Например, когда ты пишешь "\\" в строку вставляется один бекслеш, что легко проверить с помощью echo:

echo "\\"; // выведет \

Это описано в мануале: http://php.net/manual/ru/language.types.string.php Потому, чтобы вставить в строку выражение \s, мы должны по правилам PHP записать \\s:

echo "\\s"; // выведет \s

Однако, \s не является в PHP особой последовательностью, потому мы можем написать просто \s:

echo "\s"; // выведет \s

Таким образом, \s можно записать как \s и как \\s. Но если ты хочешь написать регулярку для поиска бекслеша, то придется заморочиться. В регулярке мы хотим написать \\, но если мы напишем "\\" то это вставит в строку только один бекслеш. Потому мы вынуждены писать бекслеш 4 раза:

echo "\\\\"; // выведет \\

Либо использовать форму записи, которая не интерпретирует бекслеши:

echo <<<'EOF'
\\
EOF;

// выведет \\

>>986210

> ['question1' => 'Какая планета
> ['question2' => 'Столица Англии?',
Тут нет смысла использовать разные ключи, так как они находятся в разных массивах.

> $q1->text = $array['question1']['question1'];
> $q2->text = $array['question2']['question2'];
Это унылая копипаста. Ты бы мог использовать цикл, проходясь по массиву вопросов и создавая для каждого вопроса свой объект. Либо обойтись без массива вопросов и записывать значения прямо в объект.

>>986176

Это плохая идея, так как у тебя форма и для группировки элементов надо поместить из внутрь form. Чтобы форма не отправлялось, надо отменять действие по умолчанию для события submit (preventDefault).

> событие поставил при клике на кнопку
Это неправильно так как обычную форму можно отправить и нажатием Enter (и нажатием зеленой кнопки на виртуальной клавиатуре в Андроиде например). Ты вместо того, чтобы разобраться в событиях, испортил код и сделал форму менее удобной в использовании.
Аноним 08/05/17 Пнд 19:53:00  986954
>>986117

Потому что в репозитории должен быть твой код. Зачем дублировать по всему интернету устаревшие версии библиотек? Идея композера в том, что автор программы просто описывает, какие библиотеки ему нужны, в файлике, а композер сам их скачает и поставит.

>>985775

http://php.net/manual/en/arrayiterator.offsetunset.php
Эта функция не документирована, я бы советовал ее не использовать пока. Потому что понять, как она работает, можно только изучив исходный код PHP, а мне лень это делать. Используй array_walk_recursive пока что.

Ну и кстати элемент там полезный, он содержит картинку в формате JPEG судя по всему. Можно было бы использовать ее как превьюшку.

> , что создает проблемы в базе данных когда я добавляю его в json столбец
Тут стоило бы разобраться, почему. Может у тебя ошибка в коде. Вроде судя по документации в JSON можно засунуть до 4 Гб данных.

>>985569

Использование плейсхолдеров защищает от SQL-инъекций (и от взлома базы), но не защитит от того, что пользователь вставит "Войну и Мир" в поле "имя" или введет телефон в поле email. Ты должен задать ограничения и проверять их, если не хочешь чтобы базу забили мусором.

> при помощи substr
substr не работает с кириллицей, советую использовать mb_substr.

>>985567

нужно отдавать с сервера JSON, например:

{"success": true, "id": 123456}
{"success": false, "errors": ["Пожалуйста, укажите имя пользователя"]}

> Главный вопрос, это слишком криво? так делать нельзя?
Конечно, криво.
Аноним 08/05/17 Пнд 19:53:41  986955
>>985563

> Если получаю id, то пишу (int)id, в аякс всёравно приходит строка.
Потому что в HTTP-ответе тело всегда содержит строку. Тип там не передается (точнее, есть Content-Type, но это немного другое).

>>985545

Вся кастомизация в вордпрессе по моему сводится к написанию тем или плагинов. Достаточно прочитать мануал вордпресса по этим 2 темам. Сложные вещи придется делать по сути целиком самому с нуля и пристраивать сбоку, так как вордпресс это блоговый движок, а не фреймворк для создания сложных сайтов.

>>985359

На ideone при обновлении PHP забыли включить модуль mbstring, и mb-функции не работают.

>>985354

Глянь для начала эти уроки, которые имеют косвенное отношение к теме:

https://github.com/codedokode/pasta/blob/master/js/minesweeper-mvc.md
https://github.com/codedokode/pasta/blob/master/js/spa.md

А так, на jQuery наверно будет проще.

>>985312

С помощью плейсхолдеров можно подставлять только значения, но не имена полей или таблиц. Твоц код формирует запрос вида

ORDER BY 'red'

вместо

ORDER BY red

Имена полей надо тщательно проверять и подставлять напрямую.

>>985165

Ну так у нас есть класс Question, а создание объектов мы решили пока сделать функцией, это урок для начинающих. А так, можно конечно сделать какой-нибудь QuestionLoader или QuestionService который создает вопросы.
Аноним 08/05/17 Пнд 19:54:18  986956
>>985046

Заглядывает

>>984954

Добавь еще кириллический адрес ivan@пример.рф или иван@пример.рф

>>984949

Использовать preventDefault

>>984920

Я бы сделал просто скриптом из 10 строк. А так, можно сделать класс FizzBuzzGenerator с статическим методом getFizzBuzzValues(....), куда бы передавал аргументы (не в конструктор). Либо можно сделать этот метод генератором, чтобы не хранить в памяти массив.

>>984823

По поводу архивов, 25 тред есть только с архивача. Видимо, была какая-то проблема с сервером или вайп, и тред потерялся. Вот ссылки:

- http://www.mediafire.com/file/r8vs6odv6bfe37l/threads-pr-25-arhivach.zip
- http://www.mediafire.com/file/16zwvu74mzo7yhh/threads-archive-pr-40-59.zip
- http://www.mediafire.com/file/kpot4mx3tbd4zad/threads-archive-pr-18-24-and-26-39.zip

Вообще, сфинкс и mysql работает по сети, так что ты бы мог разместить их на разных машинах, но смысла особого нет, так как надо настраивать ограничение доступа, проще поместить на одной машине.

> Есть какое-нибудь решение чтобы объединить два сервера и/или хостинга в один? Или обычно просто указывают абсолютный адрес изображения в шаблоне?
Вот там ниже кто-то советовал сетевую файловую систему, но я считаю такое решение "костылем". Его используют, когда нет ресурсов на то, чтобы переделать приложение, и подключают сетевую ФС, чтобы для приложения это выглядело как будто все файлы лежат локально. Но это работает не очень эффективно.

Сетевая ФС - это штука, которая как бы "монтирует" папку с удаленной системы в локальную файловую систему. В реальности при обращении к файлу идет запрос по сети на другой сервер. Это например существующая давно под линуксом NFS, Samba для Windows.

Тебе лучше генерировать абсолютные URL и сделать настройку домена в конфиге. Также, тебе может понадобиться решение для автоматической загрузки файлов на хостинг, тут можно подумать над использованием ssh/sftp, на худой конец FTP (он не защищен и пароль передается открытым текстом, не рекомендую) или той самой сетевой ФС (только хостинг вряд ли тебе даст ее подключить в отличие от ssh или FTP).

Вообще, для больших проектов (вроде соцсетей) обычно делают так: выделяют N серверов, на них ставят nginx для раздачи файлов + мини-приложение через которое по внутреннему API (а иногда и снаружи, например, через HTML форму) можно закачивать файлы и управлять ими. Если интересно, https://www.insight-it.ru/highload/2010/arkhitektura-facebook/#khranenie-fotografii

(вообще тут интересные статьи по highload-проектам: https://www.insight-it.ru/highload/ )

И соответственно где-то в базе хранят, на каком сервере (серверах) доступен файл. Чтобы сгенерировать нужную ссылку.

Есть еще сторонние хранилища вроде

- http://docs.aws.amazon.com/gettingstarted/latest/awsgsg-intro/gsg-aws-storage-cdn.html
- https://selectel.ru/services/cloud/storage/

Это по сути и есть те сервера с нгинксом и API для управления файлами. Насчет цен, конечно, не знаю насколько они выгодны.

> Как преобразовать $_GET запрос отправляемой формы из /search.php?q=... в /search/... ? Нужно разбирать ссылку в контроллере и делать ридерект?
Можно, но не очень понимаю смысл этого редиректа. Что мешает отдать данные сразу?

> Хотелось бы иметь собственное доменное имя.
Доменная система иерархическая. Есть корневые сервера (a.root-servers.net и так далее). Они хранят базу данных для TLD (и являются авторитетными серверами для нее), то есть они могут сказать, какой сервер является авторитетным для зоны .com, какой для зоны .ru. Эта база данных управляется ICANN.

Авторитетный сервер - это сервер, который смотрит ответ на запрос в соотв-й базе данных. Сторонние DNS-сервера вроде 8.8.8.8 - не авторитетные, так как у них нет доступа к базе доменов, они запрашивают данные у авторитетных серверов.

Каждой зоной управляет свой администратор, назначеныый ICANN, он ведет базу доменов 2 уровня в этой зоне, и его авторитетный сервер может дать адрес сервера, отвечающего за тот или иной домен.

Например доменом .ru управляет https://en.wikipedia.org/wiki/Coordination_Center_for_TLD_RU

Часто администратор не продает домены напрямую пользователям, а использует для этого посредников - сторонних регистраторов, которым дает доступ к своему API.

Соответственно при покупке домена ты добавляешь информацию о нем в соответствующий реестр, это название домена, 2 или больше NS (name server) для него, и whois-информация. И соответственно DNS-запросы к твоему домену example.com будут идти на твой сервер. А он уже будет отдавать IP-адрес для домена и другие данные.

Поскольку поддержка своего DNS-сервер требует определеных усилий, многие регистраторы позволяют при покупке домена использовать их DNS-сервер. Также, есть бесплатные DNS-сервера, например у Яндекса: https://yandex.ru/support/pdd/domain/dns.xml Но используя свой сервер, ты можешь реализовать больше возможностей, например раздавать разные Ip-адреса в зависимости от расположения пользователя, управлять временем кеширования данных и тд.

Если у тебя есть под рукой линукс, ты можешь увидеть как именно происходит разрешение запроса. Обычно приложения просто отправляют запрос на не-авторитетный DNS-сервер вроде 8.8.8.8 или сервер местного провайдера, а тот уже идет по цепочке авторитетных серверов, кеширует их ответы и тд. Но мы попробуем разрешить адрес узла вручную, используя только авторитетные сервера.

Допустим мы хотим узнать информацию о домене weather.yandex.ru. Для начала мы должны узнать названия корневых серверов. Они есть на сайте ICANN: https://www.internic.net/zones/named.cache

Далее, мы отправим запрос на корневой сервер A.ROOT-SERVERS.NET, чтобы узнать кто является авторитетным сервером для зоны .ru:

robot@debian:/tmp$ host -a -r ru a.root-servers.net
Trying "ru"
Using domain server:
Name: a.root-servers.net
Address: 198.41.0.4#53
Aliases:

;; ->>HEADER<<- opcode: QUERY, status: NOERROR, id: 30041
;; flags: qr; QUERY: 1, ANSWER: 0, AUTHORITY: 5, ADDITIONAL: 10

;; QUESTION SECTION:
;ru. IN ANY

;; AUTHORITY SECTION:
ru. 172800 IN NS a.dns.ripn.net.
ru. 172800 IN NS b.dns.ripn.net.
ru. 172800 IN NS d.dns.ripn.net.
ru. 172800 IN NS e.dns.ripn.net.
ru. 172800 IN NS f.dns.ripn.net.

;; ADDITIONAL SECTION:
a.dns.ripn.net. 172800 IN A 193.232.128.6
b.dns.ripn.net. 172800 IN A 194.85.252.62
d.dns.ripn.net. 172800 IN A 194.190.124.17
e.dns.ripn.net. 172800 IN A 193.232.142.17
f.dns.ripn.net. 172800 IN A 193.232.156.17
a.dns.ripn.net. 172800 IN AAAA 2001:678:17:0:193:232:128:6
b.dns.ripn.net. 172800 IN AAAA 2001:678:16:0:194:85:252:62
d.dns.ripn.net. 172800 IN AAAA 2001:678:18:0:194:190:124:17
e.dns.ripn.net. 172800 IN AAAA 2001:678:15:0:193:232:142:17
f.dns.ripn.net. 172800 IN AAAA 2001:678:14:0:193:232:156:17

Число 172800 - это время в секундах, на которое можно закешироввать ответ.

Если мы попробуем у корневого сервера запросить например IP-адрес для yandex.ru, то он даст тот же ответ, то есть имена серверов, отвечающих за зону .ru, и предложит нам обратиться к ним.

Далее, узнаем кто является авторитетным сервером для yandex.ru:

robot@debian:/tmp$ host -a -r yandex.ru a.dns.ripn.net
Trying "yandex.ru"
Using domain server:
Name: a.dns.ripn.net
Address: 193.232.128.6#53
Aliases:

;; ->>HEADER<<- opcode: QUERY, status: NOERROR, id: 52434
;; flags: qr; QUERY: 1, ANSWER: 0, AUTHORITY: 2, ADDITIONAL: 4

;; QUESTION SECTION:
;yandex.ru. IN ANY

;; AUTHORITY SECTION:
YANDEX.RU. 345600 IN NS ns2.yandex.RU.
YANDEX.RU. 345600 IN NS ns1.yandex.RU.

;; ADDITIONAL SECTION:
ns1.YANDEX.RU. 345600 IN A 213.180.193.1
ns2.YANDEX.RU. 345600 IN A 93.158.134.1
ns1.YANDEX.RU. 345600 IN AAAA 2a02:6b8::1
ns2.YANDEX.RU. 345600 IN AAAA 2a02:6b8:0:1::1

Сделаем запрос к серверу Яндекса:

robot@debian:/tmp$ host -a -r weather.yandex.ru ns1.yandex.ru
....
;; AUTHORITY SECTION:
weather.yandex.ru. 3600 IN NS ns4.yandex.ru.
weather.yandex.ru. 3600 IN NS ns3.yandex.ru.

Нам опять предлагают спросить у других серверов. Обратимся к ним:

robot@debian:/tmp$ host -a -r weather.yandex.ru ns3.yandex.ru
Trying "weather.yandex.ru"
Using domain server:
Name: ns3.yandex.ru
Address: 87.250.250.1#53
Aliases:

;; ->>HEADER<<- opcode: QUERY, status: NOERROR, id: 17848
;; flags: qr aa; QUERY: 1, ANSWER: 6, AUTHORITY: 0, ADDITIONAL: 0

;; QUESTION SECTION:
;weather.yandex.ru. IN ANY

;; ANSWER SECTION:
weather.yandex.ru. 172800 IN NS ns3.yandex.ru.
weather.yandex.ru. 172800 IN NS ns4.yandex.ru.
weather.yandex.ru. 600 IN A 213.180.193.248
weather.yandex.ru. 3600 IN MX 0 mx.yandex.ru.
weather.yandex.ru. 600 IN AAAA 2a02:6b8::1:248
weather.yandex.ru. 600 IN SOA ns3.yandex.ru. sysadmin.yandex.ru. 2015050
822 900 600 3600000 300

Теперь ответ получен - IP-адрес для weather.yandex.ru - это 213.180.193.248

Заметь, что система использует дублирование - для каждой зоны есть минимум 2 авторитетных сервера.

Неавторитетные DNS-сервера провайдеров обычно сами делают рекурсивные запросы и кешируют ответы, чтобы снизить нагрузку на авторитетные серверы. Если мы отправим запрос к 8.8.8.8, то он даст ответ сразу:

robot@debian:/tmp$ host -a weather.yandex.ru 8.8.8.8
....
weather.yandex.ru. 599 IN A 213.180.193.248

Конечные пользователи, как правило, пользуются такими неавторитетными серверами.
Аноним 08/05/17 Пнд 19:56:49  986957
>>984865

> Цепь блоков состоит из узлов. Когда мы устанавливаем Bitcoin Core мы создаем новый узел?
Нет, когда ты устанавливаешь клиент Bitcoin, ты становишься узлом (node) сети Биткойн.

А цепь блоков (blockchain) состоит из блоков, каждый из которых хранит N транзакций. Транзакция - это запись о переводе средств с одного кошелька на другой, подписанная ключом исходного кошелька.

Новые блоки генерируют узлы сети, которые занимаются майнингом. Узел делает ресурсоемкий перебор чисел (майнинг), чтобы найти число, соответствующее определенным условиям. Как только он его нашел, он получает возможность добавить новый блок в блокчейн (найденное число позволяет подписать новый блок, без него твой блок никто не примет). Каждый блок содержит определенное число монет, и тот, кто его создал, распоряжается ими как хочет - он может перевести их себе или на любой кошелек. Также тот, кто создал блок, может добавить в него транзакции от других участников сети, обычно он за это берет плату с них (разница между деньгами на входе транзакции и на выходе и есть эта комиссия и она достается майнеру). Создав и подписав блок, он рассылает его другим узла сети, они проверяют блок, все транзакции в нем, если они валидные, то добавляют блок в свою копию блокчейна.

Таким образом, единственный источник средств - это новые блоки. Тот, кто создал блок, получает это возможность распорядиться определенным числом новых монет. Все транзакции записаны в блокчейне, и любой, имеющий его копию, может убедиться, что кошелек X имеет Y монет.

Майнинг и комиссии создают мотивацию для создания и поддержки узлов сети.

Тот, кто хочет перевести деньги со своего кошелька, формирует транзакцию, включает в нее выплату комиссии (размер комиссии задает владелец кошелька, комиссия определяется как разница между деньгами на входе и на выходе транзакции) и рассылает эту транзакцию майнерам для включения в новый блок. Майнеры сортируют пришедшие транзакции по убыванию комиссии и включают в блок самые прибыльные для них транзакции. Так производится регулировка комисси с помощью рыночных механизмов. Ты можешь ставить комиссию меньше, но ниже вероятность, что транзакцию включат в блокчейн.

У каждой транзакции есть N входов и M выходов. Как я понимаю, перевод 1 биткойна с кошелька A с 10 биткойнами на B выглядит как транзакция, на вход которой поступает A, на выходе 1 биткойн передается в B, 8.99 передаются назад в A и 0.01 станут щедрой комиссией тому, кто включит транзакцию в блок.

https://en.bitcoin.it/wiki/Transaction
https://en.bitcoin.it/wiki/Protocol_documentation#Transaction_Verification

Также, если ты прочитаешь, то увидишь что в транзакциии есть поле, содержающее скрипт на специальном языке для ее верификации.

Биткойн интересен не только в плане использования криптографии, но и тем, как выстроена система мотивации, вознаграждений.

https://en.bitcoin.it/wiki/Block_chain

Что касается объема блокчейна - есть вариант "тонкого клиента", когда ты используешь сервер, которму доверяешь, для проверки транзакций:

https://en.bitcoin.it/wiki/Scalability#Simplified_payment_verification
https://www.quora.com/Why-isnt-the-size-of-the-blockchain-a-serious-problem-for-Bitcoin

> В примерах приведенных в документации есть пример с командой sendtoaddress https://bitcoin.org/en/developer-examples#simple-spending . Там создается новый кошелек на который переводиться сумма, но не создается кошелек с которого это сумма переводиться.

Там по моему может быть только один кошелек и он задается в конфиге: https://github.com/bitcoin/bitcoin/issues/3113

Я не знаю, может ли у одного кошелька быть несколько адресов или нет.

Также, там написано:

> The sendtoaddress RPC automatically selects an unspent transaction output (UTXO) from which to spend the satoshis. In this case, it withdrew the satoshis from our only available UTXO, the coinbase transaction for block #1 which matured with the creation of block #101.

> Мне же не обязательно хранить закрытый ключ в БД, если я хочу чтобы у каждого пользователя был собственный кошелек? Ведь его можно получить в любой момент с помощью команды dumpprivkey.

Не уверен, может ли он управлять кошельками для разных пользователей.

> В примере с simple raw transaction ( https://bitcoin.org/en/developer-examples#simple-raw-transaction ) это делать не обязательно. Получается Bitcoin Core это всего лишь кошелек, который может работать только с его собственными адресами
Ты можешь только делать операции с кошельками, к которым у тебя есть приватный ключ. Так что скорее всего кошелек там один, но возможно, для него можно создавать много адресов.

Также, в примерах используется специальный режим regtest, вместо работы с настоящей сетью Биткойн: https://bitcoin.org/en/developer-examples#regtest-mode

Там данные хранятся в файлах:

> Regtest wallets and block chain state (chainstate) are saved in the regtest subdirectory of the Bitcoin Core configuration directory.

> Наверно, должен быть способ чтобы перенести закрытые и открытые ключи на другой носитель. Как они хранятся?
Надо смотреть документацию конкретной программы. Скорее всего в файле. Вот тут есть общая информация: https://bitcoin.org/en/developer-guide#wallets
Аноним 08/05/17 Пнд 20:53:58  987000
>>984844

Не понял вопрос. Там же форма? Значит значения ее элементов присылаются на сервер в запросе (GET/POST).
Аноним 09/05/17 Втр 01:24:31  987128
>>966608 (OP)
https://ideone.com/vuROr6 как это понимать вообще? Почему рандомные буквы появляюца.
Аноним 09/05/17 Втр 01:45:06  987135
2017-05-0901-41[...].png (176Кб, 1920x1080)
2017-05-0901-41[...].png (196Кб, 1920x1080)
>>986200
Продолжаю пилить поделку:

Сделал страницу с рандомной цитатой в возможностью их листать по одной
Сделал список цитат с якорями
Сделал предложку цитат

Шаблон не осилил, верстка поехала, не смог в CSS. Благо на MVC всего один файл надо поправить, чтоб на другой перекатится.

Дальше буду админку пилить.

Я знаю, что ОП найдет десяток пунктов по которым меня можно справедливо обоссать, поэтому разборов пока никаких не прошу, только по желанию.
Аноним 09/05/17 Втр 01:49:12  987138
>>987135
проиграл с беховена
Аноним 09/05/17 Втр 02:17:37  987151
>>987135
Ну, пятюню за старания точно заслужил.
А в остальном ничего не скажу, ибо не оп.
Аноним 09/05/17 Втр 02:34:34  987157
14922771007610.jpg (36Кб, 409x405)
>>987151
Аноним 09/05/17 Втр 03:27:46  987204
>>984117
То был не я.
Аноним 09/05/17 Втр 11:11:08  987290
Не могу понять как эти вложенные циклы работают, пиздец какой-то.
Аноним 09/05/17 Втр 11:58:20  987323
>>987138
сук, бля, ору
Аноним 09/05/17 Втр 11:59:34  987326
>>987290
Пример кода покажи или задачи, что там надо.
Вопрос по индексам в БД Аноним 09/05/17 Втр 15:17:23  987588
https://github.com/codedokode/pasta/blob/master/db/databases.md#Индексы/

SELECT FROM table WHERE x <> 1
Индекс не поможет. Не знаю, уместна ли такая аналогия: мы хотим найти в словаре все слова не X c помощью бинарного поиска. Открываем словарь по середине, допустим там слово Y, ок, но куда теперь двигаться дальше? Непонятно ведь какую часть словаря теперь открывать, нужен полный перебор.

SELECT
FROM table WHERE x + y < 100
Индекс на x не будет использоваться. Это потому, что индекс построен для X, а мы запрашиваем X+Y? Затрудняюсь провести аналогию со словарём.

SELECT MAX(a) FROM table WHERE b = 2
Индекс на b поможет.

SELECT FROM table WHERE name LIKE '%Иван%'
Для такого запроса индекс по name не поможет, всё равно что искать в словаре слова с неизвестной первой буквой. Как понял, должен помочь FULLTEXT индекс и операторы AGAINST/MATCH

SELECT
FROM table WHERE b = 1 AND a < 10
Это не понял, как и эту статью https://ruhighload.com/post/Работа+с+индексами+в+MySQL
Автор сначала создаёт индекс age_gender и пишет, что значения индекса будут такими:
12male
15female
29male
89tsar

И утверждает, что запрос SELECT * FROM users WHERE age <= 29 AND gender = 'male' не будет использовать полный индекс, т.к. "значения gender будут отличаться для разных значений колонки age.". Я понимаю, отбираем 12male, 15female, 29tsar и нужно пройтись по значениям, чтобы отобрать gender = 'male'

Но потом он добавляет индекс gender_age и пишет, что теперь запросы будет использовать полный индекс. Выходит такой индекс
male12
male29
female15
tsar89
male99 -- добавил я

По аналогии отбираем male12 и male29, male99. Выходит, что по ним теперь тоже нужно пройтись, чтобы отобрать те, где age <= 29?
Аноним 09/05/17 Втр 15:24:35  987599
>>986956
>Тебе лучше генерировать абсолютные URL и сделать настройку домена в конфиге.
О какой настройке идёт речь? Можно же и так ссылаться на любой домен в html ссылке.


>> Как преобразовать $_GET запрос отправляемой формы из /search.php?q=... в /search/... ? Нужно разбирать ссылку в контроллере и делать ридерект?
>Можно, но не очень понимаю смысл этого редиректа. Что мешает отдать данные сразу?
Ничего, просто хотелось иметь красивую ссылку. В принципе, можно обойтись и без этого.


>Часто администратор не продает домены напрямую пользователям, а использует для этого посредников - сторонних регистраторов, которым дает доступ к своему API.
Значит не получиться обойтись без посредников. Это печально, потому что регистраторы с первой страницы гугла выглядят как очередные корпорации зла. Хотелось бы обойтись без них.


Аноним 09/05/17 Втр 18:46:53  987705
Есть у кого хорошие гайды по vue, особенно по архитектуре? Сейчас над одним проектом с ним работаю - пизедц там каша. Даже не знаю норма это или нет.
Аноним 09/05/17 Втр 22:16:58  987832
14620355806400.png (233Кб, 858x725)
Как решить палиндром из учебника !?
Аноним 09/05/17 Втр 23:13:23  987855
>>987832
Код показывай, вангователей тут нет.
Аноним 09/05/17 Втр 23:21:40  987857
А как оно работает вообще? Вот написал я echo что-то там, оно его и вывело. Но почему именно эхо и как распознаётся сие эхо?
Аноним 10/05/17 Срд 00:00:41  987901
palindroom.png (44Кб, 706x692)
>>987855
Не понимаю, что писать в цикле.
Аноним 10/05/17 Срд 00:25:06  987924
>>987901
Накодил тебе, пользуйся
http://ideone.com/HJgfGF
Аноним 10/05/17 Срд 02:31:22  987949
>>987857

Есть такое понятие, как синтаксис, или грамматика. Это набор правил, которые заложены в интерпретатор PHP. Эти правила описывают, из каких частей состоит программа, из каких символов состоят отдельные части. Ну например, "переменная" описывается как последовательность из знака доллара и идущих за ним определенных символов (букв, цифр и тд). Или "целое число" - это последовательность из необязательного знака минус, и 1 или более цифр от 0 до 9. Из этих маленьких неделимых кусочков (они называются терминалы) составляются более сложные части. Например, "математическое выражение" - это последовательность, состоящая из переменных, чисел, знаков математических операций. Или "цикл for" - это конструкция из ключевого слова for, 3 выражений в скобках и тела цикла.

Ты наверно не поймешь его полностью, но вот ссылки на реальный код из интерпретатора PHP, который описывает грамматику языка PHP. Имено по этим правилам анализируется текст программы перед выполнением:

https://github.com/php/php-src/blob/master/Zend/zend_language_scanner.l
https://github.com/php/php-src/blob/master/Zend/zend_language_parser.y

Это файлы содержат смесь кода на Си и языка Yacc, который позволяет описывать грамматики. Ну вот например кусочек из первого файла, описывающий синтаксис переменной:

https://github.com/php/php-src/blob/master/Zend/zend_language_scanner.l#L1870
> <ST_IN_SCRIPTING,ST_DOUBLE_QUOTES,ST_HEREDOC,ST_BACKQUOTE,ST_VAR_OFFSET>"$"{LABEL} ...
> RETURN_TOKEN(T_VARIABLE);
> ...

Это можно прочитать так:

- ST_IN_SCRIPTING - если мы находимся где-то в тексте программы
- ST_DOUBLE_QUOTES - или внутри строки с двойными кавычками
- ST_HEREDFOC - или внутри конструкции heredoc (гугли)
- ST_BACKQUOTE - или внутри строки с косыми кавычками
- ST_VAR_OFFSET - или внутри квадратных скобок
- "$"{LABEL} - и видим знак доллара, за которым идут символы, подходящие под правило LABEL
- RETURN_TOKEN(T_VARIABLE) - значит это обозначение переменной

Кстати, определение правила LABEL есть в том же файле:

https://github.com/php/php-src/blob/master/Zend/zend_language_scanner.l#L1115
> LABEL [a-zA-Z_\x80-\xff][a-zA-Z0-9_\x80-\xff]*

Это регулярное выражение определяет, какие символы можно использовать в имени переменной.

Первый файл описывает терминалы - маленькие неделимые кусочки программы. Второй файл описывает более сложные конструкции, которые можно составлять из этих кусочков.

Вот например правило, которое описывает, из чего может состоять выражение (оно довольно большое, я скопирую лишь кусочек):

https://github.com/php/php-src/blob/master/Zend/zend_language_parser.y#L872
> expr_without_variable:
> | variable '=' expr
> | expr '+' expr
> | expr '-' expr
То есть выражение может состоять из переменной, знака равно и другого выражения, или же из 2 выражений, разделенных знаком плюс или минус. Это я скопировал только часть, на самом деле там правило гораздо больше и сложнее.

Когда ты запускаешь свою программу, то PHP используя эти правила, анализирует текст программы и разбирает ее на составляющие. Программа разбивается на инструкции (команды), а каждая из них на более мелкие составные части. Ну например, строка

$x = $y + 1;

Разбивается примерно на такие части:

- команда, содержащая операцию присваивания значения переменной
- где переменная называется $x
- а значение, которое ей надо присвоить, это выражение:
- сумма 2 слагаемых
- первое - переменная $y
- второе - число 1

Что касается операции echo, про нее можно почитать в мануале: http://php.net/manual/ru/function.echo.php

Если ты дашь PHP программу, которая не соответствует правилам, то получишь ошибку "Syntax error in line N".

Ну то есть, если кратко ответить на твой вопрос, "почему именно так", потому что такие правила заложены в язык PHP его создателями.

Если тебя вдруг заинтересовала тема грамматик и синтаксического анализа, то гугл в помощь: https://www.google.ru/search?q=%D0%B3%D1%80%D0%B0%D0%BC%D0%BC%D0%B0%D1%82%D0%B8%D0%BA%D0%B0+%D1%81%D0%B8%D0%BD%D1%82%D0%B0%D0%BA%D1%81%D0%B8%D1%87%D0%B5%D1%81%D0%BA%D0%B8%D0%B9+%D0%B0%D0%BD%D0%B0%D0%BB%D0%B8%D0%B7&newwindow=1&gbv=1&sei=L0gSWbWrOoPE6ASt6aCwDA
Аноним 10/05/17 Срд 09:55:06  988018
>>987949
Ты, конечно, за словом в карман не лезешь, когда нубик всего лишь про эхо спрашивает.
Аноним 10/05/17 Срд 10:40:44  988037
>>988018
Лол, страшно представить что будет, если о переменных спросить
Аноним 10/05/17 Срд 11:26:49  988051
>>987924
Давай с последним шагом подробнее. Не понимаю, что значит дошли до конца цикла https://3v4l.org/SZo4b
Аноним 10/05/17 Срд 11:30:19  988054
>>988051
Значит что цикл закончился и пошел код уже после цикла.
Аноним 10/05/17 Срд 11:33:57  988057
>>988051
>>988054
Конкретно для твоего случая это значит нормальное завершение цикла по счетчику, а не по брику. Как уж ты будешь понимать, что у тебя он нормально все проверил сам решай.
Аноним 10/05/17 Срд 11:44:11  988062
Еле вас нашёл, тут бамп-лимит 500 постов?
Аноним 10/05/17 Срд 12:03:09  988069
>>988062
Не знаю чего оп не перекатывает раньше 1к постов, тут тухло всегда из-за этого.
Аноним 10/05/17 Срд 13:03:13  988110
>>988057
Блядь, а че ты мне долбоебу не сказал, что я из числа пытаюсь букву вернуть ?
Аноним 10/05/17 Срд 13:12:12  988116
>>988110
Ну не делать же за тебя все.
Аноним 10/05/17 Срд 13:25:01  988122
>>986957
>Что касается объема блокчейна - есть вариант "тонкого клиента", когда ты используешь сервер, которму доверяешь, для проверки транзакций:
У клиента есть ещё вариант с ограничением дискового пространства: https://bitcoin.org/en/full-node#configuration-tuning
Аноним 10/05/17 Срд 13:25:54  988123
>>988122
>https://bitcoin.org/en/full-node#configuration-tuning
https://bitcoin.org/en/full-node#reduce-storage
Аноним 10/05/17 Срд 13:40:51  988140
>>988069

Скоро перекатим.
Аноним 10/05/17 Срд 13:45:33  988148
>>988140
Я наблюдаю за тобой, не дай бог не перекатишь.
Аноним 10/05/17 Срд 16:25:41  988273
>>988051
А ты про Айфон вообще делал задачу? Какие-то непонятные вопросы задаешь для этого уровня.
Аноним 10/05/17 Срд 16:34:34  988274
>>988273
Айфон в кредит для школьника ? Вот именно ее одну и пропустил.
Аноним 10/05/17 Срд 17:03:06  988290
>>988148
Он не перекатил, пизди его.
Аноним 10/05/17 Срд 17:22:37  988298
>>988290
>>988148
Флудить укатывайтесь в другое место. Думаю, я не один такой, кто ждёт проверки/ответа от ОПа, видя новые сообщения в треде, но взамен натыкается на мусорные посты вроде ваших.
Аноним 10/05/17 Срд 17:32:39  988303
>>988298
Еще ты будешь меня гнать, с тебя не убудет мои мусорные посты отсеивать, хуй ленивый.
Я тут сижу и жду какой-нибудь энергичной движухи, которой не будет без переката, только унылая проверка нубских задачек. Ты думаешь тут одни обучающиеся сидят чтоли? Попиздеть за жизнь и разработку тоже хочется.
Аноним 10/05/17 Срд 17:45:41  988308
А пэхэпэ только для сайтов? Программы пишутся на каких-то других языках?
Аноним 10/05/17 Срд 17:52:47  988311
>>988308
Собственно то, что делает пхп на беке сайта - это тоже программа. Не все то программа, во что можно потыкать мышкой. Да, ПО других типов на нем не принято писать от слова совсем (хоть и возможно).
Аноним 10/05/17 Срд 17:56:37  988312
>>988311
А на чём пишут?
Аноним 10/05/17 Срд 18:01:12  988317
>>988312
Из актуальных направлений:
Ведро - java, kotlin, scala (и для ndk толи на крестах, толи на чем-то таком)
Яблоко - obj c, swift
Фронтенд сайтов - js и его диалекты.
Аноним 10/05/17 Срд 18:19:53  988327
>>987949
Сам язык и его правила также написаны на каком-то языке?
Аноним 10/05/17 Срд 18:22:13  988328
>>988303
>только унылая проверка нубских задачек
Бля, вот ща обидно было((
Аноним 10/05/17 Срд 18:32:40  988335
Ну пиздец, эзотерические языки, языки ада. А Сатану с массивом демонов как вызвать?
Аноним 10/05/17 Срд 18:34:09  988338
>>988335
А первый высокоуровневый язык вообще создан нацистами. Что-то тут не чисто.
Аноним 10/05/17 Срд 18:45:15  988340
Давайте перекат уже.
Аноним 10/05/17 Срд 19:51:05  988360
>>988274
Лучше вернись к ней, это стоит того.
Такие вещи надо понять однажды, чтобы потом не ошибаться во всём этом.
Аноним 10/05/17 Срд 21:09:48  988397
по поводу калькулятора из учебника опа. немного не понятно, что делать, если символ - операция. то есть, сначала мы должны проверить, что находится в нашей переменной $op(с помощью if..else), с учетом найденной операции мы должны ее сделать,сохранив результат в $result, затем обнулить $number? Или я что-то не так понял?
Аноним 10/05/17 Срд 23:10:22  988465
Перекат будет ?
Аноним 11/05/17 Чтв 00:00:23  988493
2017-05-1023-55[...].png (167Кб, 1920x1080)
>>987135
Дошли руки до админки, крудошлепствую. Приятно работать в парадигме MVC :3
Аноним 11/05/17 Чтв 00:04:54  988496
>>988493
Пиздец ты программист.
Аноним 11/05/17 Чтв 00:10:28  988498
vlcsnap-error13[...].jpg (56Кб, 792x792)
>>988496
Я вас слушаю
Аноним 11/05/17 Чтв 00:18:49  988500
>>988498
Когда на работу устраиваться будешь?
Аноним 11/05/17 Чтв 00:32:26  988510
14855370239130.jpg (218Кб, 676x905)
>>988500
Ты же видишь мой уровень, какая работа. Самому стыдно, что такой хуйней занимаюсь. Не знаю чего хочу, потерянный человек

Дворником бы работать и Хайдеггера с Шопенгауэром читать

26 лвл
Аноним 11/05/17 Чтв 00:43:53  988514
>>988510
Блин, ты прямо как я.
Аноним 11/05/17 Чтв 00:47:40  988517
>>988465

Подожди
Аноним 11/05/17 Чтв 00:48:47  988518
2.jpg (188Кб, 1157x1078)
>>988514
Все лучше, чем доту играть. Но это не точно
Аноним 11/05/17 Чтв 00:50:29  988520
>>988518
Точнее не бывает.
Аноним 11/05/17 Чтв 01:48:20  988536
>>988493
Можно ли так создавать зависимости в конструкторе?
Аноним 11/05/17 Чтв 01:55:16  988538
>>988536
Почему бы нет
Аноним 11/05/17 Чтв 01:58:37  988539
>>988538
Я просто сам так леплю объекты везде, всегда задумывался правильно ли, может стоит Dependency injection покурить, в тему ли это вообще
Аноним 11/05/17 Чтв 01:58:57  988540
Есть какая нибудь одна прога, которая все функции среды выполнит. Что бы не качать шторм, виртуалку отдельно, ну и вот это вот все
Аноним 11/05/17 Чтв 02:08:49  988542
>>988539
не знаю, не мой лвл
делаю, как в ларавел увидел
Аноним 11/05/17 Чтв 02:10:15  988543
>>988542
Зависит от логики. В конструкторе самое необходимое, если без ди, без чего жить нельзя. Все остальное по мере надобности. Либо сразу с ди делать.
Аноним 11/05/17 Чтв 02:52:32  988546
>>988298
Сиди и жди блядь ОПа, а я жду перекат.
Аноним 11/05/17 Чтв 02:56:16  988547
Алсо, гугл хром параша для разработки по локалке? Он не обновляет стили в реал тайме, только после удаления истории(хэша видимо), а вместе с ним сносятся авторизации на других сайтах, нахуй он нужен? В опере без пердолинга обновляется.
Аноним 11/05/17 Чтв 03:00:18  988548
>>988547
ctrl + f5. Такая же хуита
Аноним 11/05/17 Чтв 08:49:08  988607
>>988548
>>988547
<!--
Ребята не стоит вскрывать этот код. Вы молодые, шутливые, вам все легко.
Это не то.
Это не Чикатило и даже не архивы спецслужб. Сюда лучше не лезть.
Серьезно, любой из вас будет жалеть. Лучше закройте код и забудьте что тут писалось.
Я вполне понимаю что данным сообщением вызову дополнительный интерес, но хочу сразу предостеречь пытливых - стоп.
Остальные просто не найдут...
-->
Аноним 11/05/17 Чтв 08:57:17  988612
>>988547
Если тебе нужно обновление стилей динамически, то юзай нужный софт, например Brackets, плагин для Atom.
В хроме можно дебажить свои стили, смотреть, что генерируется и редактировать.
Аноним 11/05/17 Чтв 08:58:36  988613
Как думаете, посоны, что делать если у тебя с одной базой работают одновременно несколько клиентов? Они же там изменения делают каждый в реальном времени, невозможно держать актуальные данные у всех одновременно! Вот допустим я открыл один набор данных, за мной Вася, так пока он читает, я уже начал изменять его, а когда он решит изменить, ему полезут ошибки: нет такой записи, невозможное количество и т.п. А если еще несколько человек подключится, это ж анархия начнется. Шо делать то?
Аноним 11/05/17 Чтв 09:54:10  988636
>>988612
Да нахуй реал тайм, это наоборот не убодно, не видишь изменений, я про F5, обновляется только хтмл-разметка, стили из памяти браузера подгружаются.
Аноним 11/05/17 Чтв 11:05:56  988666
>>988613
Во-первых, возможно. Но это тема для другой парадигмы разработки, тут обычно не обсуждается.
Во-вторых, все так живут и не парятся. Это нормальная ситуация. В особо критичных по этой теме местах перекрывается частичным реалтаймом (чаты например). В остальных ошибка - значит ошибка, это правильная реакция.
Аноним 11/05/17 Чтв 11:55:51  988700
>>988666
>Но это тема для другой парадигмы разработки, тут обычно не обсуждается.
Какой? Где обсуждается?
Аноним 11/05/17 Чтв 11:58:48  988703
>>988700
Это зависит от того, что ты разрабатываешь. Если ты пилишь сайтик с реалтаймом, что соотвествует тематике треда, то следует в сторону вебсокетов посмотреть как следует, либо на паб/саб сервисы (или свои реализации), и тут уже хорошенько так башкой подумтаь. Почему дургая парадигма? Потому что разработка в этом стиле (ивент драйвен) сильно отилчается от того, что мы обычно на сайтах видим (мвс, контроллеры эти и эндпоинты). Грубо говоря начнеш ьпилить так - тебе мало поможет опыт в стандартном мвс. По этой же причине и обсуждается редко.
Аноним 11/05/17 Чтв 11:59:33  988704
>>988703
Обсуждается, мб, в жс треде. Но это не точно, я там не сижу.
Аноним 11/05/17 Чтв 12:04:07  988709
>>988703
Ага, спасибо за ответ. Поспрашиваю в жс треде.
Аноним 11/05/17 Чтв 12:17:02  988716
>>988709
Ну, это я с потолка сказал.
В жс треде возможно обсуждают сокет.ио просто (кстати это пиздец блевотня, но пока что самая быстрая блевотня из аналогов, препэар йор ебало фор грабли, доки можешь даже не читать, там ни слова правды нет). На пхп тоже есть решения, ратчет например, просто они медленнее (но удобнее притом на порядок и сука прозрачнее), тут есть люди, которые с подобным работали. Паб/саб дрисня не зависит от языка, реализаций миллиард, но такой подход покрывает не все задачи хорошо, сокеты более гибкие в этом плане. А вообще ты зря вскрываешь эту тему. Если тебе прямо ойой реалтайм не нужен, а просто стандартный жизненный цикл веб-приложения требуется, и тебя запарило, что может где-то у кого-то запись пропадет, то это пустая причина для беспокойства, ибо является нормой, просто обрабатывай эти ситуации. Не спеши в это дерьмо лезть, еще успеешь потонуть.
Аноним 11/05/17 Чтв 13:26:44  988745
У меня сегодня после обновления винды (10), перестал правильно работать XDebug в PHPStorm'е. Он останавливается на брейкпоинте, но далее, при нажатии Step Over/Into выполнение продолжается так, как будто я нажал Run.
При этом оно продолжается не правильно, а скрипт тупо падает на каком-нибудь вызове функции, типа попытка обратиться к null. И падает он независимо от того, что я нажму после точки останова (F7, F8, F9).

Кто-нибудь сталкивался с такой проблемой?
Аноним 11/05/17 Чтв 13:43:14  988759
>>988745
UPD. Нет, оно идёт не так, как будто я нажал Run, если там поставить трай кэтчи, поставить точку останова внутри трая, и после остановки на ней нажать шаг, то оно сразу выбрасывает на строчку ЗА кэтчем. И далее опять та же история.
Аноним 11/05/17 Чтв 13:58:04  988772
>>988759
UPD. Сменил версию PHP с 7 на 5.6 (использую OpenServer), тут работает. Но всё равно интересно, что случилось на седьмой такого.
https://github.com/enotocode/minesweeper.mvc Аноним 11/05/17 Чтв 14:02:23  988782
Привет, не прошло и года...

https://github.com/enotocode/minesweeper.mvc
online test https://enotocode.github.io/minesweeper.mvc/

>>Можешь прочитать про системы модулей в JS: CommonJS и AMD. Затем - про сборщики и загрузчики модулей вроде WebPack.
>>Советую настроить сборщик так, чтобы в режиме разработки не требовалась бы сборка и файлы бы грузились напрямую (это упрощает и ускоряет отладку), а в режиме релиза собирался бы единый JS файл.

Поставил webpack.
Модули node и commonjs имеют одинаковый синтаксис? Есть ли экспорт по умолчанию в commonjs: module.export = MyClass или это только фишка node модулей?
Как я понял, webpack не умеет грузить напрямую, зато есть режим watch, который обновляет сборку в режиме реального времени, и несколько видов карты файлов.

>>В классе MinesweeperGame, возможно, лучше было бы сделать 2 словаря:

>>- cells[y][x] с объектами Cell с информацией о клеточке
>>- openedCells[y][x] с true/false, показывает, открыта ли клетка
>>- markedCells[y][x] true/false

>>Можно сделать даже один словарь, если сделать у ячейки свойства isOpened/isMarked, но тогда их можно случайно поменять, и MinesweeperGame об этом не узнает (и не сгенерирует событие). Но можно конечно договориться так не делать.
>>Или можно сделать Cell тоже источником событий, и подписываться на событие открытия, но это конечно немного усложнит жизнь.

>>А у тебя этого нет, у тебя одна и та же клеточка может быть представлена 2 разными объектами в minedCells и openCells, это, мне кажется, нехорошо. Ведь тогда, если мы хотим что-то у клеточки поменять, мы должны найти все экземпляры и поменять нужное свойство. Лучше делать, что 1 сущности соответствует ровно 1 экземпляр объекта.

Я сделал один словарь cells[y][x] с объектами Cell с информацией о клеточках. Договорился сам с собою, что свойство _cells и объекты Cell - это формат для хранения данных, доступ к которым(ому) имеет только класс MinesweeperGame, и никто снаружи не знает о их существовании. Словарь наполняется автоматически при создании игры, а класс может только изменять свойства внутри каждого Cell.

>>https://github.com/enotocode/minesweeper.mvc/blob/master/app/GameEvent.js
>>Здесь, ты может сам заметил, проблема в том, что у разных событий может быть разное число аргументов, с разными названиями. Получается, надо либо сделать разные классы событий, либо сделать один класс, а аргументы передавать словарем вроде { cell: someCell, action: SomeClass.ACTION_OPEN }

>>Использование нескольких классов требует больше кода, но позволяет ставить jsDoc, добавлять кастомные методы и свойства в события.

Добавил
- ButtonEvent // события кнопок View
- CellEvent // клетки по которым кликнули во View и клетки которые отрыли/отметили в MinesweeperGame
- StatusEvent // изменение статуса игры MinesweeperGame

>>Еще, наверно, стоило бы начать раскидывать файлы по папкам.
А как делить файлы по папкам, View/Controller/Other ?

>>https://github.com/enotocode/minesweeper.mvc/tree/master/app
>>Ну и насчет 2 независимых контроллеров (ModalController/BrowserViewController) - вот я не уверен, а не лучше ли сделать ModalController/ModalView подчиненными BrowserViewController, чтобы он им говорил, когда показать диалог, а не они сами решали?
>>А то просто у тебя получается что диалог сам решает, когда ему открываться, как-то иерархия управления размывается, это как 2 директора на одном предприятии.

Сделал так:
1. пара ModalView/ModalController создается в Controller,
2. Controller же следит за событиями модели, чтобы вызывать методы show()/hide() модального окна (но через ModalController)
3. ModalController имеет прямой доступ к модели, чтобы обрабатывать события ModalView.
4. ModalView ничего ни о ком не знает

Мне не нравится 2 пункт, т.к. Controller теперь связан еще с одним классом кроме View. И пункт 3 - если у контроллера модального окна есть доступ к модели, может быть пусть сам запускает show()/hide(), или убрать доступ, тогда Controller должен подписаться на события ModalController, или ModalController иметь доступ к методу Controller что увеличит их связанность?

>>BrowserViewController очень сильно привязан к BrowserView, оно в общем-то логично, но я поймал себя на мысли, что может быть, проще было его код во View и поместить?

Понять что происходит будет определенно проще. Но пока оставлю так.

>>ConsoleController.prototype.updateCellStatus = function(status, cell){
>>Проще наверно просто пересоздавать строку с нуля, а не мучаться с заменой отдельного символа. там все равно отрисовка этого поля потом больше времени займет.
Тогда мне нужно получать все ячейки из модели при каждой отрисовке. Т.к. я договорился что внутреннее хранилище с ячейкими не покинет модели, мне нужно сделать метод получения данных о всех ячейках, т.е. создать новый формат для передачи данных или клонировать массив со всеми классами внутри. А потом отрисовывать его?

И нужно будет перерисовывать поле каждый раз, при открытии каждой клетки, но за один ход могут открыться сразу несколько соседних, если рядом с ними нет мин. Сейчас так выглядит вывод поля в консоли браузера.

>>А так тебе приходится держать копию данных модели и синхронизировать ее постоянно. Сложно и есть риск что-то забыть.
В BrowserView тоже хранится копия данных в DOM и также происходит отрисовка, мы начинаем с поля где все ячейки не раскрыты, и при появлении события от модели открываем по одной ячейке. Хотя для добаления возможности отменить действие, вариант с полной отрисовкой надежнее. Осталось решить в каком виде получать данные о клетках из модели.

>>https://github.com/enotocode/minesweeper.mvc/tree/master/templates/css
>>Это надо поместить в public.

Public вообще пришлось убрать, чтобы не дублировать файлы, т.к. gitPages берет index.html из корневой папки проекта.

>>если за раз открывается несколько клеточек, в консоль вываливается куча изображений поля

Придумал пока два варианта как это сделать.
1. Сделать буфер для задания на отрисовку и задержку на его выполнение, каждое новое задание заменяет старое, и обновляет таймер задержки. Но как-то не надежно.
2. Можно добавить два статуса для метода модели openCell(), при запуске метода статус изменяется на OPENING, в самом конце выполнения статус на - WAITING. Функция входит в рекурсию до изменения статуса на WAITING, а в режиме рекурсии не меняет совсем. View должна иметь прямой доступ к модели или сделать этот статус свойством события открытия каждой клетки.
Аноним 11/05/17 Чтв 14:52:00  988817
Снимок.JPG (26Кб, 601x297)
По студентам поясни, списочек уже есть, пилю кнопки для перехода(пролистывания) списка студентов. Как это должно работать? Не перезагружать же мне всю страницу, что бы отобразить следующих (10) студентов, это надо в js вникать. Я не понимаю.
Аноним 11/05/17 Чтв 14:52:42  988818
>>988817
>это надо в js вникать? Я не понимаю.*
Опчик.
Аноним 11/05/17 Чтв 15:03:20  988822
>>988817
Я делал без js, задача по PHP же. Перезагружал страницу, чтобы отобразить следующих 10 студентов.
Аноним 11/05/17 Чтв 15:14:02  988826
>>988822
Ну мне просто показалось странным(диким) перезагружать страницу, но раз надо тогда ладно...(если щас начать учить жаву будет ведь каша из синтаксиса и встроенных методов в языки).
Аноним 11/05/17 Чтв 16:13:14  988852
>>988613

Тут отдельные проблемы, которые надо решать по отдельности. Вот там человек пишет советы по принципу "слышал звон", а я напишу конкретно.

"я уже начал изменять его, а когда он решит изменить, ему полезут ошибки" - для предотвращения этой проблемы можно использовать блокировки. При редактировании берется блокировка на запись, и при попытке открыть форму редактирования на другом компьютере или что-то сделать с записью выводится соответствующее сообщение.

Блокировку нужно брать только на определенное время, так как если пользователь отсоединился по каким-то причинам, запись не должна оставаться заблокированной навечно. Ну и наоборот, пока пользователь редактируети данные, надо продлевать блокировку. Даже тут возможны конфликты, если пользователь A открыл форму редактирвоания, взял блокировку, затем у него пропал интернет и блокировку взял пользователь B. Когда у A появяится соединение с сетью, он обнаружит, что блокировка потеряна и данные, возможно, изменены другим пользователем.

Кстати, блокировки бывают 2 видов: пессимистичные и оптимистичные. При пессимистичной мы берем блокировку до редактирования данных и не даем другому пользователю их редактировать. При оптимистичной - мы используем версионирование и при попытке сохранить данные обнаруживаем, что они были изменены другим пользователем и предлагаем что-нибудь сделать.

Другая проблема, что данные могут изменяться одним пользователем, пока их просматривает другой пользователь. Если так принципиально обновлять их в реальном времени, то надо делать на сервере систему трансляции обновлений. Клиент, который загружает какие-то данные, соединяется с сервером, подписывается на получение их изменений, и обновляет данные на экране при поступлении таких данных. Также, в простых случаях есть вариант поллинга - вместо возни с вебсокетами можно просто отправлять запросы на сервер раз в N секунд и спрашивать, не изменились ли данные. Разумеется, на сервере надо будет хранить для каждой записи время ее изменения или номер версии, чтобы это реализовать.

То есть здесь важно сначала проработать модель, как это все будет работать, учесть все возможные ситуации, а потом только браться за написание кода. Ну и такие вещи конечно удобно решать не на уровне отдельной формы, а сделать какой-то фреймворк, чтобы это поведение легко можно было добавить к любой форме.

>>988547

Ты даже не пытался разобраться, а уже обвиняешь Хром. А правильно ли настроены кеширующие HTTP заголовки на твоем сервере? А ставишь ли ты галочку "не использовать кеш" в инструментах разработчика. 99%, что виноват не Хром, а ты.

Для очистки кеша не требуется удалять куки и терять авторизацию. Можно очистить только кеш.

>>988540

Видимо нет. Не знаю, зачем тебе виртуальная машина конечно.

>>988493

В парадигме MVC не передают request в модель. Да и если отложить в сторону MVC, я вот сомневаюсь что модели нужен request целиком - скорее всего какие-то отдельные маленькие части.

Напомню что у меня есть урок по MVC, наверно тебе пригодится: https://github.com/codedokode/pasta/blob/master/arch/mvc.md

>>988397

Когда мы в выражении "2 + 3 - 1" доходим до плюса, мы не можем выполнить сложение, так как не прочитали еще число 3. Потому мы сохраняем операцию в переменную на будущее, читаем второй аргумент, и когда дойдем до минуса, выполняем ранее сохраненную операцию.
Аноним 11/05/17 Чтв 16:13:39  988853
>>988327

Да, интерпретатор PHP - это программа, написанная в основном на языке Си. Для описания правил используется специальный язык вроде Yacc (я могу ошибаться в названии), который придуман специально для описания грамматик. То есть правила пишут на языке Yacc, а он из них генерирует код на Си, который анализирует текст программы по этим правилам.

Вообще, какой элемент компьютера выполняет программы? Это процессор, CPU. Процессор - это по сути калькулятор, который работает по программе, хранящейся в памяти. Процессор "понимает" только один язык - машинные коды. Это числа, каждое из которых обозначает определенную операцию. Ну например, там есть коды, которые обозначают команды вроде "поместить число N в регистр A", "прибавить к содержимому регистра A число X", "поместить число X в ячейку памяти Y". В памяти компьютера хранятся эти коды, процессор читает их по одному и выполняет.

То есть команды процессора очень примитивные. Потому машинные коды называют "низкоуровневым" языком. Чтобы просто посчитать какое-то математическое выражение вроде $x + $y, ты должен разложить его на отдельные операции (взять число из памяти, поместить в регистр процессора A, взять второе число, поместить в регистр B, сложить регистры A и B, поместить результат в память) и записать в виде кодов.

Приведу другой пример: в процессоре нет команды вывода текста на экран - это слишком сложно для него. Чтобы вывести букву на экран, мы должны записывать определенные числа в определенные ячейки памяти, каждая из которых управляет состоянием отдельного пикселя на экране. То есть, чтобы вывести букву "A" на экран, нужно выполнить сотни машинных команд (делая определенные пиксели экрана темными или светлыми).

Понятно, что вручную писать эти коды - долго, и неудобно. Потому люди придумали множество высокоуровневых языков программирования, на которых можно писать более понятный человеку код. Ну например, $x = $x + 1; или echo "A". Такой код удобен для человека, но непонятен процессору. Он не может его выполнить. И есть 2 способа решить эту проблему - компиляция и интерпретация.

При компиляции программа-компилятор разбирает текст высокоуровневой программы и преобразует ее в машинные коды, выдавая на выходе исполняемый файл (под Windows это будет exe-файл, под линуксом - файл формата ELF). Затем ты можешь запустить этот файл. Когда ты запускаешь exe-файл, операционная система (Windows) загружает хранящиеся в нем машинные коды с диска в память и процессор их выполняет.

Если ты хочешь что-то поменять в исходном коде - надо запускать компиляцию заново и генерировать новый исполняемый файл.

Такой подход используют языки Си, Го, Раст.

Другой подход - интерпретация. При этом подходе мы запускаем программу-интерпретатор, которая принимает на вход высокоуровневый код, разбирает его и выполняет инструкции в нем по очереди. В отличие от предыдущего подхода, тут не создается исполняемый файл, не генерируются машинные коды и не требуется перекомпиляция при правке кода. Но, как правило, с интепретатором код выполняется медленнее, так как он выполняется не напрямую процессором, а интерпретируется другой программой.

PHP использует второй подход, то есть интерпретирует код. Но сам интепретатор PHP написан на языке Си, который использует подход с компиляцией. На Гитхабе выложен исходный код интерпретатора PHP ( https://github.com/php/php-src ) на языке Си. Разработчики PHP компилируют этот код в исполняемый exe-файл и выкладывают на свой сайт, откуда все его скачивают.

То есть получается что интерпретатор php - это программа в машинных кодах, сгенерированных из исходного кода на Си. И эта программа анализирует и выполняет программу на языке PHP, которую ты написал. Такая вот система.

Кроме PHP, интепретацию используют языки Яваскрипт, Питон, Руби.

Также, если тебе интересно, есть еще смешанный подход - интерпретатор интепретирует программу, но если какой-то ее кусочек выполняется много раз, то он компилирует этот кусочек в машинные коды и выполняет их напрямую на процессоре ради повышения быстродействия. Это называется JIT - компиляция кода "на лету". Если ты используешь Хром, то в нем есть интерпретатор языка Яваскрипт, который содержит в себе JIT-компилятор.

И есть еще один вариант, когда высокоуровневая программа компилируется, но не в машинные коды процессора, а просто в какие-то условные коды (байткод), которые интерпретирует программа под названием "виртуальная машина". То есть это подход, который содержит и компиляцию в коды, и интепретацию этих кодов виртуальной машиной. Зачем такие сложности? Дело в том, что машинные коды у разных процессоров разные. Программа, скопилированная под процессор смартфона, не запустится на процессоре настольного компьютера. А программа, скопилированная для 64-битного процессора, не запустится на 32-битном. Компиляция в байткод (который не привязан к конкретной модели процессора) позволяет решить эту проблему. Также, байткод может содержать более высокоуровневые операции, чем те, что выполняет процессор. Такой код работает чуть медленнее, чем скомпилированный в настоящий машинный код, но быстрее чем при интерпретации. Этот подход используют языки Ява и C# (и другие дотнет-языки).

Код на Си работает быстрее, чем на PHP, но ему требуется компиляция перед выполнением, а также язык Си не такой высокоуровневый, как PHP и написание программы на нем займет больше времени. Также, он сложнее и требует больше времени для изучения.
Аноним 11/05/17 Чтв 16:14:06  988854
>>988308

Для разных случаев подходят разные языки.

Программы под Андроид обычно пишут на Яве (но есть варианты использования HTML/CSS/JS)
Под iOS и мак - на Swift, Objective-C
Под Windows - обычно на Си++ или Си. Иногда на Яве или C#.
Под веб - на PHP, Питоне, Руби, Яве, Яваскрипте, Го

>>988303

Толсто же. Для обсуждения жизни есть другие треды и доски. А наш тред как раз для вопросов от начинающих.

>>988069

Перекатывать я стараюсь на 700-800 постах, чтобы по возможности ответить на все вопросы в треде. И мне не принципиально, чтобы наш тред был всегда на первой странице, зачем? Я и так знаю, что он очень популярный и посещаемый.

А проблем промотать до 2-3 страницы я не вижу. Также можно просто добавить тред в закладки или не закрывать вкладку с ним.

>>987901

В этой задаче ты должен придумать какой-то алгоритм. То есть описать как-то, что должен делать компьютер, чтобы проверить, является слово палиндромом или нет. Ты должен разбить этот алгоритм на отдельные действия и уже после этого писать программу.

И не пропускай задачи, я бы тебе советовал потом попросить у ОПа дополнительную задачку на циклы и строки.
Аноним 11/05/17 Чтв 16:14:42  988855
>>987599

> О какой настройке идёт речь? Можно же и так ссылаться на любой домен в html ссылке.

Я имел в виду, если ты выносишь картинки на отдельный домен, то прописать этот домен в конфиге, чтобы его можно было легко менять, не трогая код. А также чтобы можно было на локальной версии сайта использовать локальный домен.

> Ничего, просто хотелось иметь красивую ссылку.
Так вроде никто и не делает, кроме каких-то случаев адской SEO оптимизации. Форма ведь не генерирует красивые URL.

> Значит не получиться обойтись без посредников. Это печально, потому что регистраторы с первой страницы гугла выглядят как очередные корпорации зла. Хотелось бы обойтись без них.
Можно сделать свою доменнную зону, правда это дороговато... Но вообще, тут есть причины: администратор зоны не очень хочет заморачиваться с продажами, техподдержкой - ему проще дать доступ к API и получать деньги. Ну и как плюс, один регистратор может работать с многими зонами, давать какие-то дополнительные услуги вроде DNS-хостинга.

Если тебя беспокоит цена, можно поискать какие-то дешевые зоны, есть даже бесплатные домены (вроде .tk), но там какие-то свои подвохи. Бывают бесплатные домены 3-го уровня.


>>987588

> SELECT FROM table WHERE x <> 1
> Индекс не поможет.
Верно

> SELECT FROM table WHERE x + y < 100
> Индекс на x не будет использоваться. Это потому, что индекс построен для X, а мы запрашиваем X+Y? Затрудняюсь провести аналогию со словарём.
Чтобы индекс использовался, поле должно стоять отдельно (x < 100), а не как часть выражения. В других БД, вроде Oracle или Postgres, кстати, есть функциональные индексы - индексы по результату выражения. Там можно было бы построить индекс по выражению x + y и использовать его: https://postgrespro.ru/docs/postgrespro/9.5/indexes-expressional.html

> SELECT MAX(a) FROM table WHERE b = 2
> Индекс на b поможет.
Да, он поможет, но можно сделать лучше. Индекс по b позволит быстро найти все записи с b = 2, но далее СУБД будет обходить их по одной, ища запись с наибольшим значением a. А что, если их миллион? Тут лучше поможет индекс по паре (b, a). В нем при b = 2 записи будут выстроены по возрастанию a, и мы можем просто взять последнюю запись у которой b = 2, в ней a будет наибольшее.

Если записать:

b | a
1 100
2 1
2 5
2 100
3 1

Тут MySQL может быстро найти в индексе начало и конец диапазона где b = 2, и взять MAX(a) из последней записи "2 100". Хотя конечно это стоило бы проверить с помощью EXPLAIN.

Также, если бы мы написали запрос WHERE b = 2 AND a <= 5 то он оптимизируется так: MySQL найдет в индексе строку "2 5" и будет шагать вверх (в сторону убывания a), пока b = 2. То есть он возьмет записи "2 5", затем "2 1", затем увидит "1 100" и остановится, так как b больше не равно 2. Или может, наоборот, он найдет строку "2 1" и будет шагать от нее вперед по индексу, пока одно из условий (b = 2 или a <= 5) не нарушится. Это по сути то же самое.

> SELECT FROM table WHERE name LIKE '%Иван%'
> Для такого запроса индекс по name не поможет, всё равно что искать в словаре слова с неизвестной первой буквой. Как понял, должен помочь FULLTEXT индекс и операторы AGAINST/MATCH
FULLTEXT не поможет (по моему) при поиске по части слова, только по полному слову или началу слова.

> SELECT FROM table WHERE b = 1 AND a < 10
Тут надо разобраться как работают составные индексы пол нескольким полям. В индексе (x, y, z) записи выстраиваются по возрастанию x, при равном x - по возрастанию y, при равных x и y - по возрастанию z.

Такой индекс позволяет искать по условиям:

x = ?
x = ? AND y = ?
x < ?
x = ? AND y < ?
x = ? AND y BETWEEN ? AND ?

MAX(y) WHERE x = ?

и так далее.

Тут нам нужен индекс, где записи, подпадающие под условие b = 1 AND a < 10 будут идти одним блоком, так что можно найти в индексе точку где b = 1, a = 10, и шагать от нее в сторону убывания a, пока b не поменяет свое значение (перестанет быть равной 1).

> Но потом он добавляет индекс gender_age и пишет, что теперь запросы будет использовать полный индекс. Выходит такой индекс
У тебя неправильный индекс, так как male99 должен идти между male29 и female15, а не в конце списка. Если ты построишь индекс правильно, то увидишь что для выполнения запроса достаточно найти в индексе запись male29 и шагать от нее к началу индекса, до тех пор, пока первое поле все еще равно male.
https://github.com/grigoryMovchan/AphorismCMS/ Аноним 11/05/17 Чтв 16:16:05  988856
>>987135

Немного глянул https://github.com/grigoryMovchan/AphorismCMS/

Вообще, я бы тебе советовал почитать комментарии к задаче о студентах в ОП посте. Так как не хочется по второму разу их пересказывать.

У тебя есть папка public - почему тогда index.php вне нее? Ты по моему не понял идею, там идея в том, что public должна быть корневой папкой веб-сервера, и код лежит снаружи нее.

Папка vendor не должна быть в репозитории (ее надо добавить в гитигнор), зато в него стоит положить composer.lock.

https://github.com/grigoryMovchan/AphorismCMS/blob/master/db.sql#L25
> ENGINE=MyISAM
Почитай сравнение движков MyISAM и InnoDB

> `password_user` varchar(255) NOT NULL,
Тут нужен комментарий, что там хранится. Надеюсь, соленый хеш пароля, а не сам пароль в открытом виде?

В базе нужно проставить внешние ключи.

https://github.com/grigoryMovchan/AphorismCMS/blob/master/app/core/Route.php#L69
Тут непонимание протокола HTTP. Нельзя отдавать заголовок Location при коде 4xx. Он используется с кодом 3xx в тех случаях, когда страница переехала на другой адрес. Если страница не найдена, надо отдавать код 404. Почитай назначение кодов состояния HTTP.

Класс Route должен правильно называться FrontController.

Используемый подход к роутингу плох тем, что для одной страницы существует много URL, например /controller, /CONTROLLER, /controller/index/1/2/3, /controller/index/4/5/6 - при ошибке в URL ты можешь не заметить что ошибся, и на индексации поисковиками это плохо скажется.

Убрать query string из URL лучше с помощью parse_url().

> $url = 'Location: /' . strtolower($controllerName) . '/';
> header($url);
Тут после отдачи Location скрипт продолжает выполняться.

Также, код в Route->start() плохочитаемый, попробуй сам прочесть. Стена кода, ты бы хоть на функции ее разбил.

https://github.com/grigoryMovchan/AphorismCMS/blob/master/app/core/Request.php#L45
> public function getFeedbackString($separator = "<br>")
Класс Request не должен отвечать за вывод информации и ее оформление, генерацию HTML кода.

https://github.com/grigoryMovchan/AphorismCMS/blob/master/app/core/View.php#L8
> @param str
> @param arr
В phpdoc нет типов str и arr: https://phpdoc.org/docs/latest/guides/types.html

> public function generate($content_view, $template_view, $data = null, $error = null)
Не очень понятно, почему для $error сделан отдельный аргумент, а все остальное передается в $data.

https://github.com/grigoryMovchan/AphorismCMS/blob/master/app/core/Model.php
Этот класс выглядит сомнительно (например не очень понятно что такое в твоем понимании Model, для чего этот класс предназначен. Надо добавить комментарий перед ним). ensure() там явно не к месту, так как эта функция может быть полезна не только в модели и ее лучше сделать отдельным статическим методов в отдельном классе.

Не соблюдается PSR-4. Например неймспейс имеет вид namespace Application\Core, а папка называется core, с маленькой буквы.

https://github.com/grigoryMovchan/AphorismCMS/blob/master/app/configs/app_example.ini
Плохой пример так как не содержит пояснений и непонятно зачем там 2 секции с БД.

https://github.com/grigoryMovchan/AphorismCMS/blob/master/app/controllers/QuotesController.php#L16
> $this->data['thisPage'][0] = 'quotes';
Тут по моему ты добавляешь элемент 0 в несуществующий массив. Зачем так запутывать код?

> $this->quotes = new QuotesModel($this->request);
Непонятно зачем Модели нужен request. В MVC модели он не нужен.

https://github.com/grigoryMovchan/AphorismCMS/blob/master/app/models/QuotesModel.php#L22
> $this->dbh = new MysqlModel(ConfigModel::UNMARRIED);
Почитай мой урок про DI. https://github.com/codedokode/pasta/blob/master/arch/di.md

https://github.com/grigoryMovchan/AphorismCMS/blob/master/app/models/QuotesModel.php#L207
тут видно полное непонимание идеи MVC. Идея ведь там отделить логику приложения (модель) от взаимодействия с внешним миром (контроллер).

У тебя получилась плохая функция getQuote(). Вот допустим я хочу получить цитату с id = 7. Как ее получить, если в функцию нельзя передать id?

Получить след/пред цитату проще через WHERE id > ? ORDER BY id LIMIT 1

Также, подход что при ошибке поиска возвращается имитация цитаты, тоже плохой. Ведь при твоем подходе непонятно как определить, что цитата не найдена. Да и при ненайденной цитате логичнее отдавать 404, а не притворяться что такая страница есть.

Выборка случайного id неэффективно сделана. Погугли про оптимизацию выборки случайной записи.

https://github.com/grigoryMovchan/AphorismCMS/blob/master/app/models/MysqlModel.php
Тут непонятно на каком основании MysqlModel наследуется от Model (у них ведь ничего общего нет) и почему она вообще назвается Model. тебе надо определиться для начала, что ты называешь моделью.

https://github.com/grigoryMovchan/AphorismCMS/blob/master/app/models/MysqlModel.php#L50
Тут неудачный подход, ты зря пытаешься объединить разные функции (fetchAll, fetch) в одну - это только создает неудобства. Ну например, из-за этого функция возвращает разные типы данных, а не один определенный. Ну и непонятно, зачем это сделано, такое ощущение что ты себе жизнь усложнить пытаешься.

> без понятия зачем тут нужен новый уровень абстракции
Если без понятия, то не надо делать. Ты должен понимать каждую строчку кода. А так ты просто копируешь то, что увидел где-то, не понимая. Это какой-то культ карго. Зачем? Если ты хочешь произвести впечатление на собеседовании своим кодом, то любой опытный программист легко тебя раскусит.

https://github.com/grigoryMovchan/AphorismCMS/blob/master/app/models/AuthModel.php#L66
Тут зачем htmlspecialchars? Это же не шаблон.

Вообще AuthModel требует упорядочивания, там как-то все перемешано. ну например, операции с БД можно из него убрать в другой класс.

Функции и переменные надо назвыать единообразно, а не одну с подчеркиванием, а другую кемел кейсом.

Дальше не смотрел.

По дизайну - я не дизайнер, но кое-что бросается в глаза. У тебя на странице с цитатой самый "тяжелый" элемент, на который обращаешь внимание в первую очередь - это кнопки. Мало того, что они залиты, так еще и ярко-белым цветом, который ярче, чем текст. Но ведь главный-то элемент наверно не они, а цитата. Я бы советовал попробовать поиграть с кнопками - сделать их менее яркими или убрать белую заливку - и посмотреть что получится. Также, плашка слева от цитаты тоже выглядит тяжеловатой, может быть стоит попробовать сделать ее побледнее?

Непонятно, что обозначает третья кнопка с иконкой комментария (вторая - это случайная цитата или "поделиться"? Тоже не очень понятно).

В форме ошибки нечитаемы, белый текст на светлом фоне.


> Шаблон не осилил, верстка поехала, не смог в CSS.
Пройди наши задачи на HTML/CSS из ОП поста и наверно тогда осилишь. Также, есть заблуждение, что использование бустрапа избавляет от необходимости учить CSS (как и использование ORM избавляет от необходимости учить SQL).
Аноним 11/05/17 Чтв 16:16:41  988857
>>987128

Потому что в PHP применение ++ к строке с буквами вызвает изменение букв (ставится следующая в алфавите буква вместо последней). Описано тут: http://php.net/manual/ru/language.operators.increment.php

>>988817

Можно перезагружать страницу, почему нет? Ну и роботам, поисковикам с такими сайтами работать проще.

>>988745

Вряд ли тут дело в Windows.
Аноним 11/05/17 Чтв 16:19:13  988858
>>988852
>Также, в простых случаях есть вариант поллинга - вместо возни с вебсокетами можно просто отправлять запросы на сервер раз в N секунд и спрашивать, не изменились ли данные
Он оказывается менее эффективным, чем сокеты для тех же целей. Для таких простых случаев реализация простая и там и там.
>пишет советы по принципу "слышал звон"
А вот тут иди нахуй. Я больше года работаю над распределенными реалтаймовыми сервисами, заебался как черт, потому и отговариваю понемногу вообще от затеи этой, если ему не горит.
Test Аноним 11/05/17 Чтв 16:35:40  988866
Тестовое сообщение не обращайте внимания.

Добро пожаловать в наш уютный тред. Тут мы изучаем язык PHP (а также JS/CSS/HTML/SQL), решаем задачки и даже делаем простые сайты! Зачем? Кто-то хочет научиться программировать, кто-то - делать сайты, кто-то - просто размять мозги и заняться чем-то полезным.

Пожалуйста, пишите один большой пост вместо нескольких маленьких и не флудите не по теме. ОПу ведь все это читать придется.

Это тред для начинающих. Не написал за свою жизнь ни одной программы и имеешь тройку по математике? Ты наш человек.

Устанавливать пока что ничего не требуется, разве что редактор кода вроде Sublime Text 3, Notepad++, Visual Studio Code, Netbeans PHP или PhpStorm (с ним будет удобнее).

Предыдущий тред был тут: >>966608 (OP) ( http://arhivach.org/thread/254710/ )


Что самое главное для программиста? Умение аккуратно оформлять код (читай второй пост, прежде чем писать код).

Правила: ведем себя воспитанно, помогаем новичкам, постим ссылки на решения задачек, ОП их проверяет и дает советы и замечания. ОП заходит редко, где-то раз в 2-3 дня, у него мало времени, не жди его, решай задачки дальше. ОП отвечает на все вопросы по его задачкам и учебнику, а вот насчет каких-то других вещей - только если останется время. Но в треде немало анонимных экспертов разного уровня, так что вряд ли вопрос останется без ответа.

У нас есть уроки по основам PHP, они собраны и выложены по адресу http://archive-ipq-co.narod.ru/ Это учебник для изучающих с нуля, то есть если ты вообще ничего не знаешь, то надо начать с него. Он простой и понятный (по крайней мере в начале). Там есть задачи, их надо решать обязательно (чтобы стать программистом, надо писать код — иначе никак). Пости ссылки на решения в тред, мы их проверим, напишем замечания и дадим советы по улучшению.

Если не знаешь как решать, запости код, напиши в каком месте остановился и попроси подсказку.

Ты прошел весь учебник? Молодец, но это были лишь основы языка PHP, этого недостаточно. Вот что в идеале надо изучить еще: ООП, как работает веб-сервер, HTML/CSS, SQL, PDO, работа с таблицами в БД, работа с формами, MVC, git, composer, JS, фреймворки, автоматизированное тестирование.

Надо переходить к более серьезным задачкам, которые научат тебя всему этому.

- для начала прочти урок https://github.com/codedokode/pasta/blob/master/soft/web-server.md
- установи Апач + PHP (советы выше и ниже) и читай туториал http://php.net/manual/ru/tutorial.php
- Учи HTML/CSS и SQL, PDO, хотя бы основы
- Далее простая, но полезная задача сделать список студентов, в ней много полезных советов: https://github.com/codedokode/pasta/blob/master/student-list.md
- Более сложная задача сделать файлообменник на микрофреймворке Slim: https://gist.github.com/codedokode/9424217
- Еще более сложная и долгая задача на Yii/Symfony: https://gist.github.com/codedokode/8733007
- После нее можно изучать автоматизированное тестирование https://gist.github.com/codedokode/a455bde7d0748c0a351a
- Если ты все решил, переходи к Symfony 2/Doctrine 2
- Почитать про паттерны http://designpatternsphp.readthedocs.org/ru/latest/README.html (если ты не изучил ни одного фреймворка, то это будет рановато), тут с примерами кода http://designpatternsphp.readthedocs.org/ru/latest/README.html . Имей в виду что без примеров использования их учить бесполезно - не поймешь, хочешь увидеть примеры использования паттернов - ковыряй исходники Симфони, например Symfony Forms. Не заучивай паттерны - смотри код и думай, зачем тут они использованы.

Чтобы делать эти задания, тебе надо установить Апач + PHP (можно заодно сразу и MySQL) на компьютер. Вот полезные инструкции:

https://github.com/codedokode/pasta/blob/master/soft/php-install.md
https://github.com/codedokode/pasta/blob/master/soft/apache-install.md

Может тебе понадобится пользоваться командной строкой, вот гайд https://github.com/codedokode/pasta/blob/master/soft/cli.md

Решения задач лучше показать мне, особенно на ООП,так как сам ты вряд ли увидишь все ошибки. Пости свой код на гитхаб и вкидывай ссылку в тред по мере решения. Я прокомментирую и укажу на ошибки.

Также, у нас есть задачи которые позволят тебе изучить или подтянуть до нормального уровня знания JS/HTML/CSS/SQL. Решай их параллельно с задачами выше.

- HTML/CSS: https://github.com/codedokode/pasta/blob/master/html/html.md
- JS: https://gist.github.com/codedokode/ce30e7a036f18f416ae0
- SPA (сложно): https://github.com/codedokode/pasta/blob/master/js/spa.md
- Проверялка решений на JS: http://dkab.github.io/jasmine-tests/
- MySQL: https://github.com/codedokode/pasta/blob/master/db/databases.md

Что почитать

- Мануал по PHP — http://www.php.net/manual/ru/langref.php
- Сайт phptherightway (перевод на русский: http://getjump.me/ru-php-the-right-way/ )
- По PHP: Профессиональное программирование на PHP Джордж Шлосснейгл
- По PHP: Мэтт Зандстра — PHP: Объекты, шаблоны, методики программирования
- JS: learn.javascript.ru
- Про Git: https://git-scm.com/book/ru/v1

Оформляй код аккуратно!!! — например пропусти через phpformatter.com . Также, если ты пользуешься IDE вроде PhpStorm, Netbeans, Eclipse, то в них эта опция встроена, подробнее: https://gist.github.com/codedokode/8759492

У ОПа нет аккаунтов и групп вконтакте, в фейсбуке, в твиттере, все "пхп-треды" там поддельные.

Платиновые вопросы

- Почему PHP? Потому что фейсбук и википедия на нем написаны, и вакансий море, и учить легко.
- Сайт опять упал!!!!! — Не паникуй, а открой http://rghost.ru/6bfCY9lfl и получи личную немного устаревшую оффлайновую копию сайта (можно читать хоть на андроиде без интернета)
- Что надо знать чтобы найти работу - разработчику: PHP, SQL, HTML/CSS, JS, ООП, Git, композер, MVC, фреймворк. Верстальщику - HTML/CSS, JS, jQuery
- Можно подробнее про поиск работы, собеседования - нет, ОП писать не будет, но может кто из анонов захочет рассказать. Поищите тред перезвонивших, а также раздел /wrk/.
- Сколько времени надо изучать все это? - все зависит от тебя, но не меньше 6-8 месяцев
- Посоветуйте редактор кода - Sublime Text 3, Notepad++, PhpStorm
- Нужен ли ООП, фреймворки, MVC, git, composer? — Да, однозначно. Посмотри любую вакансию.
- Что самое главное для программиста? Умение аккуратно оформлять код.
- ОП, сделай за меня мою работу или домашнее задание? — Это конечно, хорошая идея, но нет.
- Подскажи сайты для поиска работы, я не умею гуглить? — hh.ru, geekjob.ru, moikrug.ru (склеен с brainstorage.me), fl.ru, upwork.com (бывший одеск). Имей в виду, что кроме фриланса есть еще постоянная удаленная работа (remote job) когда тебе не надо тратить время на поиск заказов и переговоры с неадекватными заказчиками.
Тестовое сообщение Аноним 11/05/17 Чтв 16:36:31  988867
Еще предыдущие треды ищутся в гугле по словам "клуб php" или в архиваче. Еще есть такой архив тредов: http://phpclub.rf.gd/

Мейлач лежит? Есть запасной тред:
Аноним 11/05/17 Чтв 16:41:06  988872
Переходите в новый тред: >>988868 (OP)

-----

Кому я не ответил - напомните о себе в новом треде. enotocode, твой сегодняшний пост вижу, можно не напоминать.
Аноним 11/05/17 Чтв 20:37:35  988966
14922771007610.jpg (36Кб, 409x405)
>>988493
>>988856
Спасибо за код ревью

>>988493-кун
Аноним 12/05/17 Птн 23:15:47  989673
>>988855
Спасибо за понятный и доходчивый ответ по индексам. А можно ещё задачу, где нужно подумать над проставлением индексов? Ты вроде раньше кому-то предлагал БД для борды проектировать, но я, к сожалению, не смог этот пост найти c полным текстом задачи.
Аноним 13/05/17 Суб 12:43:14  989900
Где перекат?
Аноним 13/05/17 Суб 12:54:14  989902
Снимок.JPG (16Кб, 720x83)
14292767895161.png (266Кб, 680x680)
Заебала блядь руснявая хуйня. НУ ЧЕ ОПЯТЬ? КОДИРОВОЧКА?
Засунул Собакину в $_SESSIONS['last_name'], достал Собакину на другой странице $last_name = $_SESSIONS['last_name'].
И ПОЛУЧИЛ ВОПРОСИКИ)))
Я просто за сегодня порвался с таких нелепых проблем.
Кругом выставил header("Content-Type: text/html; charset=utf-8"); Даже маме своей позонил и сказал: "ХЕАДЕР ЧАРСЕТ УТФ-8".
Скачал нотепад, проверил BOM, НИГДЕ НЕТ ЕГО, НИ НА ОДНОЙ БЛЯДСКОЙ СТРАНИЦЕ. Пхп-сторм не даёт удалить BOM, потому что его НЕТ.
СТАРТ

<?php
session_start();

В самой первой строке, самого первого файла, фронт-котроллер, что бы ты не делал - всё начинается с него.

Потушите мою Собакину.

Как достать Сабакину из массива Сессии?
Аноним 14/05/17 Вск 00:26:54  990177
>>989900
>>989673

Этот тред закрыт. Переходите в новый тред >>988868 (OP)

Здесь на вопросы никто отвечать больше не будет.
Аноним 02/06/17 Птн 18:18:34  1000160
>>989902
Коля, ты просто мудак, смирись.

[Назад][Обновить тред][Вверх][Каталог] [Реквест разбана] [Подписаться на тред] [ ] 987 | 104 | 247
Назад Вверх Каталог Обновить

Топ тредов
Избранное